2015 AIME II Math Jam
Go back to the Math Jam ArchiveAoPS instructors discuss all 15 problems of the 2015 AIME II.
Copyright © 2024 AoPS Incorporated. This page is copyrighted material. You can view and print this page for your own use, but you cannot share the contents of this file with others.
Facilitator: Jeremy Copeland
copeland
2015-03-27 18:39:40
This classroom is moderated. That means you won't be able to see the things you type. We do see those things, however, so don't worry. The Math Jam will begin at around 7pm ET, 4pm PT.
This classroom is moderated. That means you won't be able to see the things you type. We do see those things, however, so don't worry. The Math Jam will begin at around 7pm ET, 4pm PT.
Guendabiaani
2015-03-27 18:52:22
Hello
Hello
copeland
2015-03-27 18:52:24
Hi!
Hi!
1023ong
2015-03-27 18:52:59
I'm so pumped for this!
I'm so pumped for this!
copeland
2015-03-27 18:53:00
Me too! I just fell out of my chair.
Me too! I just fell out of my chair.
nosaj
2015-03-27 18:53:28
How long does it take for AoPS to prepare the Math Jam?
How long does it take for AoPS to prepare the Math Jam?
copeland
2015-03-27 18:53:29
About 20 hours of work, maybe?
About 20 hours of work, maybe?
copeland
2015-03-27 18:54:02
I lied, actually. I don't have a chair. I had one once but I don't know where it went off to.
I lied, actually. I don't have a chair. I had one once but I don't know where it went off to.
nosaj
2015-03-27 18:54:24
Well, thanks for all the hard work!
Well, thanks for all the hard work!
beanielove2
2015-03-27 18:54:24
Wow! Thanks for doing this!
Wow! Thanks for doing this!
copeland
2015-03-27 18:54:25
Of course, but don't get the wrong impression. I'm not big on "working hard."
Of course, but don't get the wrong impression. I'm not big on "working hard."
vinayak-kumar
2015-03-27 18:54:50
which did you think was harder, aime i or aime ii?
which did you think was harder, aime i or aime ii?
copeland
2015-03-27 18:54:51
I don't want to speculate on things like that. This one was harder for me since I was in charge of it, though.
I don't want to speculate on things like that. This one was harder for me since I was in charge of it, though.
nosaj
2015-03-27 18:55:41
Why does Richard never lead Math Jams anymore?
Why does Richard never lead Math Jams anymore?
copeland
2015-03-27 18:55:42
He does the classes ones. It all depends on this and that. This year he didn't do any because of the site launch.
He does the classes ones. It all depends on this and that. This year he didn't do any because of the site launch.
Dayranger
2015-03-27 18:56:35
Yay! I love math.
Yay! I love math.
copeland
2015-03-27 18:56:36
Gosh, this is the right place for you.
Gosh, this is the right place for you.
hlasker1
2015-03-27 18:56:59
Richard did do the AIME I Math Jam
Richard did do the AIME I Math Jam
copeland
2015-03-27 18:57:01
Dave did that one.
Dave did that one.
unskillfulnoob
2015-03-27 18:57:21
Who exactly is copeland?
Who exactly is copeland?
copeland
2015-03-27 18:57:23
That's me! I'm the School Director here at AoPS.
That's me! I'm the School Director here at AoPS.
copeland
2015-03-27 18:57:33
My name's Jeremy Copeland.
My name's Jeremy Copeland.
unskillfulnoob
2015-03-27 18:59:08
like your qualifications, and do you do or come up with the imo problems or something?
like your qualifications, and do you do or come up with the imo problems or something?
copeland
2015-03-27 18:59:10
Oh.
Oh.
copeland
2015-03-27 18:59:30
I was an undergrad at Reed college, Ph.D. from University of Chicago. I was an instructor at MIT for 3 years before joining AoPS.
I was an undergrad at Reed college, Ph.D. from University of Chicago. I was an instructor at MIT for 3 years before joining AoPS.
copeland
2015-03-27 18:59:58
I don't write IMO problems. I've written several USAMTS problems if you find that interesting.
I don't write IMO problems. I've written several USAMTS problems if you find that interesting.
ImpossibleCube
2015-03-27 19:00:07
Is that a unicorn in your avatar? http://www.artofproblemsolving.com/community/user/66362
Is that a unicorn in your avatar? http://www.artofproblemsolving.com/community/user/66362
copeland
2015-03-27 19:00:10
Yes.
Yes.
copeland
2015-03-27 19:00:14
He loves math, too.
He loves math, too.
unskillfulnoob
2015-03-27 19:01:30
lastly, do you work in a room at home by yourself or is it like you in a building and there are other aops people with you near you right now?
lastly, do you work in a room at home by yourself or is it like you in a building and there are other aops people with you near you right now?
copeland
2015-03-27 19:01:32
I'm standing among 15-20 other AoPSers at HQ right now.
I'm standing among 15-20 other AoPSers at HQ right now.
Tommy2000
2015-03-27 19:02:02
standing?
standing?
copeland
2015-03-27 19:02:03
(See above)
(See above)
Rmehtany
2015-03-27 19:02:30
where is HQ
where is HQ
copeland
2015-03-27 19:02:31
Sunny San Diego.
Sunny San Diego.
1023ong
2015-03-27 19:02:59
Greetings other AoPSers!
Greetings other AoPSers!
henryweng
2015-03-27 19:02:59
is it start?
is it start?
copeland
2015-03-27 19:03:05
Hey, how about we start?
Hey, how about we start?
C-bass
2015-03-27 19:03:22
Hello!
Hello!
shiningsunnyday
2015-03-27 19:03:22
Good morning from China guys
Good morning from China guys
henryweng
2015-03-27 19:03:22
good!
good!
C-bass
2015-03-27 19:03:22
YEEEEEEEEEEEEEEEEEEEEEES
YEEEEEEEEEEEEEEEEEEEEEES
jamesxie
2015-03-27 19:03:22
yay
yay
xayy
2015-03-27 19:03:22
yay
yay
MathStudent2002
2015-03-27 19:03:22
Yeah!
Yeah!
RedDevil
2015-03-27 19:03:22
perfect!
perfect!
AlcumusGuy
2015-03-27 19:03:22
great idea!
great idea!
Dayranger
2015-03-27 19:03:22
great idea
great idea
bhildak
2015-03-27 19:03:22
yeah
yeah
Tommy2000
2015-03-27 19:03:22
Sure
Sure
copeland
2015-03-27 19:03:25
Alright.
Alright.
copeland
2015-03-27 19:03:31
The classroom is moderated, meaning that you can type into the classroom, but these comments will not go directly into the room.
The classroom is moderated, meaning that you can type into the classroom, but these comments will not go directly into the room.
copeland
2015-03-27 19:03:33
Please do not ask about administrative aspects of the contests, and please do not ask me to speculate about the results. I have no idea what the index will be for the USAJMO or the USAMO.
Please do not ask about administrative aspects of the contests, and please do not ask me to speculate about the results. I have no idea what the index will be for the USAJMO or the USAMO.
copeland
2015-03-27 19:03:37
Welcome to the 2014 AIME II Math Jam!
Welcome to the 2014 AIME II Math Jam!
copeland
2015-03-27 19:03:38
I'm Jeremy Copeland and I'll be leading tonight's discussion.
I'm Jeremy Copeland and I'll be leading tonight's discussion.
copeland
2015-03-27 19:03:41
Before we get started I would like to take a moment to explain our virtual classroom to those who have not previously participated in a Math Jam or one of our online classes.
Before we get started I would like to take a moment to explain our virtual classroom to those who have not previously participated in a Math Jam or one of our online classes.
copeland
2015-03-27 19:03:47
The classroom is moderated, meaning that students can type into the classroom, but these comments will not go directly into the room. These comments go to the instructors, who may choose to share your comments with the room.
The classroom is moderated, meaning that students can type into the classroom, but these comments will not go directly into the room. These comments go to the instructors, who may choose to share your comments with the room.
copeland
2015-03-27 19:03:52
This helps keep the class organized and on track. This also means that only well-written comments will be dropped into the classroom, so please take time writing responses that are complete and easy to read.
This helps keep the class organized and on track. This also means that only well-written comments will be dropped into the classroom, so please take time writing responses that are complete and easy to read.
copeland
2015-03-27 19:04:12
There are a lot of students here! As I said, only a relatively small fraction of the well-written comments will be passed to the entire group. Please do not take it personally if your responses do not get posted, and please do not complain about it. I expect this Math Jam to be much larger than our typical class, so please be patient with me---there are quite a few of you here tonight!!
There are a lot of students here! As I said, only a relatively small fraction of the well-written comments will be passed to the entire group. Please do not take it personally if your responses do not get posted, and please do not complain about it. I expect this Math Jam to be much larger than our typical class, so please be patient with me---there are quite a few of you here tonight!!
copeland
2015-03-27 19:04:17
Also, we won't be going through the math quite as thoroughly as we do in our classes -- I can't teach all the necessary material for every problem as we go. Another difference between tonight and our regular online classes is that it is very unlikely that we'll be able to answer every single question you ask. We always to try do so in our regular online classes, but we have a large number of students tonight! So, please go ahead and ask questions, but also please understand if we aren't able to answer them all!
Also, we won't be going through the math quite as thoroughly as we do in our classes -- I can't teach all the necessary material for every problem as we go. Another difference between tonight and our regular online classes is that it is very unlikely that we'll be able to answer every single question you ask. We always to try do so in our regular online classes, but we have a large number of students tonight! So, please go ahead and ask questions, but also please understand if we aren't able to answer them all!
copeland
2015-03-27 19:04:28
We do have two teaching assistants with us tonight to help answer your questions: Luis (Duelist) and Alex (BOGTRO).
We do have two teaching assistants with us tonight to help answer your questions: Luis (Duelist) and Alex (BOGTRO).
copeland
2015-03-27 19:04:34
Luis Ares took many math contests in high school, and still enjoys solving math problems as well as playing ultimate frisbee in his spare time.
Luis Ares took many math contests in high school, and still enjoys solving math problems as well as playing ultimate frisbee in his spare time.
BOGTRO
2015-03-27 19:04:49
Hi!
Hi!
copeland
2015-03-27 19:04:50
BOGTRO is Alexander Katz, better known in some parts as "Alex the Kat", a three-time olympiad qualifier who will attend MIT next year. When not doing math, Alex busies himself with chess, programming, and creating his own attempts at competition writing.
BOGTRO is Alexander Katz, better known in some parts as "Alex the Kat", a three-time olympiad qualifier who will attend MIT next year. When not doing math, Alex busies himself with chess, programming, and creating his own attempts at competition writing.
Duelist
2015-03-27 19:05:12
Hi!
Hi!
FractalMathHistory
2015-03-27 19:05:20
Ares the god of war!
Ares the god of war!
copeland
2015-03-27 19:05:21
Katz, the god of, um,. . .
Katz, the god of, um,. . .
copeland
2015-03-27 19:05:29
They can answer questions by whispering to you or by opening a window with you to chat 1-on-1. However, due to the large size of the session tonight, they may not be able to get to you right away (or at all). Repeating your question over and over is more likely to annoy us than to get it answered faster, so please, just ask your question once and be patient, and please understand that we may not be able to answer all the questions tonight.
They can answer questions by whispering to you or by opening a window with you to chat 1-on-1. However, due to the large size of the session tonight, they may not be able to get to you right away (or at all). Repeating your question over and over is more likely to annoy us than to get it answered faster, so please, just ask your question once and be patient, and please understand that we may not be able to answer all the questions tonight.
copeland
2015-03-27 19:05:32
Please also remember that the purpose of this Math Jam is to work through the solutions to AIME problems, and not to merely present the answers. "Working through the solutions" includes discussing problem-solving tactics. Also on occasion we may stop to prove things that you wouldn't necessary need to prove while doing the contest. So please, when a question is posted, do not simply respond with the final answer. That's not why we're here. We're going to work through the problems step-by-step, and comments that skip key steps or jump ahead in the problem, without providing explanation or motivation, won't be acknowledged.
Please also remember that the purpose of this Math Jam is to work through the solutions to AIME problems, and not to merely present the answers. "Working through the solutions" includes discussing problem-solving tactics. Also on occasion we may stop to prove things that you wouldn't necessary need to prove while doing the contest. So please, when a question is posted, do not simply respond with the final answer. That's not why we're here. We're going to work through the problems step-by-step, and comments that skip key steps or jump ahead in the problem, without providing explanation or motivation, won't be acknowledged.
copeland
2015-03-27 19:05:43
Let's get started! We're going to work through all 15 problems from the 2014 AIME II, in order.
Let's get started! We're going to work through all 15 problems from the 2014 AIME II, in order.
ThePiPie
2015-03-27 19:06:01
2015...?
2015...?
copeland
2015-03-27 19:06:04
Hm.
Hm.
copeland
2015-03-27 19:06:10
Which one do you want? 2014 or 2015?
Which one do you want? 2014 or 2015?
swirlykick
2015-03-27 19:06:33
2015
2015
Rmehtany
2015-03-27 19:06:33
2015
2015
acegikmoqsuwy2000
2015-03-27 19:06:33
2016.
2016.
derpyuniverse
2015-03-27 19:06:33
2015 lol
2015 lol
urmilla
2015-03-27 19:06:33
2015
2015
Tommy2000
2015-03-27 19:06:33
2016
2016
awesome123
2015-03-27 19:06:33
isn't it the 2015 AIME II
isn't it the 2015 AIME II
Jyzhang12
2015-03-27 19:06:33
2015!
2015!
caressezhu
2015-03-27 19:06:33
2015
2015
jamesxie
2015-03-27 19:06:33
2015!
2015!
amburger66
2015-03-27 19:06:33
2015 would be nice
2015 would be nice
epiclucario
2015-03-27 19:06:33
2016 please
2016 please
Abeymom
2015-03-27 19:06:33
2015
2015
mathperson9
2015-03-27 19:06:33
2015
2015
copeland
2015-03-27 19:06:39
Should have voted for '16.
Should have voted for '16.
copeland
2015-03-27 19:06:42
Let's get started! We're going to work through all 15 problems from the 2015 AIME II, in order.
Let's get started! We're going to work through all 15 problems from the 2015 AIME II, in order.
copeland
2015-03-27 19:06:44
1. Let $N$ be the least positive integer that is both 22 percent less than one integer and 16 percent greater than another integer. Find the remainder when $N$ is divided by 1000.
1. Let $N$ be the least positive integer that is both 22 percent less than one integer and 16 percent greater than another integer. Find the remainder when $N$ is divided by 1000.
copeland
2015-03-27 19:06:52
How do we write "$N$ is 22 percent less than one integer"?
How do we write "$N$ is 22 percent less than one integer"?
Gina
2015-03-27 19:07:15
N = .78x
N = .78x
ompatel99
2015-03-27 19:07:15
N=0.78*m
N=0.78*m
acegikmoqsuwy2000
2015-03-27 19:07:15
N=0.78x
N=0.78x
Aang
2015-03-27 19:07:15
N=0.78x
N=0.78x
shiningsunnyday
2015-03-27 19:07:15
0.78X
0.78X
epiclucario
2015-03-27 19:07:15
.78x=N
.78x=N
beanielove2
2015-03-27 19:07:15
$N=0.78m$, where $m$ is an integer
$N=0.78m$, where $m$ is an integer
copeland
2015-03-27 19:07:18
There is some integer $a$ such that $N=0.78a$.
There is some integer $a$ such that $N=0.78a$.
copeland
2015-03-27 19:07:21
How do we write "$N$ is 16 percent more than another integer"?
How do we write "$N$ is 16 percent more than another integer"?
AlisonH
2015-03-27 19:07:41
N=1.16y
N=1.16y
ompatel99
2015-03-27 19:07:41
N=1.16k
N=1.16k
vinayak-kumar
2015-03-27 19:07:41
N=1.16x$
N=1.16x$
WhaleVomit
2015-03-27 19:07:41
$N=1.16b$
$N=1.16b$
nosaj
2015-03-27 19:07:41
$N=1.16b$
$N=1.16b$
gxah
2015-03-27 19:07:41
N=1.16b
N=1.16b
Rmink41
2015-03-27 19:07:41
N=1.16y
N=1.16y
cxiong
2015-03-27 19:07:41
N=1.16y
N=1.16y
copeland
2015-03-27 19:07:43
There is some integer $b$ such that $N=1.16b$.
There is some integer $b$ such that $N=1.16b$.
copeland
2015-03-27 19:07:47
Therefore \[0.78a=1.16b\] or $78a=116b$. Those are both even so we really have
Therefore \[0.78a=1.16b\] or $78a=116b$. Those are both even so we really have
copeland
2015-03-27 19:07:49
$39a=58b$.
$39a=58b$.
copeland
2015-03-27 19:07:53
What does that tell us?
What does that tell us?
Darn
2015-03-27 19:08:28
We must have that $29$ and $39$ both divide $N$.
We must have that $29$ and $39$ both divide $N$.
Jyzhang12
2015-03-27 19:08:28
Multiple of 58 and 39
Multiple of 58 and 39
cxiong
2015-03-27 19:08:28
b is a multiple of 39, a is a multiple of 58
b is a multiple of 39, a is a multiple of 58
amburger66
2015-03-27 19:08:28
a is a multiple of 58, while b is a multiple of 39
a is a multiple of 58, while b is a multiple of 39
jam10307
2015-03-27 19:08:28
divisible by 39 and 58
divisible by 39 and 58
copeland
2015-03-27 19:08:31
Since 39 and 58 are coprime, $a$ is a multiple of 58 and $b$ is a multiple of 39. We can write $a=58c$ and $b=39c$.
Since 39 and 58 are coprime, $a$ is a multiple of 58 and $b$ is a multiple of 39. We can write $a=58c$ and $b=39c$.
copeland
2015-03-27 19:08:41
What is $N$ in terms of $c$?
What is $N$ in terms of $c$?
copeland
2015-03-27 19:09:54
Be careful that you're actually using an expression for $N$ here.
Be careful that you're actually using an expression for $N$ here.
acegikmoqsuwy2000
2015-03-27 19:10:01
N=0.78*58c=1.16*39c
N=0.78*58c=1.16*39c
ompatel99
2015-03-27 19:10:01
N=29*39c/25
N=29*39c/25
henryweng
2015-03-27 19:10:01
N=58*0.78c
N=58*0.78c
swirlykick
2015-03-27 19:10:01
.78*58c
.78*58c
firemike
2015-03-27 19:10:01
N = .78 * 58c
N = .78 * 58c
Dayranger
2015-03-27 19:10:01
0.78x58c
0.78x58c
copeland
2015-03-27 19:10:04
We have \[N=0.78a=\frac{78}{100}\cdot58c=\frac{39\cdot29}{25}\cdot c.\]
We have \[N=0.78a=\frac{78}{100}\cdot58c=\frac{39\cdot29}{25}\cdot c.\]
copeland
2015-03-27 19:10:06
What is the smallest possible value for $N$?
What is the smallest possible value for $N$?
xayy
2015-03-27 19:10:39
39*29
39*29
AlisonH
2015-03-27 19:10:39
1131
1131
C-bass
2015-03-27 19:10:39
39 x 29
39 x 29
ZZmath9
2015-03-27 19:10:39
39*29=1131, when c is 25
39*29=1131, when c is 25
swirlykick
2015-03-27 19:10:39
39*29=1131 so 131
39*29=1131 so 131
xayy
2015-03-27 19:10:39
39*29 or 1131
39*29 or 1131
ompatel99
2015-03-27 19:10:39
39*29=1131
39*29=1131
WhaleVomit
2015-03-27 19:10:39
39*29
39*29
AlcumusGuy
2015-03-27 19:10:41
39*29 = 1131
39*29 = 1131
copeland
2015-03-27 19:10:43
The smallest integer multiple of $\dfrac{39\cdot29}{25}$ is \[39\cdot29=(40-1)(30-1)=1200-40-30+1=1131.\]
The smallest integer multiple of $\dfrac{39\cdot29}{25}$ is \[39\cdot29=(40-1)(30-1)=1200-40-30+1=1131.\]
copeland
2015-03-27 19:10:44
This multiple is achieved at $c=25$ so $a=58c$ and $b=39c$ are both integers.
This multiple is achieved at $c=25$ so $a=58c$ and $b=39c$ are both integers.
copeland
2015-03-27 19:10:47
The final answer is $\boxed{131}$.
The final answer is $\boxed{131}$.
copeland
2015-03-27 19:11:01
Kind of a tough problem to start with. Let's make up for that. . .
Kind of a tough problem to start with. Let's make up for that. . .
copeland
2015-03-27 19:11:02
2. In a new school 40 percent of the students are freshmen, 30 percent are sophomores, 20 percent are juniors, and 10 percent are seniors. All freshmen are required to take Latin, and 80 percent of the sophomores, 50 percent of the juniors, and 20 percent of the seniors elect to take Latin. The probability that a randomly chosen Latin student is a sophomore is $\frac mn$, where $m$ and $n$ are relatively prime positive integers. Find $m+n$.
2. In a new school 40 percent of the students are freshmen, 30 percent are sophomores, 20 percent are juniors, and 10 percent are seniors. All freshmen are required to take Latin, and 80 percent of the sophomores, 50 percent of the juniors, and 20 percent of the seniors elect to take Latin. The probability that a randomly chosen Latin student is a sophomore is $\frac mn$, where $m$ and $n$ are relatively prime positive integers. Find $m+n$.
copeland
2015-03-27 19:11:07
Not much to this problem. What do we do?
Not much to this problem. What do we do?
maverick8
2015-03-27 19:11:51
WLOG, let there be 100 students
WLOG, let there be 100 students
cxiong
2015-03-27 19:11:51
assume the school has 1000 people
assume the school has 1000 people
Jyzhang12
2015-03-27 19:11:51
Assume total students is 100
Assume total students is 100
WhaleVomit
2015-03-27 19:11:51
assume there are 100 people in the school
assume there are 100 people in the school
vinayak-kumar
2015-03-27 19:11:51
assume there are 100 students
assume there are 100 students
gxah
2015-03-27 19:11:51
let there be 100 students
let there be 100 students
copeland
2015-03-27 19:11:52
We could do that. It's a good approach.
We could do that. It's a good approach.
amburger66
2015-03-27 19:12:04
find out the total number of Latin students, and how many of them are sophomores
find out the total number of Latin students, and how many of them are sophomores
24iam24
2015-03-27 19:12:04
find percent of all people who take latin
find percent of all people who take latin
shiningsunnyday
2015-03-27 19:12:04
Count what we want/Total
Count what we want/Total
swirlykick
2015-03-27 19:12:04
Find out how many total people take latin, and how many are sophmores
Find out how many total people take latin, and how many are sophmores
copeland
2015-03-27 19:12:15
And my favorite technique:
And my favorite technique:
burunduchok
2015-03-27 19:12:17
make a chart
make a chart
copeland
2015-03-27 19:12:19
We divide the number of Latin sophomores by the total number of Latin students.
We divide the number of Latin sophomores by the total number of Latin students.
copeland
2015-03-27 19:12:22
I love tables. Let's make a table of the percents in each category.
I love tables. Let's make a table of the percents in each category.
copeland
2015-03-27 19:12:24
copeland
2015-03-27 19:12:25
(Note that all of these values are in the unit "percent of students".)
(Note that all of these values are in the unit "percent of students".)
copeland
2015-03-27 19:12:28
What's the answer?
What's the answer?
Poseidon2001
2015-03-27 19:13:13
25
25
Tommy2000
2015-03-27 19:13:13
6/19
6/19
henryweng
2015-03-27 19:13:13
=24/76=6/19
=24/76=6/19
maverick8
2015-03-27 19:13:13
25
25
Rocksolid
2015-03-27 19:13:13
025
025
cxiong
2015-03-27 19:13:13
24/76 = 6/19
24/76 = 6/19
caressezhu
2015-03-27 19:13:13
25
25
nosaj
2015-03-27 19:13:13
24/76=6/19
24/76=6/19
xayy
2015-03-27 19:13:13
24/76=6/19 or 25
24/76=6/19 or 25
ShadowQueenPeach
2015-03-27 19:13:13
6/19
6/19
copeland
2015-03-27 19:13:22
The probability we are looking for is
\[
\dfrac{\text{Latin sophomores}}{\text{total Latin students}}=
\dfrac{24}{40+24+10+2}=\dfrac{24}{76}=\dfrac{6}{19}.\] The answer is $6+19=\boxed{25}$.
The probability we are looking for is
\[
\dfrac{\text{Latin sophomores}}{\text{total Latin students}}=
\dfrac{24}{40+24+10+2}=\dfrac{24}{76}=\dfrac{6}{19}.\] The answer is $6+19=\boxed{25}$.
copeland
2015-03-27 19:13:39
Don't worry about AIMEifying at the end. That step is (usually) pretty simple.
Don't worry about AIMEifying at the end. That step is (usually) pretty simple.
copeland
2015-03-27 19:13:50
6/19 is a much clearer answer for this. I know you can mostly add.
6/19 is a much clearer answer for this. I know you can mostly add.
copeland
2015-03-27 19:13:55
3. Let $m$ be the least positive integer divisible by 17 whose digits sum to 17. Find $m$.
3. Let $m$ be the least positive integer divisible by 17 whose digits sum to 17. Find $m$.
copeland
2015-03-27 19:14:08
Any suggestions?
Any suggestions?
maverick8
2015-03-27 19:14:44
m is a 3 digit number because 89 and 98 don't work
m is a 3 digit number because 89 and 98 don't work
math-rules
2015-03-27 19:14:44
since 89 and 98 don't work, let m = 100a+10b+c
since 89 and 98 don't work, let m = 100a+10b+c
C-bass
2015-03-27 19:14:44
Find numbers whose digits sum to 17, since this is the AIME and the only possible numbers are either 2- or 3- digit #s
Find numbers whose digits sum to 17, since this is the AIME and the only possible numbers are either 2- or 3- digit #s
nosaj
2015-03-27 19:14:44
You know that $m$ can't be four digits because this is the AIME. :P
You know that $m$ can't be four digits because this is the AIME. :P
mathperson9
2015-03-27 19:14:44
If m's digits are xyz, then x+y+z=17 and 100x+10y+z mod 17 is 0
If m's digits are xyz, then x+y+z=17 and 100x+10y+z mod 17 is 0
shiningsunnyday
2015-03-27 19:14:44
Put in the form 100a+10b+c
Put in the form 100a+10b+c
copeland
2015-03-27 19:14:47
The only possible 2-digit numbers whose digits sum to 17 are 89 and 98, but neither of those are multiples of 17.
The only possible 2-digit numbers whose digits sum to 17 are 89 and 98, but neither of those are multiples of 17.
copeland
2015-03-27 19:14:48
So $m$ is going to be a 3-digit number. (Or more, but $m$ is also an AIME answer!)
So $m$ is going to be a 3-digit number. (Or more, but $m$ is also an AIME answer!)
copeland
2015-03-27 19:14:50
We'll probably want to name the digits. So let's say $m$ is the number $abc$, where $a$ is the hundreds digit, $b$ is the tens digit, and $c$ is the units digit.
We'll probably want to name the digits. So let's say $m$ is the number $abc$, where $a$ is the hundreds digit, $b$ is the tens digit, and $c$ is the units digit.
copeland
2015-03-27 19:15:00
Now we can write $m = 100a + 10b + c$.
Now we can write $m = 100a + 10b + c$.
copeland
2015-03-27 19:15:10
But what else do we know about $m$?
But what else do we know about $m$?
owm
2015-03-27 19:15:53
a+b+c=17
a+b+c=17
Rmehtany
2015-03-27 19:15:53
a+b+c =17
a+b+c =17
flyrain
2015-03-27 19:15:53
a+b+c=17
a+b+c=17
C-bass
2015-03-27 19:15:53
a + b + c = 17
a + b + c = 17
dhruv
2015-03-27 19:15:53
a+b+c=17
a+b+c=17
mssmath
2015-03-27 19:15:53
a+b+c=17
a+b+c=17
copeland
2015-03-27 19:16:06
The digits of $m$ sum to 17, so $a+b+c = 17$.
The digits of $m$ sum to 17, so $a+b+c = 17$.
xayy
2015-03-27 19:16:35
divisible by 17
divisible by 17
Dayranger
2015-03-27 19:16:35
m = 17n
m = 17n
Poseidon2001
2015-03-27 19:16:35
divisible by 17
divisible by 17
copeland
2015-03-27 19:16:37
$m$ is a multiple of 17, so $100a + 10b + c = 17n$ for some positive integer $n$.
$m$ is a multiple of 17, so $100a + 10b + c = 17n$ for some positive integer $n$.
copeland
2015-03-27 19:16:45
So we have the system of equations
\begin{align*}
100a + 10b + c &= 17n, \\
a+b+c &= 17.
\end{align*}
So we have the system of equations
\begin{align*}
100a + 10b + c &= 17n, \\
a+b+c &= 17.
\end{align*}
copeland
2015-03-27 19:16:46
Now what?
Now what?
Rmehtany
2015-03-27 19:17:12
subtract them
subtract them
ompatel99
2015-03-27 19:17:12
Get rid of c
Get rid of c
Tommy2000
2015-03-27 19:17:12
hmm, subtract
hmm, subtract
math-rules
2015-03-27 19:17:12
subtract second from first
subtract second from first
mathperson9
2015-03-27 19:17:12
SUbtract
SUbtract
mssmath
2015-03-27 19:17:12
Subtract!
Subtract!
Aang
2015-03-27 19:17:12
Eliminate
Eliminate
owm
2015-03-27 19:17:12
subtract?
subtract?
sicilianfan
2015-03-27 19:17:12
subtract them
subtract them
copeland
2015-03-27 19:17:16
We can subtract the equations and get rid of $c$. That leaves us with $99a + 9b = 17(n-1)$.
We can subtract the equations and get rid of $c$. That leaves us with $99a + 9b = 17(n-1)$.
copeland
2015-03-27 19:17:16
How does that help?
How does that help?
Tommy2000
2015-03-27 19:17:54
n is 1 mod 9
n is 1 mod 9
Rmehtany
2015-03-27 19:17:54
n-1 divisble by 9
n-1 divisble by 9
Darn
2015-03-27 19:17:54
We know that n-1 is divisible by 9
We know that n-1 is divisible by 9
mxgo
2015-03-27 19:17:54
We know n-1 is divisble by 9
We know n-1 is divisble by 9
jam10307
2015-03-27 19:17:54
n is 10+9x
n is 10+9x
copeland
2015-03-27 19:17:57
Factoring the left side gives $9(11a+b) = 17(n-1)$.
Factoring the left side gives $9(11a+b) = 17(n-1)$.
copeland
2015-03-27 19:18:00
This means that $n-1$ must be a multiple of 9, or equivalently that $n$ is 1 plus a multiple of 9. Therefore $17n$ is $17$ plus a multiple of $17\cdot9=153$.
This means that $n-1$ must be a multiple of 9, or equivalently that $n$ is 1 plus a multiple of 9. Therefore $17n$ is $17$ plus a multiple of $17\cdot9=153$.
copeland
2015-03-27 19:18:07
Know the answer yet?
Know the answer yet?
dhruv
2015-03-27 19:18:45
476
476
maverick8
2015-03-27 19:18:45
476
476
caressezhu
2015-03-27 19:18:45
476=17*28
476=17*28
burunduchok
2015-03-27 19:18:45
476
476
gxah
2015-03-27 19:18:45
476
476
24iam24
2015-03-27 19:18:45
476
476
ompatel99
2015-03-27 19:18:45
476
476
maverick8
2015-03-27 19:18:45
$ 476 $
$ 476 $
ZZmath9
2015-03-27 19:18:45
the answer is 17 + 3 * 153 = 17 + 459 = 476.
the answer is 17 + 3 * 153 = 17 + 459 = 476.
C-bass
2015-03-27 19:18:45
476
476
copeland
2015-03-27 19:18:48
It's easy to finish just by checking by hand, by starting at $10\cdot17 = 170$ and adding $9\cdot17 = 153$ until we get one that has a digit sum of 17.
It's easy to finish just by checking by hand, by starting at $10\cdot17 = 170$ and adding $9\cdot17 = 153$ until we get one that has a digit sum of 17.
copeland
2015-03-27 19:18:50
$10\cdot17 = 170$, nope
$10\cdot17 = 170$, nope
copeland
2015-03-27 19:18:54
$19\cdot17 = 170+153 = 323$, nope
$19\cdot17 = 170+153 = 323$, nope
copeland
2015-03-27 19:18:57
$28\cdot 17 = 323+153 = 476$, success!
$28\cdot 17 = 323+153 = 476$, success!
copeland
2015-03-27 19:18:58
The answer is $\boxed{476}$.
The answer is $\boxed{476}$.
copeland
2015-03-27 19:19:09
A fifth of the way done! (By some metric.)
A fifth of the way done! (By some metric.)
Darn
2015-03-27 19:19:25
you could've taken a modular approach
you could've taken a modular approach
copeland
2015-03-27 19:19:26
True.
True.
copeland
2015-03-27 19:19:51
17 and 9 are both interesting in some way. However that doesn't turn out to help as much as all that for this particular problem.
17 and 9 are both interesting in some way. However that doesn't turn out to help as much as all that for this particular problem.
copeland
2015-03-27 19:19:59
4. In an isosceles trapezoid, the parallel bases have lengths $\log 3$ and $\log 192$, and the altitude to those bases has length $\log 16$. the perimeter of the trapezoid can be written in the form $\log 2^p3^q$, where $p$ and $q$ are positive integers. Find $p + q$.
4. In an isosceles trapezoid, the parallel bases have lengths $\log 3$ and $\log 192$, and the altitude to those bases has length $\log 16$. the perimeter of the trapezoid can be written in the form $\log 2^p3^q$, where $p$ and $q$ are positive integers. Find $p + q$.
copeland
2015-03-27 19:20:15
(The base of the log probably doesn't matter here since they didn't tell us. In fact it won't.)
(The base of the log probably doesn't matter here since they didn't tell us. In fact it won't.)
copeland
2015-03-27 19:20:18
I sense that this is one of those problems that is a lot easier than it looks as long as you know your logarithm facts.
I sense that this is one of those problems that is a lot easier than it looks as long as you know your logarithm facts.
copeland
2015-03-27 19:20:20
What do we do first?
What do we do first?
maverick8
2015-03-27 19:20:33
Draw a diagram, of course
Draw a diagram, of course
Darn
2015-03-27 19:20:33
Draw a diagram
Draw a diagram
shiningsunnyday
2015-03-27 19:20:33
Diagram!
Diagram!
simon1221
2015-03-27 19:20:33
diagram!!
diagram!!
tdeng
2015-03-27 19:20:33
diagram
diagram
math-rules
2015-03-27 19:20:33
draw a diagram
draw a diagram
Dracae8
2015-03-27 19:20:33
diagram
diagram
cxiong
2015-03-27 19:20:33
draw the diagram
draw the diagram
Rocksolid
2015-03-27 19:20:33
picture
picture
copeland
2015-03-27 19:20:37
Draw a diagram, of course. This is obviously not to scale but the labels are going to fit.
Draw a diagram, of course. This is obviously not to scale but the labels are going to fit.
copeland
2015-03-27 19:20:38
copeland
2015-03-27 19:20:40
Now what?
Now what?
Jyzhang12
2015-03-27 19:21:13
Draw a picture and add 2 heights
Draw a picture and add 2 heights
24iam24
2015-03-27 19:21:13
drop altitudes
drop altitudes
simon1221
2015-03-27 19:21:13
make 2 triangles
make 2 triangles
Jyzhang12
2015-03-27 19:21:13
Draw 2 heights
Draw 2 heights
AlcumusGuy
2015-03-27 19:21:13
draw altitudes
draw altitudes
ompatel99
2015-03-27 19:21:13
Drop altitudes from the endpoints of the top base to the bottom base
Drop altitudes from the endpoints of the top base to the bottom base
Darn
2015-03-27 19:21:13
Draw the altitude to make two right triangles
Draw the altitude to make two right triangles
hlasker1
2015-03-27 19:21:13
make a right triangle
make a right triangle
sicilianfan
2015-03-27 19:21:13
form a right triangle by dropping altitude from one of the top vertices to the base
form a right triangle by dropping altitude from one of the top vertices to the base
MathStudent2002
2015-03-27 19:21:13
Two right triangles and a rectangle
Two right triangles and a rectangle
rt03
2015-03-27 19:21:13
Make two right triangles on the ends
Make two right triangles on the ends
copeland
2015-03-27 19:21:15
Let's drop some altitudes from the vertices.
Let's drop some altitudes from the vertices.
copeland
2015-03-27 19:21:16
copeland
2015-03-27 19:21:20
And the base tells us that $a+\log3+a=\log192$. What is $a$?
And the base tells us that $a+\log3+a=\log192$. What is $a$?
Darn
2015-03-27 19:21:48
So $a=\log 8$
So $a=\log 8$
cxiong
2015-03-27 19:21:48
log8
log8
maverick8
2015-03-27 19:21:48
log 8
log 8
dantx5
2015-03-27 19:21:48
log 8
log 8
countingarithmetic
2015-03-27 19:21:48
log 8
log 8
NextEinstein
2015-03-27 19:21:48
log 8
log 8
azmath333
2015-03-27 19:21:48
log 8
log 8
simon1221
2015-03-27 19:21:48
log 8?
log 8?
BPM14
2015-03-27 19:21:48
log8
log8
Abeymom
2015-03-27 19:21:48
log 8
log 8
sicilianfan
2015-03-27 19:21:48
log 8
log 8
copeland
2015-03-27 19:21:50
$a=\frac12(\log192-\log3)=\frac12\log64=\log8$.
$a=\frac12(\log192-\log3)=\frac12\log64=\log8$.
copeland
2015-03-27 19:21:52
copeland
2015-03-27 19:21:53
Very not to scale.
Very not to scale.
copeland
2015-03-27 19:21:56
Now what?
Now what?
Darn
2015-03-27 19:22:39
We can split the logs and we find that it's a 3-4-5 right triangle, so $h= 5\log 2 = \log 32$.
We can split the logs and we find that it's a 3-4-5 right triangle, so $h= 5\log 2 = \log 32$.
math-rules
2015-03-27 19:22:39
we have 3-4-5 triangle (scaled by log 2)
we have 3-4-5 triangle (scaled by log 2)
Dracae8
2015-03-27 19:22:39
3-4-5 triangle!
3-4-5 triangle!
Tommy2000
2015-03-27 19:22:39
3-4-5 triangle so h=log32
3-4-5 triangle so h=log32
simon1221
2015-03-27 19:22:39
the triangles are 3 4 5 * log2
the triangles are 3 4 5 * log2
24iam24
2015-03-27 19:22:39
log8=3log2 log16=4log2 so 3-4-5 right triangle and h=5log2
log8=3log2 log16=4log2 so 3-4-5 right triangle and h=5log2
vinayak-kumar
2015-03-27 19:22:39
the sides are 3-4-5 triangles
the sides are 3-4-5 triangles
ompatel99
2015-03-27 19:22:39
log8=3log2. log16=4log2. h=5log2
log8=3log2. log16=4log2. h=5log2
azmath333
2015-03-27 19:22:39
use 3-4-5 right triangles
use 3-4-5 right triangles
copeland
2015-03-27 19:22:42
We could use the Pythagorean Theorem right away, but we can also simplify those lengths:
We could use the Pythagorean Theorem right away, but we can also simplify those lengths:
copeland
2015-03-27 19:22:44
copeland
2015-03-27 19:22:47
That's a 3-4-5 triangle:
That's a 3-4-5 triangle:
copeland
2015-03-27 19:22:48
copeland
2015-03-27 19:22:55
What is the perimeter?
What is the perimeter?
tdeng
2015-03-27 19:23:40
16log2+2log3
16log2+2log3
Rmink41
2015-03-27 19:23:40
16log2+2log3
16log2+2log3
nosaj
2015-03-27 19:23:40
Now just add: $16 \log 2 + 2\log 3$.
Now just add: $16 \log 2 + 2\log 3$.
firemike
2015-03-27 19:23:40
16log2 + 2log3
16log2 + 2log3
mathperson9
2015-03-27 19:23:40
16log2+2log3
16log2+2log3
Dayranger
2015-03-27 19:23:40
16 log 2 + 2 log 3
16 log 2 + 2 log 3
temp8909
2015-03-27 19:23:40
16log2+2log3
16log2+2log3
ShadowQueenPeach
2015-03-27 19:23:49
log (2^16 * 3^2)
log (2^16 * 3^2)
flyrain
2015-03-27 19:23:49
log 3+log 192+10 log 2= log 3^2*2^16 so 018
log 3+log 192+10 log 2= log 3^2*2^16 so 018
amburger66
2015-03-27 19:23:49
log (192*3*32*32) = log (2^16*3^2)
log (192*3*32*32) = log (2^16*3^2)
copeland
2015-03-27 19:23:55
The perimeter is
\[5\log2+\log3+5\log2+3\log2+\log3+3\log2=16\log2+2\log3=\log2^{16}3^2.\]
The perimeter is
\[5\log2+\log3+5\log2+3\log2+\log3+3\log2=16\log2+2\log3=\log2^{16}3^2.\]
copeland
2015-03-27 19:23:56
And the final answer?
And the final answer?
AlisonH
2015-03-27 19:24:21
18
18
Poseidon2001
2015-03-27 19:24:21
18
18
xayy
2015-03-27 19:24:21
018
018
maverick8
2015-03-27 19:24:21
eighteen
eighteen
Rmehtany
2015-03-27 19:24:21
18
18
slenderestman
2015-03-27 19:24:21
18
18
koteswari
2015-03-27 19:24:21
18
18
mxgo
2015-03-27 19:24:21
18
18
mathwizard888
2015-03-27 19:24:21
18
18
linmichael
2015-03-27 19:24:21
18
18
mathperson9
2015-03-27 19:24:21
18
18
xiaodongxi
2015-03-27 19:24:21
18
18
copeland
2015-03-27 19:24:23
The final answer is $16+2=\boxed{18}$.
The final answer is $16+2=\boxed{18}$.
copeland
2015-03-27 19:24:24
By the way, to scale that diagram looks like this:
By the way, to scale that diagram looks like this:
copeland
2015-03-27 19:24:25
1023ong
2015-03-27 19:24:46
that one doesn't look as pretty
that one doesn't look as pretty
copeland
2015-03-27 19:24:52
Thanks. I like mine better, too.
Thanks. I like mine better, too.
copeland
2015-03-27 19:24:56
This one is easier to solve.
This one is easier to solve.
copeland
2015-03-27 19:24:58
5. Two unit squares are selected at random without replacement from an $n\times n$ grid of unit squares. Find the least positive integer $n$ such that the probability that the two selected squares are horizontally or vertically adjacent is less than $\frac 1{2015}$.
5. Two unit squares are selected at random without replacement from an $n\times n$ grid of unit squares. Find the least positive integer $n$ such that the probability that the two selected squares are horizontally or vertically adjacent is less than $\frac 1{2015}$.
copeland
2015-03-27 19:25:03
In probability problems I usually like to count the denominator first.
In probability problems I usually like to count the denominator first.
copeland
2015-03-27 19:25:10
How many ways are there to select 2 squares?
How many ways are there to select 2 squares?
IsabeltheCat
2015-03-27 19:25:55
n^2 choose 2
n^2 choose 2
tdeng
2015-03-27 19:25:55
$\dbinom{n^2}{2}$
$\dbinom{n^2}{2}$
vinayak-kumar
2015-03-27 19:25:55
$\binom{n^2}2$
$\binom{n^2}2$
Eugenis
2015-03-27 19:25:55
$\binom{n^2}{2}$
$\binom{n^2}{2}$
Aang
2015-03-27 19:25:55
N^2C2
N^2C2
math-rules
2015-03-27 19:25:55
Poseidon2001
2015-03-27 19:25:55
n^2 choose 2
n^2 choose 2
lcsmart
2015-03-27 19:25:55
$N^2*(N^2-1)/2$
$N^2*(N^2-1)/2$
owm
2015-03-27 19:25:55
n^2 chose 2
n^2 chose 2
danusv
2015-03-27 19:25:55
n^2 Choose 2
n^2 Choose 2
burunduchok
2015-03-27 19:25:55
n^2 choose 2
n^2 choose 2
Eugenis
2015-03-27 19:25:55
$\binom{n^2}{2}$
$\binom{n^2}{2}$
copeland
2015-03-27 19:25:57
There are $n^2$ squares, so there are $\dbinom{n^2}{2} = \dfrac{n^2(n^2-1)}{2}$ ways to choose 2 of them (without caring about the order).
There are $n^2$ squares, so there are $\dbinom{n^2}{2} = \dfrac{n^2(n^2-1)}{2}$ ways to choose 2 of them (without caring about the order).
copeland
2015-03-27 19:25:59
How many pairs of squares are horizontally or vertically adjacent?
How many pairs of squares are horizontally or vertically adjacent?
copeland
2015-03-27 19:26:03
A simpler question: how many pairs within a single row are horizontally adjacent?
A simpler question: how many pairs within a single row are horizontally adjacent?
cxiong
2015-03-27 19:26:25
n-1
n-1
SockFoot
2015-03-27 19:26:25
$n-1$
$n-1$
slenderestman
2015-03-27 19:26:25
n-1
n-1
nosaj
2015-03-27 19:26:25
n-1
n-1
NextEinstein
2015-03-27 19:26:25
n-1
n-1
maverick8
2015-03-27 19:26:25
n-1
n-1
vinayak-kumar
2015-03-27 19:26:25
$n-1$
$n-1$
copeland
2015-03-27 19:26:27
There are $n-1$ such pairs: the first two, the next two, and so on, up to squares $(n-1)$ and $n$ at the end of the row.
There are $n-1$ such pairs: the first two, the next two, and so on, up to squares $(n-1)$ and $n$ at the end of the row.
copeland
2015-03-27 19:26:29
(Or, you can think about selecting one of the $n-1$ interior vertical edges inside the row, and then taking the squares on either side.)
(Or, you can think about selecting one of the $n-1$ interior vertical edges inside the row, and then taking the squares on either side.)
copeland
2015-03-27 19:26:33
So each row has $n-1$ pairs.
So each row has $n-1$ pairs.
copeland
2015-03-27 19:26:39
And, there are $n$ rows. So there are $n(n-1)$ pairs of horizontally-adjacent squares.
And, there are $n$ rows. So there are $n(n-1)$ pairs of horizontally-adjacent squares.
copeland
2015-03-27 19:26:40
And, the same logic works for columns too, right?
And, the same logic works for columns too, right?
C-bass
2015-03-27 19:26:49
same with each column
same with each column
SockFoot
2015-03-27 19:26:49
you are correct sir
you are correct sir
Abeymom
2015-03-27 19:26:49
yes
yes
copeland
2015-03-27 19:26:51
So there are also $n(n-1)$ pairs of vertically-adjacent squares.
So there are also $n(n-1)$ pairs of vertically-adjacent squares.
copeland
2015-03-27 19:26:53
That makes $2n(n-1)$ total pairs of adjacent square.
That makes $2n(n-1)$ total pairs of adjacent square.
copeland
2015-03-27 19:26:56
Thus, our probability is $\dfrac{2n(n-1)}{\frac{n^2(n^2-1)}{2}} = \dfrac{4n(n-1)}{n^2(n^2-1)}$.
Thus, our probability is $\dfrac{2n(n-1)}{\frac{n^2(n^2-1)}{2}} = \dfrac{4n(n-1)}{n^2(n^2-1)}$.
copeland
2015-03-27 19:27:04
Can this be simplified further?
Can this be simplified further?
24iam24
2015-03-27 19:27:47
4/n(n+1)
4/n(n+1)
Rmehtany
2015-03-27 19:27:47
4/n(n+1)
4/n(n+1)
IsabeltheCat
2015-03-27 19:27:47
yes; 4/n(n+1)
yes; 4/n(n+1)
SockFoot
2015-03-27 19:27:47
$\dfrac{4}{n(n+1)}$
$\dfrac{4}{n(n+1)}$
AlcumusGuy
2015-03-27 19:27:47
$\frac{4}{n(n+1)}$
$\frac{4}{n(n+1)}$
dhruv
2015-03-27 19:27:47
4/n(n+1)
4/n(n+1)
gxah
2015-03-27 19:27:47
$\frac{4}{n^{2}+n}$
$\frac{4}{n^{2}+n}$
copeland
2015-03-27 19:27:51
A factor of $n$ cancels, and $(n-1)$ cancels with $(n^2-1)$ leaving just $(n+1)$.
A factor of $n$ cancels, and $(n-1)$ cancels with $(n^2-1)$ leaving just $(n+1)$.
copeland
2015-03-27 19:27:53
So our probability is $\dfrac{4}{n(n+1)}$.
So our probability is $\dfrac{4}{n(n+1)}$.
copeland
2015-03-27 19:27:57
How do we finish from here?
How do we finish from here?
ompatel99
2015-03-27 19:28:31
It's <1/2015
It's <1/2015
cxiong
2015-03-27 19:28:31
set that to less than 1/2015
set that to less than 1/2015
countingarithmetic
2015-03-27 19:28:31
less than 2015
less than 2015
ImpossibleTriangle
2015-03-27 19:28:31
4/n(n+1) < 1/2015
4/n(n+1) < 1/2015
shiningsunnyday
2015-03-27 19:28:31
<1/2015, then finish off the equation
<1/2015, then finish off the equation
copeland
2015-03-27 19:28:33
We're looking for the least positive $n$ such that $\dfrac{4}{n(n+1)} < \dfrac{1}{2015}$.
We're looking for the least positive $n$ such that $\dfrac{4}{n(n+1)} < \dfrac{1}{2015}$.
C-bass
2015-03-27 19:28:43
n(n+1) > 8060
n(n+1) > 8060
TheEconomist
2015-03-27 19:28:43
n(n+1) > 8060
n(n+1) > 8060
copeland
2015-03-27 19:28:45
Since $n$ is positive, this simplifies to $n(n+1) > 8060$.
Since $n$ is positive, this simplifies to $n(n+1) > 8060$.
copeland
2015-03-27 19:29:05
Since $90^2 = 8100$, that's about where we need to look.
Since $90^2 = 8100$, that's about where we need to look.
swirlykick
2015-03-27 19:29:14
090
090
BPM14
2015-03-27 19:29:14
n = 90
n = 90
henryweng
2015-03-27 19:29:14
n=90
n=90
caressezhu
2015-03-27 19:29:14
90
90
copeland
2015-03-27 19:29:17
Indeed,
\begin{align*}
89(90) = 8010 &< 8060, \\
90(91) = 8190 &> 8060.
\end{align*}
Indeed,
\begin{align*}
89(90) = 8010 &< 8060, \\
90(91) = 8190 &> 8060.
\end{align*}
copeland
2015-03-27 19:29:18
So $n = \boxed{090}$ is our answer.
So $n = \boxed{090}$ is our answer.
copeland
2015-03-27 19:29:26
Alright, a third of the way done!
Alright, a third of the way done!
copeland
2015-03-27 19:29:37
How are you guys doing? Having a good time?
How are you guys doing? Having a good time?
cxiong
2015-03-27 19:29:49
yep
yep
Eugenis
2015-03-27 19:29:49
Yes sir
Yes sir
gxah
2015-03-27 19:29:49
yup
yup
Poseidon2001
2015-03-27 19:29:49
yes
yes
Dracae8
2015-03-27 19:29:49
yes
yes
cxiong
2015-03-27 19:29:49
bluephoenix
2015-03-27 19:29:49
YAH
YAH
dhruv
2015-03-27 19:29:49
yessir
yessir
mikromgrom
2015-03-27 19:29:49
Yes
Yes
Dayranger
2015-03-27 19:29:49
yes!!!
yes!!!
MathStudent2002
2015-03-27 19:29:49
Yeah sure
Yeah sure
firemike
2015-03-27 19:29:49
yse!
yse!
hlasker1
2015-03-27 19:29:49
yeah
yeah
ArgusKelvoy
2015-03-27 19:29:49
Sure.
Sure.
ompatel99
2015-03-27 19:29:49
It's straightforward so far
It's straightforward so far
rt03
2015-03-27 19:29:49
Yes!
Yes!
copeland
2015-03-27 19:29:51
Me too.
Me too.
copeland
2015-03-27 19:30:02
I think the last 2/3 of the exam are going to be a little more work.
I think the last 2/3 of the exam are going to be a little more work.
copeland
2015-03-27 19:30:05
6. Steve says to Jon, "I am thinking of a polynomial whose roots are all positive integers. The polynomial has the form
$$P(x) = 2x^3 - 2ax^2 + (a^2-81)x - c$$
for some positive integers $a$ and $c$. Can you tell me the values of $a$ and $c$?"
6. Steve says to Jon, "I am thinking of a polynomial whose roots are all positive integers. The polynomial has the form
$$P(x) = 2x^3 - 2ax^2 + (a^2-81)x - c$$
for some positive integers $a$ and $c$. Can you tell me the values of $a$ and $c$?"
copeland
2015-03-27 19:30:06
After some calculations, Jon says, "There is more than one such polynomial."
After some calculations, Jon says, "There is more than one such polynomial."
copeland
2015-03-27 19:30:07
Steve says, "You're right. Here is the value of $a$." He writes down a positive integer and asks, "Can you tell me the value of $c$?"
Steve says, "You're right. Here is the value of $a$." He writes down a positive integer and asks, "Can you tell me the value of $c$?"
copeland
2015-03-27 19:30:08
Jon says, "There are still two possible values of $c$."
Jon says, "There are still two possible values of $c$."
copeland
2015-03-27 19:30:09
Find the sum of the two possible values of $c$.
Find the sum of the two possible values of $c$.
copeland
2015-03-27 19:30:16
What a problem. . .
What a problem. . .
copeland
2015-03-27 19:30:30
What's going to be our key tool here?
What's going to be our key tool here?
ompatel99
2015-03-27 19:30:46
Vieta's equations
Vieta's equations
maverick8
2015-03-27 19:30:46
Use Vieta's Formulas
Use Vieta's Formulas
SockFoot
2015-03-27 19:30:46
probably going to use vietas
probably going to use vietas
Eugenis
2015-03-27 19:30:46
Vieta's
Vieta's
cxiong
2015-03-27 19:30:46
vieta's!
vieta's!
SockFoot
2015-03-27 19:30:46
V I E T A S
V I E T A S
bluephoenix
2015-03-27 19:30:46
vietas
vietas
ompatel99
2015-03-27 19:30:46
Vieta
Vieta
mohanxue612
2015-03-27 19:30:46
vieta
vieta
tdeng
2015-03-27 19:30:46
vietas?
vietas?
24iam24
2015-03-27 19:30:46
vieta
vieta
vinayak-kumar
2015-03-27 19:30:46
Vieta!
Vieta!
gxah
2015-03-27 19:30:46
vieta's
vieta's
ImpossibleTriangle
2015-03-27 19:30:46
vieta
vieta
High
2015-03-27 19:30:46
vieta's
vieta's
amburger66
2015-03-27 19:30:46
vieta's?
vieta's?
Borealis
2015-03-27 19:30:46
Vieta's
Vieta's
copeland
2015-03-27 19:30:50
Ding ding.
Ding ding.
copeland
2015-03-27 19:30:51
Vieta's formulas! We'll want to try to relate the roots of the cubic (which we know to be integers!) to the unknown coefficients.
Vieta's formulas! We'll want to try to relate the roots of the cubic (which we know to be integers!) to the unknown coefficients.
copeland
2015-03-27 19:30:55
Let's call the roots $p$, $q$, and $r$. What equations can we write?
Let's call the roots $p$, $q$, and $r$. What equations can we write?
copeland
2015-03-27 19:31:01
Pick one at random to write.
Pick one at random to write.
24iam24
2015-03-27 19:31:29
p+q+r=a
p+q+r=a
Eugenis
2015-03-27 19:31:29
$p+q+r=a$
$p+q+r=a$
vinayak-kumar
2015-03-27 19:31:29
p+q+r=a
p+q+r=a
swirlykick
2015-03-27 19:31:29
p+q+r=a
p+q+r=a
acegikmoqsuwy2000
2015-03-27 19:31:29
p+q+r=a
p+q+r=a
C-bass
2015-03-27 19:31:29
p + q + r = a
p + q + r = a
SockFoot
2015-03-27 19:31:29
$p+q+r=a$
$p+q+r=a$
High
2015-03-27 19:31:29
p+q+r=a
p+q+r=a
acegikmoqsuwy2000
2015-03-27 19:31:41
pq+qr+pr=(a^2-81)/2
pq+qr+pr=(a^2-81)/2
swirlykick
2015-03-27 19:31:41
pr+pq+qr=(a^2 -81)/2
pr+pq+qr=(a^2 -81)/2
24iam24
2015-03-27 19:31:41
pq+qr+pr=a^2-81
pq+qr+pr=a^2-81
maverick8
2015-03-27 19:31:41
pq+qr+pr=(a^2-81)/2
pq+qr+pr=(a^2-81)/2
math-rules
2015-03-27 19:31:59
pqr=c/2
pqr=c/2
dhruv
2015-03-27 19:31:59
pqr=c/2
pqr=c/2
ShadowQueenPeach
2015-03-27 19:31:59
pqr = c/2
pqr = c/2
vinayak-kumar
2015-03-27 19:31:59
pqr=c/2
pqr=c/2
copeland
2015-03-27 19:32:02
Vieta's formulas give us:
\begin{align*}
p+q+r &= a, \\
pq + pr + qr &= \dfrac{a^2-81}{2}, \\
pqr &= \dfrac{c}{2}.
\end{align*}
Vieta's formulas give us:
\begin{align*}
p+q+r &= a, \\
pq + pr + qr &= \dfrac{a^2-81}{2}, \\
pqr &= \dfrac{c}{2}.
\end{align*}
copeland
2015-03-27 19:32:05
(Notice how they carefully arranged the minus signs in the original cubic so that we don't have any negative terms above!)
(Notice how they carefully arranged the minus signs in the original cubic so that we don't have any negative terms above!)
copeland
2015-03-27 19:32:10
Any suggestions for how to work with these?
Any suggestions for how to work with these?
Eugenis
2015-03-27 19:32:55
Square the first equation?
Square the first equation?
High
2015-03-27 19:32:55
square p+q+r
square p+q+r
shiningsunnyday
2015-03-27 19:32:55
Square the first?
Square the first?
NextEinstein
2015-03-27 19:32:55
square first one
square first one
slenderestman
2015-03-27 19:32:55
square the frst equation
square the frst equation
Borealis
2015-03-27 19:32:55
Square the first!
Square the first!
24iam24
2015-03-27 19:32:55
square first
square first
copeland
2015-03-27 19:32:58
That last one doesn't look too helpful yet, but perhaps the first two can be combined in a fruitful way to eliminate $a$ somehow.
That last one doesn't look too helpful yet, but perhaps the first two can be combined in a fruitful way to eliminate $a$ somehow.
copeland
2015-03-27 19:33:00
How about we square the first equation? That will give us lots of terms that appear in the second equation, and both right hand sides will be in terms of $a^2$.
How about we square the first equation? That will give us lots of terms that appear in the second equation, and both right hand sides will be in terms of $a^2$.
copeland
2015-03-27 19:33:04
Specifically,\[a^2=(p+q+r)^2 = p^2 + q^2 + r^2 + 2pq + 2pr + 2qr.\]
Specifically,\[a^2=(p+q+r)^2 = p^2 + q^2 + r^2 + 2pq + 2pr + 2qr.\]
maverick8
2015-03-27 19:33:49
Substitute the first equation into the second
Substitute the first equation into the second
vinayak-kumar
2015-03-27 19:33:49
substitute pq+pr+qr
substitute pq+pr+qr
ShadowQueenPeach
2015-03-27 19:33:49
multiply the next equation by 2
multiply the next equation by 2
dragon_summation
2015-03-27 19:33:49
Multiply second equation by 2
Multiply second equation by 2
countingarithmetic
2015-03-27 19:33:49
$p^2 + q^2 + r^2 = 81$
$p^2 + q^2 + r^2 = 81$
ompatel99
2015-03-27 19:33:56
p^2+q^2+r^2=81
p^2+q^2+r^2=81
Rmehtany
2015-03-27 19:33:56
p^2+q^2+r^2 = 81
p^2+q^2+r^2 = 81
copeland
2015-03-27 19:33:58
Ah -- we see $2(pq+pr+qr)$ in our second equation. We can substitute $a^2 - 81$ in for it.
Ah -- we see $2(pq+pr+qr)$ in our second equation. We can substitute $a^2 - 81$ in for it.
copeland
2015-03-27 19:33:59
Now we have
\[
p^2 + q^2 + r^2 + a^2 - 81 = a^2.
\]
Now we have
\[
p^2 + q^2 + r^2 + a^2 - 81 = a^2.
\]
copeland
2015-03-27 19:34:01
That's great! The $a^2$'s cancel and we're left with just
\[
p^2 + q^2 + r^2 = 81.
\]
That's great! The $a^2$'s cancel and we're left with just
\[
p^2 + q^2 + r^2 = 81.
\]
copeland
2015-03-27 19:34:08
Now what?
Now what?
room456
2015-03-27 19:34:30
bash out triples of p q and r
bash out triples of p q and r
High
2015-03-27 19:34:30
find integer values that work
find integer values that work
maverick8
2015-03-27 19:34:30
Guess and check
Guess and check
cxiong
2015-03-27 19:34:30
just try to find squares that add up to 81
just try to find squares that add up to 81
ompatel99
2015-03-27 19:34:30
not that many cases
not that many cases
dantx5
2015-03-27 19:34:30
bash
bash
Tommy2000
2015-03-27 19:34:30
Find the integer solutions
Find the integer solutions
Eugenis
2015-03-27 19:34:30
List out the triples that when squared are $81$
List out the triples that when squared are $81$
tdeng
2015-03-27 19:34:30
find 3 squares whose sum is 81
find 3 squares whose sum is 81
copeland
2015-03-27 19:34:32
Oh yeah.
Oh yeah.
copeland
2015-03-27 19:34:33
This equation probably does not have a lot of positive integer solutions. Perhaps we can just list them all.
This equation probably does not have a lot of positive integer solutions. Perhaps we can just list them all.
copeland
2015-03-27 19:34:36
How can we organize listing all the solutions in a sensible way?
How can we organize listing all the solutions in a sensible way?
NextEinstein
2015-03-27 19:35:11
largest square
largest square
vinayak-kumar
2015-03-27 19:35:11
WLOG assume $p\ge q\ge r$
WLOG assume $p\ge q\ge r$
mikromgrom
2015-03-27 19:35:11
casework on 1 variable
casework on 1 variable
andrewlin
2015-03-27 19:35:11
Assume p<=q<=r
Assume p<=q<=r
nosaj
2015-03-27 19:35:11
by the largest value
by the largest value
copeland
2015-03-27 19:35:14
Let's assume that $p \ge q \ge r$. (It doesn't matter what order we write the roots in, since all our equations are symmetric.)
Let's assume that $p \ge q \ge r$. (It doesn't matter what order we write the roots in, since all our equations are symmetric.)
copeland
2015-03-27 19:35:24
What do we know about $p$?
What do we know about $p$?
slenderestman
2015-03-27 19:36:03
less than 9
less than 9
tdeng
2015-03-27 19:36:03
<9
<9
C-bass
2015-03-27 19:36:03
p>5
p>5
vinayak-kumar
2015-03-27 19:36:03
p can range from 6 to 8
p can range from 6 to 8
cxiong
2015-03-27 19:36:03
0 < p < 9
0 < p < 9
slenderestman
2015-03-27 19:36:03
less than 9 and greater than 5
less than 9 and greater than 5
ShadowQueenPeach
2015-03-27 19:36:03
it is less than 9
it is less than 9
xayy
2015-03-27 19:36:03
less than 9
less than 9
High
2015-03-27 19:36:03
p < 9
p < 9
dantx5
2015-03-27 19:36:03
p>=6
p>=6
rt03
2015-03-27 19:36:03
it is < than 9
it is < than 9
copeland
2015-03-27 19:36:06
$p=9$ is too big, and $p=5$ is too small (since $3 \cdot 5^2$ isn't big enough). So $p$ must be 6, 7, or 8.
$p=9$ is too big, and $p=5$ is too small (since $3 \cdot 5^2$ isn't big enough). So $p$ must be 6, 7, or 8.
copeland
2015-03-27 19:36:07
If $p=8$, are there any solutions?
If $p=8$, are there any solutions?
SockFoot
2015-03-27 19:36:26
$(4,1)$
$(4,1)$
Dayranger
2015-03-27 19:36:26
8, 4, 1
8, 4, 1
maverick8
2015-03-27 19:36:26
(1,4,8)
(1,4,8)
koteswari
2015-03-27 19:36:26
Yes, 64,16,1
Yes, 64,16,1
mathwizard888
2015-03-27 19:36:26
4,1
4,1
slenderestman
2015-03-27 19:36:26
8, 4, 1
8, 4, 1
Abeymom
2015-03-27 19:36:26
q=4 r=1
q=4 r=1
Darn
2015-03-27 19:36:26
$841$
$841$
copeland
2015-03-27 19:36:28
We must have $q^2 + r^2 = 81 - 64 = 17$. The only solution is $(p,q,r) = (8,4,1)$.
We must have $q^2 + r^2 = 81 - 64 = 17$. The only solution is $(p,q,r) = (8,4,1)$.
copeland
2015-03-27 19:36:29
How about $p=7$?
How about $p=7$?
henryweng
2015-03-27 19:37:05
(4,4,7)
(4,4,7)
ImpossibleTriangle
2015-03-27 19:37:05
7,4,4
7,4,4
Rocksolid
2015-03-27 19:37:05
7,4,4
7,4,4
azmath333
2015-03-27 19:37:05
(7,4,4)
(7,4,4)
bluephoenix
2015-03-27 19:37:05
wait, 7, 4, 4
wait, 7, 4, 4
amburger66
2015-03-27 19:37:05
7,4,4
7,4,4
copeland
2015-03-27 19:37:07
We must have $q^2 + r^2 = 81 - 49 = 32$. The only solution is $(p,q,r) = (7,4,4)$.
We must have $q^2 + r^2 = 81 - 49 = 32$. The only solution is $(p,q,r) = (7,4,4)$.
copeland
2015-03-27 19:37:08
How about $p=6$?
How about $p=6$?
swirlykick
2015-03-27 19:37:26
6,6,3
6,6,3
ShadowQueenPeach
2015-03-27 19:37:26
6,6,3
6,6,3
24iam24
2015-03-27 19:37:26
6, 6, 3
6, 6, 3
sjag
2015-03-27 19:37:26
(6,6,3)
(6,6,3)
maverick8
2015-03-27 19:37:26
6,6,3
6,6,3
ThePiPie
2015-03-27 19:37:26
6, 6, 3
6, 6, 3
copeland
2015-03-27 19:37:28
We must have $q^2 + r^2 = 81 - 36 = 45$. The only solution is $(p,q,r) = (6,6,3)$.
We must have $q^2 + r^2 = 81 - 36 = 45$. The only solution is $(p,q,r) = (6,6,3)$.
copeland
2015-03-27 19:37:31
So these three are our only solutions. Here's a chart:
\[\begin{array}{c|c|c}
p&q&r \\ \hline
8 & 4 & 1 \\ \hline
7 & 4 & 4 \\ \hline
6 & 6 & 3
\end{array}\]
So these three are our only solutions. Here's a chart:
\[\begin{array}{c|c|c}
p&q&r \\ \hline
8 & 4 & 1 \\ \hline
7 & 4 & 4 \\ \hline
6 & 6 & 3
\end{array}\]
copeland
2015-03-27 19:37:32
How do we finish?
How do we finish?
dhruv
2015-03-27 19:38:24
we find the values of c that correspond to the roots
we find the values of c that correspond to the roots
shiningsunnyday
2015-03-27 19:38:24
Find values of C
Find values of C
24iam24
2015-03-27 19:38:24
so thats a and we can find c
so thats a and we can find c
andrewlin
2015-03-27 19:38:24
calculate c
calculate c
Darn
2015-03-27 19:38:24
$pqr=\frac{c}{2}$
$pqr=\frac{c}{2}$
Dayranger
2015-03-27 19:38:24
find all possible values for c
find all possible values for c
copeland
2015-03-27 19:38:29
We can compute $a$ and $c$ for these. Recall that $a$ is their sum, and $c$ is twice their product:
We can compute $a$ and $c$ for these. Recall that $a$ is their sum, and $c$ is twice their product:
copeland
2015-03-27 19:38:30
\[\begin{array}{c|c|c|c|c}
p&q&r&a&c \\ \hline
8 & 4 & 1 & 13 & 64 \\ \hline
7 & 4 & 4 & 15 & 224 \\ \hline
6 & 6 & 3 & 15 & 216
\end{array}\]
\[\begin{array}{c|c|c|c|c}
p&q&r&a&c \\ \hline
8 & 4 & 1 & 13 & 64 \\ \hline
7 & 4 & 4 & 15 & 224 \\ \hline
6 & 6 & 3 & 15 & 216
\end{array}\]
copeland
2015-03-27 19:38:31
Knowing the value of $a$ didn't help Jon, so what can we conclude?
Knowing the value of $a$ didn't help Jon, so what can we conclude?
MathStudent2002
2015-03-27 19:38:53
Then $a=15$
Then $a=15$
C-bass
2015-03-27 19:38:53
a is 15
a is 15
ompatel99
2015-03-27 19:38:53
a must be 15
a must be 15
xayy
2015-03-27 19:38:53
a=15
a=15
Abeymom
2015-03-27 19:38:53
a=15
a=15
shiningsunnyday
2015-03-27 19:38:53
It must be 15
It must be 15
gxah
2015-03-27 19:39:05
224+216=440
224+216=440
AlisonH
2015-03-27 19:39:05
c=216 and 224
c=216 and 224
mathmaster2012
2015-03-27 19:39:05
a = 15, answer is 224+216=440
a = 15, answer is 224+216=440
firemike
2015-03-27 19:39:05
224;216
224;216
caressezhu
2015-03-27 19:39:05
440
440
firemike
2015-03-27 19:39:05
224 + 216
224 + 216
danusv
2015-03-27 19:39:05
224+216 = 440
224+216 = 440
copeland
2015-03-27 19:39:08
$a$ must be 15 -- there are two possible solutions.
$a$ must be 15 -- there are two possible solutions.
copeland
2015-03-27 19:39:10
We're asked for the sum of the two $c$'s for these solutions, so our answer is $224 + 216 = \boxed{440}$.
We're asked for the sum of the two $c$'s for these solutions, so our answer is $224 + 216 = \boxed{440}$.
Darn
2015-03-27 19:39:31
2/5 of the way there!
2/5 of the way there!
copeland
2015-03-27 19:39:33
Gogo fractions!
Gogo fractions!
copeland
2015-03-27 19:39:36
7. Triangle $ABC$ has side lengths $AB=12$, $BC=25$, and $CA=17$. Rectangle $PQRS$ has vertex $P$ on $\overline{AB}$, vertex $Q$ on $\overline{AC}$, and vertices $R$ and $S$ on $\overline{BC}$. In terms of the side length $PQ=w$, the area of $PQRS$ can be expressed as the quadratic polynomial \[\operatorname{Area}(PQRS)=\alpha w-\beta w^2.\]Then the coefficient $\beta=\dfrac mn$, where $m$ and $n$ are relatively prime positive integers. Find $m+n$.
7. Triangle $ABC$ has side lengths $AB=12$, $BC=25$, and $CA=17$. Rectangle $PQRS$ has vertex $P$ on $\overline{AB}$, vertex $Q$ on $\overline{AC}$, and vertices $R$ and $S$ on $\overline{BC}$. In terms of the side length $PQ=w$, the area of $PQRS$ can be expressed as the quadratic polynomial \[\operatorname{Area}(PQRS)=\alpha w-\beta w^2.\]Then the coefficient $\beta=\dfrac mn$, where $m$ and $n$ are relatively prime positive integers. Find $m+n$.
copeland
2015-03-27 19:39:46
When I first saw this problem it took me a second to decide whether I thought the area formula was a restriction on the problem or a consequence of the rest of the statement. That is, I took a moment to decide whether that fact was needed to solve the problem or if it was just required to AIME-ify the answer.
When I first saw this problem it took me a second to decide whether I thought the area formula was a restriction on the problem or a consequence of the rest of the statement. That is, I took a moment to decide whether that fact was needed to solve the problem or if it was just required to AIME-ify the answer.
copeland
2015-03-27 19:39:59
It is a consequence of the rest of the setup, but I will point out along the way the place where you can use that statement to shortcut a bit of the solution. That is, if you assume that the area has to have that form, you can shorten some of the computations.
It is a consequence of the rest of the setup, but I will point out along the way the place where you can use that statement to shortcut a bit of the solution. That is, if you assume that the area has to have that form, you can shorten some of the computations.
copeland
2015-03-27 19:40:07
What do we do first?
What do we do first?
maverick8
2015-03-27 19:40:20
DiAgRaM
DiAgRaM
Dracae8
2015-03-27 19:40:20
picture!
picture!
swirlykick
2015-03-27 19:40:20
Draw a diagram
Draw a diagram
tdeng
2015-03-27 19:40:20
diagram
diagram
Dayranger
2015-03-27 19:40:20
diagram
diagram
hlasker1
2015-03-27 19:40:20
diagram!
diagram!
SockFoot
2015-03-27 19:40:20
please give diagram
please give diagram
bluephoenix
2015-03-27 19:40:20
draw a diagram!
draw a diagram!
slenderestman
2015-03-27 19:40:20
draw diagram
draw diagram
dhruv
2015-03-27 19:40:20
draw a diagram
draw a diagram
maverick8
2015-03-27 19:40:20
diagram
diagram
ompatel99
2015-03-27 19:40:20
Picture
Picture
_--__--_
2015-03-27 19:40:20
Draw a diagram?
Draw a diagram?
copeland
2015-03-27 19:40:23
Draw a diagram!
Draw a diagram!
copeland
2015-03-27 19:40:24
copeland
2015-03-27 19:40:25
copeland
2015-03-27 19:40:27
copeland
2015-03-27 19:40:30
Now what do we do?
Now what do we do?
copeland
2015-03-27 19:40:54
The area is $wh$ so we need to find $h$.
The area is $wh$ so we need to find $h$.
copeland
2015-03-27 19:40:55
How can we compute $h$?
How can we compute $h$?
ShadowQueenPeach
2015-03-27 19:41:37
use similar triangles
use similar triangles
Tommy2000
2015-03-27 19:41:37
similar triangles
similar triangles
mssmath
2015-03-27 19:41:37
Similiarity
Similiarity
slenderestman
2015-03-27 19:41:37
similar triangles?
similar triangles?
copeland
2015-03-27 19:41:40
There are similar triangles here.
There are similar triangles here.
copeland
2015-03-27 19:41:42
copeland
2015-03-27 19:41:43
$\triangle ABC\sim\triangle PBQ$. If the height of $\triangle ABC$ is $H$, then what is $h$?
$\triangle ABC\sim\triangle PBQ$. If the height of $\triangle ABC$ is $H$, then what is $h$?
mathwrath
2015-03-27 19:42:44
H * (25-w)/25
H * (25-w)/25
dhruv
2015-03-27 19:42:44
(25-w)/25 * H
(25-w)/25 * H
High
2015-03-27 19:42:44
H - Hw/25
H - Hw/25
SockFoot
2015-03-27 19:42:44
$H-H\cdot\dfrac{w}{25}$
$H-H\cdot\dfrac{w}{25}$
swirlykick
2015-03-27 19:42:44
(H-h)/H = W/25
(H-h)/H = W/25
vinayak-kumar
2015-03-27 19:42:44
$H-wH/25$
$H-wH/25$
copeland
2015-03-27 19:42:50
\[\dfrac {H-h}H=\dfrac w{25}\]or\[H-h=\dfrac{Hw}{25}\] Therefore $h=H-\dfrac{H}{25}\cdot w$.
\[\dfrac {H-h}H=\dfrac w{25}\]or\[H-h=\dfrac{Hw}{25}\] Therefore $h=H-\dfrac{H}{25}\cdot w$.
copeland
2015-03-27 19:42:55
The area is \[w\cdot h=wH-\frac H{25}w^2.\] The value we are looking for is $\dfrac H{25}$.
The area is \[w\cdot h=wH-\frac H{25}w^2.\] The value we are looking for is $\dfrac H{25}$.
copeland
2015-03-27 19:43:02
How can we find the height of $\triangle ABC$?
How can we find the height of $\triangle ABC$?
mohanxue612
2015-03-27 19:43:20
finding the area of abc
finding the area of abc
vinayak-kumar
2015-03-27 19:43:20
Find the area of the triangle
Find the area of the triangle
simon1221
2015-03-27 19:43:20
area two ways
area two ways
copeland
2015-03-27 19:43:26
Good. Tell me more.
Good. Tell me more.
awesome123
2015-03-27 19:43:43
use herons formula
use herons formula
24iam24
2015-03-27 19:43:43
we can find H with heron's
we can find H with heron's
maverick8
2015-03-27 19:43:43
Heron
Heron
awesome123
2015-03-27 19:43:43
herons formula
herons formula
swirlykick
2015-03-27 19:43:43
Herons formula
Herons formula
Borealis
2015-03-27 19:43:43
Let's use Heron's formula and then find the altitude as area = 1/2 * b * h
Let's use Heron's formula and then find the altitude as area = 1/2 * b * h
Dracae8
2015-03-27 19:43:43
herons --> bh/2
herons --> bh/2
dhruv
2015-03-27 19:43:43
heron's formula!
heron's formula!
cxiong
2015-03-27 19:43:43
heron's formula
heron's formula
1023ong
2015-03-27 19:43:43
herons was beautiful
herons was beautiful
copeland
2015-03-27 19:43:47
There are two reasonable approaches and one silly one.
There are two reasonable approaches and one silly one.
copeland
2015-03-27 19:43:49
The first reasonable approach is to drop an altitude and set up a large number of equations. That's what I did originally because I'm lazy.
The first reasonable approach is to drop an altitude and set up a large number of equations. That's what I did originally because I'm lazy.
copeland
2015-03-27 19:43:52
The second reasonable approach is to hope that this is a nice triangle and throw Heron's formula at it to try to find the area.
The second reasonable approach is to hope that this is a nice triangle and throw Heron's formula at it to try to find the area.
copeland
2015-03-27 19:43:57
The silly one is Stewart's Theorem. If you know a lot about Stewart's Theorem then you know it reduces to Heron where the cevian is an altitude. If you don't know that much about Stewart's Theorem then you are a lot like me.
The silly one is Stewart's Theorem. If you know a lot about Stewart's Theorem then you know it reduces to Heron where the cevian is an altitude. If you don't know that much about Stewart's Theorem then you are a lot like me.
copeland
2015-03-27 19:44:07
I think the most enlightening approach is Heron, especially given how many other problems today that we have solved/will solve by setting up massive systems of quadratic equations.
I think the most enlightening approach is Heron, especially given how many other problems today that we have solved/will solve by setting up massive systems of quadratic equations.
copeland
2015-03-27 19:44:11
What is the area of this triangle by Heron?
What is the area of this triangle by Heron?
cxiong
2015-03-27 19:44:53
90
90
vinayak-kumar
2015-03-27 19:44:53
90
90
caressezhu
2015-03-27 19:44:53
90
90
maverick8
2015-03-27 19:44:53
90
90
andrewlin
2015-03-27 19:44:53
90
90
swirlykick
2015-03-27 19:44:53
90
90
awesome123
2015-03-27 19:44:53
90
90
nosaj
2015-03-27 19:44:53
90
90
Rmehtany
2015-03-27 19:44:53
90
90
copeland
2015-03-27 19:44:56
The perimeter is $12+17+25=54$ so the semiperimeter is $s=27$.
The perimeter is $12+17+25=54$ so the semiperimeter is $s=27$.
copeland
2015-03-27 19:45:01
The area solves
\begin{align*}
A^2
&=s(s-12)(s-17)(s-25)\\
&=27\cdot15\cdot10\cdot2\\
&=(3^3)(3\cdot5)(2\cdot5)\cdot2\\
&=(3^2\cdot5\cdot2)^2\\
&=90^2.
\end{align*}
The area is 90.
The area solves
\begin{align*}
A^2
&=s(s-12)(s-17)(s-25)\\
&=27\cdot15\cdot10\cdot2\\
&=(3^3)(3\cdot5)(2\cdot5)\cdot2\\
&=(3^2\cdot5\cdot2)^2\\
&=90^2.
\end{align*}
The area is 90.
copeland
2015-03-27 19:45:11
What is the height?
What is the height?
maverick8
2015-03-27 19:45:58
36/5
36/5
24iam24
2015-03-27 19:45:58
so H=180/25=36/5
so H=180/25=36/5
gxah
2015-03-27 19:45:58
36/5
36/5
simon1221
2015-03-27 19:45:58
180/25 or 36/5
180/25 or 36/5
dhruv
2015-03-27 19:45:58
36/5
36/5
mathwizard888
2015-03-27 19:45:58
36/5
36/5
countingarithmetic
2015-03-27 19:45:58
36/5
36/5
hlasker1
2015-03-27 19:45:58
36/5
36/5
swe1
2015-03-27 19:45:58
36/5
36/5
copeland
2015-03-27 19:46:03
The height solves $90=\frac{25H}2$ so the height is $H=\dfrac{180}{25}=\dfrac{36}5$
The height solves $90=\frac{25H}2$ so the height is $H=\dfrac{180}{25}=\dfrac{36}5$
copeland
2015-03-27 19:46:07
What's the answer to the problem?
What's the answer to the problem?
Darn
2015-03-27 19:46:45
36+125=161
36+125=161
shiningsunnyday
2015-03-27 19:46:45
36+125=161
36+125=161
dhruv
2015-03-27 19:46:45
H/25=36/125 =>161
H/25=36/125 =>161
High
2015-03-27 19:46:45
36/5*1/25=36/125 --> 161
36/5*1/25=36/125 --> 161
azmath333
2015-03-27 19:46:45
161
161
ryanyz10
2015-03-27 19:46:45
161
161
owm
2015-03-27 19:46:45
36+125
36+125
mathwrath
2015-03-27 19:46:45
36/125=>161
36/125=>161
24iam24
2015-03-27 19:46:45
36/125 so 161
36/125 so 161
copeland
2015-03-27 19:46:49
We want to compute $\dfrac{H}{25}=\dfrac{36}{125}$. The answer is $36+125=\boxed{161}$.
We want to compute $\dfrac{H}{25}=\dfrac{36}{125}$. The answer is $36+125=\boxed{161}$.
copeland
2015-03-27 19:46:56
Incidentally, notice if we assume the formula from the problem right at the beginning then we know that $h=\dfrac{\operatorname{Area}(PQRS)}{w}=\alpha-\beta w$. How could we finish from there?
Incidentally, notice if we assume the formula from the problem right at the beginning then we know that $h=\dfrac{\operatorname{Area}(PQRS)}{w}=\alpha-\beta w$. How could we finish from there?
vinayak-kumar
2015-03-27 19:48:13
let w=25
let w=25
mssmath
2015-03-27 19:48:13
We know a from w=0, h=max cases
We know a from w=0, h=max cases
copeland
2015-03-27 19:48:16
If we know the value of $h$ at two different widths we can get two equations in $\alpha$ and $\beta$ that we can solve for $\beta$.
If we know the value of $h$ at two different widths we can get two equations in $\alpha$ and $\beta$ that we can solve for $\beta$.
copeland
2015-03-27 19:48:18
If $w=25$ then $h=0$. That gives\[0=\alpha+25\beta.\]
If $w=25$ then $h=0$. That gives\[0=\alpha+25\beta.\]
copeland
2015-03-27 19:48:18
What other point would you choose?
What other point would you choose?
cxiong
2015-03-27 19:48:57
w = 12.5
w = 12.5
vinayak-kumar
2015-03-27 19:48:57
w=25/2? seems the easiest
w=25/2? seems the easiest
24iam24
2015-03-27 19:48:57
w=0
w=0
1023ong
2015-03-27 19:48:57
midsegment?
midsegment?
swe1
2015-03-27 19:48:57
$w = \frac{25}{2}$
$w = \frac{25}{2}$
ompatel99
2015-03-27 19:48:57
w=0
w=0
acegikmoqsuwy2000
2015-03-27 19:48:57
at the top of the triangle when w=0
at the top of the triangle when w=0
copeland
2015-03-27 19:49:10
You can choose $w=\dfrac{25}2$. That makes the $h$ half the height of the triangle so we're reduced to finding the height of the triangle.
You can choose $w=\dfrac{25}2$. That makes the $h$ half the height of the triangle so we're reduced to finding the height of the triangle.
copeland
2015-03-27 19:49:11
You could also choose $w=0$. Then the rectangle has area 0 and the height is equal to the height of the triangle. Again you need the height of the triangle but the computation is easier.
You could also choose $w=0$. Then the rectangle has area 0 and the height is equal to the height of the triangle. Again you need the height of the triangle but the computation is easier.
copeland
2015-03-27 19:49:15
However note that we divided by $w$ to find $h=\dfrac{\operatorname{Area}(PQRS)}{w}$ so you really need to mumble something about continuity before this is kosher.
However note that we divided by $w$ to find $h=\dfrac{\operatorname{Area}(PQRS)}{w}$ so you really need to mumble something about continuity before this is kosher.
copeland
2015-03-27 19:49:24
Also, this is an awesome triangle:
Also, this is an awesome triangle:
copeland
2015-03-27 19:49:25
copeland
2015-03-27 19:49:38
Also, yes, I swapped two of the vertices in my diagram. My bad.
Also, yes, I swapped two of the vertices in my diagram. My bad.
copeland
2015-03-27 19:49:57
copeland
2015-03-27 19:50:11
The guy on the left is a 3-4-5 triangle and the one on the right is everybody's favorite 36-77-85 Pythagorean triple.
The guy on the left is a 3-4-5 triangle and the one on the right is everybody's favorite 36-77-85 Pythagorean triple.
Tommy2000
2015-03-27 19:50:26
Yes, I knew that one!
Yes, I knew that one!
copeland
2015-03-27 19:50:39
Raise your hand if you knew that one!
Raise your hand if you knew that one!
copeland
2015-03-27 19:50:49
Liars. All.
Liars. All.
copeland
2015-03-27 19:50:56
8. Let $a$ and $b$ be positive integers satisfying $\dfrac{ab+1}{a+b}<\dfrac32$. The maximum possible value of $\dfrac{a^3b^3+1}{a^3+b^3}$ is $\dfrac pq$, where $p$ and $q$ are realtively prime positive integers. Find $p+q$.
8. Let $a$ and $b$ be positive integers satisfying $\dfrac{ab+1}{a+b}<\dfrac32$. The maximum possible value of $\dfrac{a^3b^3+1}{a^3+b^3}$ is $\dfrac pq$, where $p$ and $q$ are realtively prime positive integers. Find $p+q$.
copeland
2015-03-27 19:51:10
Notice anything about the expression $\dfrac{ab+1}{a+b}$? How strict is it?
Notice anything about the expression $\dfrac{ab+1}{a+b}$? How strict is it?
1915933
2015-03-27 19:51:41
"Relatively" is misspelled.
"Relatively" is misspelled.
copeland
2015-03-27 19:51:56
realtively. That's what I meant.
realtively. That's what I meant.
Poseidon2001
2015-03-27 19:52:20
what do you mean by strict?
what do you mean by strict?
copeland
2015-03-27 19:52:23
Good question.
Good question.
copeland
2015-03-27 19:53:03
What about the inequality $\dfrac{x^6+3x^2}{x^2}<1$? Does that have a "lot" of solutions?
What about the inequality $\dfrac{x^6+3x^2}{x^2}<1$? Does that have a "lot" of solutions?
ryanyz10
2015-03-27 19:53:44
no.. very quickyl is greater than 1
no.. very quickyl is greater than 1
Rmehtany
2015-03-27 19:53:44
no
no
henryweng
2015-03-27 19:53:44
no solution
no solution
dhruv
2015-03-27 19:53:44
no
no
copeland
2015-03-27 19:53:48
Bad example.
Bad example.
copeland
2015-03-27 19:54:01
$\dfrac{x^6+3x^2}{x^2+1}<1$?
$\dfrac{x^6+3x^2}{x^2+1}<1$?
tdeng
2015-03-27 19:54:36
how do you define a "lot"
how do you define a "lot"
copeland
2015-03-27 19:54:42
The area of the region, say.
The area of the region, say.
vinayak-kumar
2015-03-27 19:54:57
it doesnt have a lot either
it doesnt have a lot either
copeland
2015-03-27 19:55:00
Why not?
Why not?
mssmath
2015-03-27 19:55:17
x going to infinity is bad
x going to infinity is bad
copeland
2015-03-27 19:55:19
It's degree 6 over degree 2. Eventually the numerator wins out and we fail.
It's degree 6 over degree 2. Eventually the numerator wins out and we fail.
copeland
2015-03-27 19:55:27
Back to our problem:
Back to our problem:
copeland
2015-03-27 19:55:35
$\dfrac{ab+1}{a+b}$ How strict is it?
$\dfrac{ab+1}{a+b}$ How strict is it?
SockFoot
2015-03-27 19:55:54
well $ab$ is gonna be a lot bigger than $a+b$
well $ab$ is gonna be a lot bigger than $a+b$
SockFoot
2015-03-27 19:55:54
most of the time
most of the time
ompatel99
2015-03-27 19:55:57
Very strict. Product triumphs over sum later
Very strict. Product triumphs over sum later
copeland
2015-03-27 19:56:00
Right.
Right.
copeland
2015-03-27 19:56:10
This problem is trying to be tricky, but we're on to it.
This problem is trying to be tricky, but we're on to it.
copeland
2015-03-27 19:56:26
It's quadratic over linear so it should usually be big for positive integers. There aren't going to be very many solutions to this equation. That's nice.
It's quadratic over linear so it should usually be big for positive integers. There aren't going to be very many solutions to this equation. That's nice.
copeland
2015-03-27 19:56:31
(I also notice that this looks like the hyperbolic cotangent sum formula, but that helps less than zero.)
(I also notice that this looks like the hyperbolic cotangent sum formula, but that helps less than zero.)
copeland
2015-03-27 19:56:35
What should we do?
What should we do?
nosaj
2015-03-27 19:57:06
should we cross multiply?
should we cross multiply?
cpma213
2015-03-27 19:57:06
Multiply a+b over?
Multiply a+b over?
vinayak-kumar
2015-03-27 19:57:06
cross multiply?
cross multiply?
C-bass
2015-03-27 19:57:06
multiply both sides by a + b and 2
multiply both sides by a + b and 2
copeland
2015-03-27 19:57:08
Let's multiply by $2(a+b)$ and see where that gets us.
Let's multiply by $2(a+b)$ and see where that gets us.
copeland
2015-03-27 19:57:10
Since $a$ and $b$ are positive, $\dfrac{ab+1}{a+b}<\dfrac32$ if and only if \[2ab+2<3a+3b.\]Bringing everything to one side gives \[2ab-3a-3b+2<0.\]
Since $a$ and $b$ are positive, $\dfrac{ab+1}{a+b}<\dfrac32$ if and only if \[2ab+2<3a+3b.\]Bringing everything to one side gives \[2ab-3a-3b+2<0.\]
copeland
2015-03-27 19:57:10
Now what?
Now what?
cpma213
2015-03-27 19:57:28
SFFT?
SFFT?
acegikmoqsuwy2000
2015-03-27 19:57:28
2ab+2<3a+3b then SFFT/
2ab+2<3a+3b then SFFT/
Tommy2000
2015-03-27 19:57:28
SFFT
SFFT
bluephoenix
2015-03-27 19:57:28
SFFT
SFFT
ryanyz10
2015-03-27 19:57:28
SFFT
SFFT
gxah
2015-03-27 19:57:28
SFFT
SFFT
acegikmoqsuwy2000
2015-03-27 19:57:28
simons favorite factoring trick?
simons favorite factoring trick?
_--__--_
2015-03-27 19:57:28
SFFT?
SFFT?
copeland
2015-03-27 19:57:31
Simon's Favorite Factoring Trick. Of course we need that somewhere on the AIME.
Simon's Favorite Factoring Trick. Of course we need that somewhere on the AIME.
copeland
2015-03-27 19:57:32
You can read about SFFT on our Wiki later here:
http://artofproblemsolving.com/wiki/index.php/Simon%27s_Favorite_Factoring_Trick
You can read about SFFT on our Wiki later here:
http://artofproblemsolving.com/wiki/index.php/Simon%27s_Favorite_Factoring_Trick
copeland
2015-03-27 19:57:36
SFFT is hard for this expression. What do we do to make it easier?
SFFT is hard for this expression. What do we do to make it easier?
NextEinstein
2015-03-27 19:58:01
multiply by 2 and SFFT
multiply by 2 and SFFT
cpma213
2015-03-27 19:58:01
Multiply by 2 again for SFFT
Multiply by 2 again for SFFT
vinayak-kumar
2015-03-27 19:58:01
multiply by 2 hehe
multiply by 2 hehe
nosaj
2015-03-27 19:58:01
mulitply by 2
mulitply by 2
Aviously
2015-03-27 19:58:01
Multiply ny 2
Multiply ny 2
copeland
2015-03-27 19:58:04
The middle terms are symmetric so we would benefit from the leading term being a square. What product of binomials do we want to pull out of \[4ab-6a-6b+4<0?\]
The middle terms are symmetric so we would benefit from the leading term being a square. What product of binomials do we want to pull out of \[4ab-6a-6b+4<0?\]
BPM14
2015-03-27 19:58:57
(2a-3)(2b-3)<5
(2a-3)(2b-3)<5
Darn
2015-03-27 19:58:57
$(2a-3)(2b-3) < 5$
$(2a-3)(2b-3) < 5$
_--__--_
2015-03-27 19:58:57
(2a-3)(2b-3)<5
(2a-3)(2b-3)<5
ryanyz10
2015-03-27 19:58:57
(2a - 3) (2b - 3)
(2a - 3) (2b - 3)
Aviously
2015-03-27 19:58:57
(2a+3)(2b+3)
(2a+3)(2b+3)
nosaj
2015-03-27 19:58:57
Now it is (2a-3)(2b-3)-5<0
Now it is (2a-3)(2b-3)-5<0
vinayak-kumar
2015-03-27 19:58:57
(2a-3)(2b-3)
(2a-3)(2b-3)
AlisonH
2015-03-27 19:58:57
(2a-3)(2b-3)
(2a-3)(2b-3)
copeland
2015-03-27 19:59:01
We can rewrite this quadratic as \[(2a-3)(2b-3)<5.\] $a$ and $b$ are both positive. By symmetry we can assume that $a\leq b$.
We can rewrite this quadratic as \[(2a-3)(2b-3)<5.\] $a$ and $b$ are both positive. By symmetry we can assume that $a\leq b$.
copeland
2015-03-27 19:59:02
If $a=1$ what are the solutions?
If $a=1$ what are the solutions?
cpma213
2015-03-27 19:59:44
b is anything
b is anything
vinayak-kumar
2015-03-27 19:59:44
any b
any b
SockFoot
2015-03-27 19:59:44
$b=$anything
$b=$anything
henryweng
2015-03-27 19:59:44
b can be anything
b can be anything
tdeng
2015-03-27 19:59:44
b can be anything
b can be anything
Rmehtany
2015-03-27 19:59:44
b>-1
b>-1
copeland
2015-03-27 19:59:47
If $a=1$ then we want to solve $-(2b-3)<5$ or $2b>-8$. That's every $b$.
If $a=1$ then we want to solve $-(2b-3)<5$ or $2b>-8$. That's every $b$.
copeland
2015-03-27 19:59:53
If $a=2$ what are the solutions?
If $a=2$ what are the solutions?
cpma213
2015-03-27 20:00:29
b=1,2,3
b=1,2,3
Dayranger
2015-03-27 20:00:29
b < 4
b < 4
amburger66
2015-03-27 20:00:29
b<4
b<4
Poseidon2001
2015-03-27 20:00:29
b<4
b<4
chessderek
2015-03-27 20:00:29
anything less than 4
anything less than 4
vinayak-kumar
2015-03-27 20:00:29
2 and 3
2 and 3
Darn
2015-03-27 20:00:29
wait i mean b=2,3
wait i mean b=2,3
mathperson9
2015-03-27 20:00:29
b less than 4
b less than 4
countingarithmetic
2015-03-27 20:00:29
b = 2, 3
b = 2, 3
copeland
2015-03-27 20:00:31
If $a=2$ then we want to solve $(2b-3)<5$ or $b<4$. That's $(a,b)=(2,2),(2,3)$.
If $a=2$ then we want to solve $(2b-3)<5$ or $b<4$. That's $(a,b)=(2,2),(2,3)$.
copeland
2015-03-27 20:00:32
If $a=3$ what are the solutions?
If $a=3$ what are the solutions?
swirlykick
2015-03-27 20:01:14
no solutions
no solutions
awesomemathlete
2015-03-27 20:01:14
none
none
_--__--_
2015-03-27 20:01:14
b<7/3, so no solutions.
b<7/3, so no solutions.
dantx5
2015-03-27 20:01:14
none
none
henryweng
2015-03-27 20:01:14
but a<=b so no solution
but a<=b so no solution
Rmink41
2015-03-27 20:01:14
None
None
24iam24
2015-03-27 20:01:14
no solution
no solution
copeland
2015-03-27 20:01:16
If $a=3$ then we want to solve $3(2b-3)<5$ or $6b-9<5$. That simplifies to \[b<\dfrac{7}3\] so there aren't any new solutions with $b\geq 3$.
If $a=3$ then we want to solve $3(2b-3)<5$ or $6b-9<5$. That simplifies to \[b<\dfrac{7}3\] so there aren't any new solutions with $b\geq 3$.
copeland
2015-03-27 20:01:20
We're done. If $a,b>3$ then $(2a-3)(2b-3)>9$ and we fail.
We're done. If $a,b>3$ then $(2a-3)(2b-3)>9$ and we fail.
copeland
2015-03-27 20:01:22
The only solutions to the first inequality with $a\leq b$ are
$\bullet$ $(1,b)$ for all $b$
$\bullet$ $(2,2)$, $(2,3)$
The only solutions to the first inequality with $a\leq b$ are
$\bullet$ $(1,b)$ for all $b$
$\bullet$ $(2,2)$, $(2,3)$
copeland
2015-03-27 20:01:27
What is the maximum possible value of $\dfrac{1^3+b^3}{1^3b^3+1}$?
What is the maximum possible value of $\dfrac{1^3+b^3}{1^3b^3+1}$?
ShadowQueenPeach
2015-03-27 20:02:00
1
1
tdeng
2015-03-27 20:02:00
1
1
swirlykick
2015-03-27 20:02:00
1
1
Darn
2015-03-27 20:02:00
1 lol
1 lol
joey8189681
2015-03-27 20:02:00
1
1
simon1221
2015-03-27 20:02:00
1
1
nosaj
2015-03-27 20:02:00
1
1
rt03
2015-03-27 20:02:00
1
1
Aang
2015-03-27 20:02:00
1
1
vinayak-kumar
2015-03-27 20:02:00
it will always be 1 haha
it will always be 1 haha
dhruv
2015-03-27 20:02:00
1
1
czhu000
2015-03-27 20:02:00
1
1
hlasker1
2015-03-27 20:02:00
1
1
mathwizard888
2015-03-27 20:02:00
1
1
jam10307
2015-03-27 20:02:00
1
1
copeland
2015-03-27 20:02:02
This is always equal to 1 so the maximum is 1.
This is always equal to 1 so the maximum is 1.
copeland
2015-03-27 20:02:05
What is the maximum possible value if $a=2$?
What is the maximum possible value if $a=2$?
Eugenis
2015-03-27 20:03:00
Hmm, do we really have to chug through all these cases?
Hmm, do we really have to chug through all these cases?
24iam24
2015-03-27 20:03:00
217/35=31/5
217/35=31/5
swirlykick
2015-03-27 20:03:00
oops, i mean 217/35=31/5
oops, i mean 217/35=31/5
henryweng
2015-03-27 20:03:00
max=31/5
max=31/5
chessderek
2015-03-27 20:03:00
31/5
31/5
simon1221
2015-03-27 20:03:00
31/5?
31/5?
cpma213
2015-03-27 20:03:00
31/5
31/5
High
2015-03-27 20:03:00
b= 3 and (a^3*b^3+1)(a^3+b^3) = 217/35 = 31/5
b= 3 and (a^3*b^3+1)(a^3+b^3) = 217/35 = 31/5
Eugenis
2015-03-27 20:03:00
$\frac{31}{5}$
$\frac{31}{5}$
ompatel99
2015-03-27 20:03:00
31/5
31/5
joey8189681
2015-03-27 20:03:00
217/35
217/35
vinayak-kumar
2015-03-27 20:03:00
(216+1)/(8+27)=31/5
(216+1)/(8+27)=31/5
bluephoenix
2015-03-27 20:03:00
217/35
217/35
tdeng
2015-03-27 20:03:00
217/35
217/35
copeland
2015-03-27 20:03:04
For $b=2$ we have \[\dfrac{2^32^3+1}{2^3+2^3}=\dfrac{8\cdot8+1}{8+8}=\dfrac{65}{16}\]
and $b=3$ gives \[\dfrac{2^33^3+1}{2^3+3^3}=\dfrac{217}{35}.\] The first one is around 4 and the second one is around 6 so the second is larger.
For $b=2$ we have \[\dfrac{2^32^3+1}{2^3+2^3}=\dfrac{8\cdot8+1}{8+8}=\dfrac{65}{16}\]
and $b=3$ gives \[\dfrac{2^33^3+1}{2^3+3^3}=\dfrac{217}{35}.\] The first one is around 4 and the second one is around 6 so the second is larger.
copeland
2015-03-27 20:03:08
The maximum value is \[\dfrac{217}{35}=\dfrac{31}{5}\] so the answer is $31+5=\boxed{36}$.
The maximum value is \[\dfrac{217}{35}=\dfrac{31}{5}\] so the answer is $31+5=\boxed{36}$.
bluephoenix
2015-03-27 20:03:48
Yay, 8/15 of the way there!!
Yay, 8/15 of the way there!!
copeland
2015-03-27 20:03:51
Alright.
Alright.
copeland
2015-03-27 20:03:58
Let's keep rocking.
Let's keep rocking.
copeland
2015-03-27 20:04:02
9. A cylindrical barrel with radius 4 feet and height 10 feet is full of water. A solid cube with side length 8 feet is set into the barrel so that the diagonal of the cube is vertical. The volume of water thus displaced is $v$ cubic feet. Find $v^2$.
9. A cylindrical barrel with radius 4 feet and height 10 feet is full of water. A solid cube with side length 8 feet is set into the barrel so that the diagonal of the cube is vertical. The volume of water thus displaced is $v$ cubic feet. Find $v^2$.
copeland
2015-03-27 20:04:05
copeland
2015-03-27 20:04:08
Thanks to AoPS community member chezbgone2 for the great picture that he or she posted in the community and that we stol...er, I mean, appropriated (under the "publicly perform" subclause of article 7 of the AoPS Terms of Service of course) for this Math Jam.
Thanks to AoPS community member chezbgone2 for the great picture that he or she posted in the community and that we stol...er, I mean, appropriated (under the "publicly perform" subclause of article 7 of the AoPS Terms of Service of course) for this Math Jam.
copeland
2015-03-27 20:04:19
Everybody say, "Way to go, chez!"
Everybody say, "Way to go, chez!"
High
2015-03-27 20:04:49
way to go chez
way to go chez
RegretDragunity
2015-03-27 20:04:49
Way to go, chez!!
Way to go, chez!!
henryweng
2015-03-27 20:04:49
way to go
way to go
ThePiPie
2015-03-27 20:04:49
way to go, chez!
way to go, chez!
bluephoenix
2015-03-27 20:04:49
Way to go, chez!!!!!!!!!!!!!!!!!!1
Way to go, chez!!!!!!!!!!!!!!!!!!1
_--__--_
2015-03-27 20:04:49
Way to go, chez!
Way to go, chez!
flyrain
2015-03-27 20:04:49
Way to go, chez!
Way to go, chez!
ssk9208
2015-03-27 20:04:49
Way to go, chez!
Way to go, chez!
Tommy2000
2015-03-27 20:04:49
GO CHEZ!!!!!!!
GO CHEZ!!!!!!!
cobbler
2015-03-27 20:04:49
Way to go, cheese!
Way to go, cheese!
tdeng
2015-03-27 20:04:49
way to go chez!
way to go chez!
azmath333
2015-03-27 20:04:49
Way to go, chez!
Way to go, chez!
chessderek
2015-03-27 20:04:49
Way to go, chezbgone2!
Way to go, chezbgone2!
C-bass
2015-03-27 20:04:49
way to go, chez
way to go, chez
SockFoot
2015-03-27 20:04:49
WAY TO G0O, CHEZ!!!!!!!!!
WAY TO G0O, CHEZ!!!!!!!!!
IsaacZ123
2015-03-27 20:04:49
CHEZ I LOVE YOU
CHEZ I LOVE YOU
cobbler
2015-03-27 20:04:49
chez, sorry
chez, sorry
vinayak-kumar
2015-03-27 20:04:49
Way to go, chez!
Way to go, chez!
yunchao
2015-03-27 20:04:49
Way to go, chez!
Way to go, chez!
rt03
2015-03-27 20:04:49
Way to go, chez!
Way to go, chez!
24iam24
2015-03-27 20:04:49
Go chez!
Go chez!
pops2724
2015-03-27 20:04:49
way to go, chez1
way to go, chez1
owm
2015-03-27 20:04:54
Way to go Chez!
Way to go Chez!
xayy
2015-03-27 20:04:54
way to go chez
way to go chez
caressezhu
2015-03-27 20:04:54
Way to go, chez
Way to go, chez
copeland
2015-03-27 20:04:58
The volume we're looking for is the volume of the portion of the cube that lies below the rim of the cylinder.
The volume we're looking for is the volume of the portion of the cube that lies below the rim of the cylinder.
copeland
2015-03-27 20:05:08
What's the cross-section of the cube look like in the plane of the cylinder's top?
What's the cross-section of the cube look like in the plane of the cylinder's top?
math_cool
2015-03-27 20:05:42
Equilateral triangle
Equilateral triangle
hnkevin42
2015-03-27 20:05:42
an equilateral triangle?
an equilateral triangle?
Borealis
2015-03-27 20:05:42
An equilateral triangle!
An equilateral triangle!
Abeymom
2015-03-27 20:05:42
triangle equilateral
triangle equilateral
shiningsunnyday
2015-03-27 20:05:42
Equilateral triangle
Equilateral triangle
kaserav
2015-03-27 20:05:42
An equilateral triangle.
An equilateral triangle.
swe1
2015-03-27 20:05:42
An equilateral triangle
An equilateral triangle
math-rules
2015-03-27 20:05:42
equilateral triangle
equilateral triangle
copeland
2015-03-27 20:05:45
By the symmetry of the cube, the three edges that are adjacent to the sunken vertex hit the cylinder at points that are equally-spaced around the rim.
By the symmetry of the cube, the three edges that are adjacent to the sunken vertex hit the cylinder at points that are equally-spaced around the rim.
vinayak-kumar
2015-03-27 20:05:55
An equilateral triangle with its circumcircle of radius 4
An equilateral triangle with its circumcircle of radius 4
copeland
2015-03-27 20:05:57
So the cross-section is an equilateral triangle inscribed in the rim, which is a circle of radius 4:
So the cross-section is an equilateral triangle inscribed in the rim, which is a circle of radius 4:
copeland
2015-03-27 20:06:01
copeland
2015-03-27 20:06:05
What's the side length of that triangle?
What's the side length of that triangle?
phultrix
2015-03-27 20:06:35
4sqrt(3)
4sqrt(3)
cpma213
2015-03-27 20:06:35
4sqrt(3)
4sqrt(3)
Darn
2015-03-27 20:06:35
4sqrt3
4sqrt3
acegikmoqsuwy2000
2015-03-27 20:06:35
4sqrt3
4sqrt3
czhu000
2015-03-27 20:06:35
$4\sqrt{3}$
$4\sqrt{3}$
lovejj
2015-03-27 20:06:35
4sqrt3
4sqrt3
_--__--_
2015-03-27 20:06:35
4sqrt(3)
4sqrt(3)
Poseidon2001
2015-03-27 20:06:35
4sqrt3
4sqrt3
Aviously
2015-03-27 20:06:35
$4\sqrt{3}$
$4\sqrt{3}$
lcsmart
2015-03-27 20:06:37
4sqrt3
4sqrt3
copeland
2015-03-27 20:06:39
If you don't see it right away, extend an altitude from the center of circle to one of the sides:
If you don't see it right away, extend an altitude from the center of circle to one of the sides:
copeland
2015-03-27 20:06:41
copeland
2015-03-27 20:06:43
The small triangle is 30-60-90, so half of the big triangle's side length is $2\sqrt3$, and thus the big triangle has side length $4\sqrt3$.
The small triangle is 30-60-90, so half of the big triangle's side length is $2\sqrt3$, and thus the big triangle has side length $4\sqrt3$.
copeland
2015-03-27 20:06:59
(Sanity check: It's definitely between 4 and 8.)
(Sanity check: It's definitely between 4 and 8.)
copeland
2015-03-27 20:07:01
Great -- now what?
Great -- now what?
SockFoot
2015-03-27 20:07:50
find height of the tetrahedron
find height of the tetrahedron
High
2015-03-27 20:07:50
Find the height of the pyramid
Find the height of the pyramid
uberminecraft722
2015-03-27 20:07:50
Construct a pyramid using the triangle
Construct a pyramid using the triangle
acegikmoqsuwy2000
2015-03-27 20:07:50
now we find the volume of the tetrahedron inside the cylinder by finding the height
now we find the volume of the tetrahedron inside the cylinder by finding the height
raptorw
2015-03-27 20:07:50
find the height of the tetrahedron
find the height of the tetrahedron
Rmehtany
2015-03-27 20:07:50
find the height of the pyramid
find the height of the pyramid
shiningsunnyday
2015-03-27 20:07:50
Volume of terahedron
Volume of terahedron
Rmehtany
2015-03-27 20:07:50
tetrahedron
tetrahedron
copeland
2015-03-27 20:07:59
Imaging slicing the piece of the cube that's below the rim of the cylinder.
Imaging slicing the piece of the cube that's below the rim of the cylinder.
copeland
2015-03-27 20:08:01
If we turn it upside-down and set it on a table, it's a pyramid whose base is an equilateral triangle with side length $4\sqrt3$.
If we turn it upside-down and set it on a table, it's a pyramid whose base is an equilateral triangle with side length $4\sqrt3$.
copeland
2015-03-27 20:08:02
What do the other three faces look like?
What do the other three faces look like?
nosaj
2015-03-27 20:08:30
right iscoscles
right iscoscles
IsaacZ123
2015-03-27 20:08:30
45-45-90 triangles
45-45-90 triangles
High
2015-03-27 20:08:30
isosceles right triangles?
isosceles right triangles?
swe1
2015-03-27 20:08:30
Isosceles right triangles
Isosceles right triangles
raptorw
2015-03-27 20:08:30
45-45-90 triangles
45-45-90 triangles
maverick8
2015-03-27 20:08:30
45-45-90 right triangles
45-45-90 right triangles
SockFoot
2015-03-27 20:08:30
45 45 90 right triangles
45 45 90 right triangles
copeland
2015-03-27 20:08:33
They're all isosceles right triangles. (They're triangular corners of faces of the original cube.)
They're all isosceles right triangles. (They're triangular corners of faces of the original cube.)
copeland
2015-03-27 20:08:34
So what are the edge lengths?
So what are the edge lengths?
maverick8
2015-03-27 20:09:06
2 sqrt 6
2 sqrt 6
Rmehtany
2015-03-27 20:09:06
2rad6
2rad6
uberminecraft722
2015-03-27 20:09:06
2sqrt(6)
2sqrt(6)
Abeymom
2015-03-27 20:09:06
2 sqrt 6
2 sqrt 6
Aviously
2015-03-27 20:09:06
$2\sqrt{6}$
$2\sqrt{6}$
malarm
2015-03-27 20:09:06
2rt6
2rt6
Darn
2015-03-27 20:09:06
So it's $2\sqrt6$
So it's $2\sqrt6$
copeland
2015-03-27 20:09:08
The legs of an isosceles right triangle are each $\dfrac{1}{\sqrt2}$ times the hypotenuse. Thus, they're $\dfrac{1}{\sqrt2} \cdot 4\sqrt{3} = 2\sqrt6$.
The legs of an isosceles right triangle are each $\dfrac{1}{\sqrt2}$ times the hypotenuse. Thus, they're $\dfrac{1}{\sqrt2} \cdot 4\sqrt{3} = 2\sqrt6$.
copeland
2015-03-27 20:09:12
How do we finish?
How do we finish?
Abeymom
2015-03-27 20:10:03
look for height using pyth
look for height using pyth
uberminecraft722
2015-03-27 20:10:03
find the height, then the volume
find the height, then the volume
maverick8
2015-03-27 20:10:03
Base*height/3
Base*height/3
rt03
2015-03-27 20:10:03
find the height
find the height
copeland
2015-03-27 20:10:09
Base and height and, yeah.
Base and height and, yeah.
copeland
2015-03-27 20:10:11
What's the base?
What's the base?
copeland
2015-03-27 20:10:30
I mean, qualitatively, which base?
I mean, qualitatively, which base?
vinayak-kumar
2015-03-27 20:10:55
the equilateral triangle
the equilateral triangle
Rmehtany
2015-03-27 20:10:55
equilateral
equilateral
azmath333
2015-03-27 20:10:55
Equilateral triangle
Equilateral triangle
uberminecraft722
2015-03-27 20:10:55
The equilateral triangle base
The equilateral triangle base
IsaacZ123
2015-03-27 20:10:55
the 45-45-90 base
the 45-45-90 base
Darn
2015-03-27 20:10:55
oh it's the 45-45-90
oh it's the 45-45-90
maverick8
2015-03-27 20:10:55
A 45-45-90
A 45-45-90
swe1
2015-03-27 20:10:55
One of the isosceles right triangles, it's easier
One of the isosceles right triangles, it's easier
copeland
2015-03-27 20:11:06
Either works, actually. Let's use the 45-45-90 base.
Either works, actually. Let's use the 45-45-90 base.
copeland
2015-03-27 20:11:14
If we set our pyramid flat onto one of the bases that was originally a side of the cube, we have a right triangular pyramid: a pyramid in which three edges are mutually perpendicular.
If we set our pyramid flat onto one of the bases that was originally a side of the cube, we have a right triangular pyramid: a pyramid in which three edges are mutually perpendicular.
copeland
2015-03-27 20:11:16
copeland
2015-03-27 20:11:20
What's the volume of this?
What's the volume of this?
simon1221
2015-03-27 20:12:07
1/3 base height
1/3 base height
Darn
2015-03-27 20:12:07
Equal to $(2\sqrt{6})^3/6$
Equal to $(2\sqrt{6})^3/6$
Rmehtany
2015-03-27 20:12:07
8rad6
8rad6
rt03
2015-03-27 20:12:07
8sqrt6
8sqrt6
tdeng
2015-03-27 20:12:07
8sqrt6?
8sqrt6?
_--__--_
2015-03-27 20:12:07
8sqrt(6)?
8sqrt(6)?
vinayak-kumar
2015-03-27 20:12:07
2rt(6)^3/6=8rt(6)
2rt(6)^3/6=8rt(6)
Tommy2000
2015-03-27 20:12:07
$(2\sqrt6)^3/6=8sqrt6$
$(2\sqrt6)^3/6=8sqrt6$
nosaj
2015-03-27 20:12:07
1/3 bh = 1/3*1/2*(2sqrt6)^3
1/3 bh = 1/3*1/2*(2sqrt6)^3
swe1
2015-03-27 20:12:07
$\frac{(2\sqrt{6})^3}{6} = 8\sqrt{6}$
$\frac{(2\sqrt{6})^3}{6} = 8\sqrt{6}$
copeland
2015-03-27 20:12:10
It's just $\frac16$ times the product of the lengths of the three mutually perpendicular sides.
It's just $\frac16$ times the product of the lengths of the three mutually perpendicular sides.
copeland
2015-03-27 20:12:11
These sides all have length $2\sqrt6$, so our volume is
\[
\frac16 \cdot (2\sqrt6)^3 = 8\sqrt6.
\]
These sides all have length $2\sqrt6$, so our volume is
\[
\frac16 \cdot (2\sqrt6)^3 = 8\sqrt6.
\]
copeland
2015-03-27 20:12:16
Our final answer is the volume squared, so it's $(8\sqrt6)^2 = 64 \cdot 6 = \boxed{384}$.
Our final answer is the volume squared, so it's $(8\sqrt6)^2 = 64 \cdot 6 = \boxed{384}$.
Darn
2015-03-27 20:12:35
3/5 done!!
3/5 done!!
chessderek
2015-03-27 20:12:35
3/5 done
3/5 done
bluephoenix
2015-03-27 20:12:35
oh boy, 3/5 of the way
oh boy, 3/5 of the way
Dayranger
2015-03-27 20:12:35
3/5 way through!
3/5 way through!
copeland
2015-03-27 20:12:42
You guys are killing the fractions part today, too.
You guys are killing the fractions part today, too.
copeland
2015-03-27 20:12:53
10. Call a permutation $a_1, a_2, \ldots, a_n$ of the integers $1,2,\ldots, n$ quasi-increasing if $a_k \le a_{k+1} + 2$ for each $1 \le k \le n - 1$. For example, $54321$ and $14253$ are quasi-increasing permutations of the integers $1,2,3,4,5,$ but $45123$ is not. Find the number of quasi-increasing permutations of the integers $1,2, \ldots, 7$.
10. Call a permutation $a_1, a_2, \ldots, a_n$ of the integers $1,2,\ldots, n$ quasi-increasing if $a_k \le a_{k+1} + 2$ for each $1 \le k \le n - 1$. For example, $54321$ and $14253$ are quasi-increasing permutations of the integers $1,2,3,4,5,$ but $45123$ is not. Find the number of quasi-increasing permutations of the integers $1,2, \ldots, 7$.
copeland
2015-03-27 20:12:58
How might we get started?
How might we get started?
swirlykick
2015-03-27 20:13:15
small cases
small cases
ompatel99
2015-03-27 20:13:15
Small cases?
Small cases?
ShadowQueenPeach
2015-03-27 20:13:15
try smaller cases
try smaller cases
BPM14
2015-03-27 20:13:15
Make small cases with 1, then 2 numbers, etc.
Make small cases with 1, then 2 numbers, etc.
SockFoot
2015-03-27 20:13:15
test small cases
test small cases
copeland
2015-03-27 20:13:18
Trying a few smaller cases sounds like a good idea. How many quasi-increasing permutations are there of length 1?
Trying a few smaller cases sounds like a good idea. How many quasi-increasing permutations are there of length 1?
Aviously
2015-03-27 20:13:35
I
I
AlisonH
2015-03-27 20:13:35
1
1
tdeng
2015-03-27 20:13:35
1
1
czhu000
2015-03-27 20:13:35
1
1
Abeymom
2015-03-27 20:13:35
1
1
gxah
2015-03-27 20:13:35
1
1
Superwiz
2015-03-27 20:13:36
1
1
copeland
2015-03-27 20:13:38
Every permutation of length 1 is quasi-increasing. In fact, for $n \le 3$ we see every permutation of length $n$ is quasi-increasing.
Every permutation of length 1 is quasi-increasing. In fact, for $n \le 3$ we see every permutation of length $n$ is quasi-increasing.
copeland
2015-03-27 20:13:42
Things really start to get interesting at $n = 4$. How many quasi-increasing permutations of length 4 are there?
Things really start to get interesting at $n = 4$. How many quasi-increasing permutations of length 4 are there?
copeland
2015-03-27 20:13:52
That's a bit more difficult, but it still seems like most sequences are quasi-increasing. How can a permutation of $1,2,3,4$ not be quasi-increasing?
That's a bit more difficult, but it still seems like most sequences are quasi-increasing. How can a permutation of $1,2,3,4$ not be quasi-increasing?
IsaacZ123
2015-03-27 20:14:49
if 1 is next to 4
if 1 is next to 4
czhu000
2015-03-27 20:14:49
If 4 is followed by 1
If 4 is followed by 1
ShadowQueenPeach
2015-03-27 20:14:49
4 is behind 1
4 is behind 1
maverick8
2015-03-27 20:14:49
A jump from 4 to 1
A jump from 4 to 1
RegretDragunity
2015-03-27 20:14:49
if 4 is followed by 1
if 4 is followed by 1
simon1221
2015-03-27 20:14:49
4123 or anything with 41
4123 or anything with 41
High
2015-03-27 20:14:49
1 comes after 4
1 comes after 4
Rocksolid
2015-03-27 20:14:49
If it contains the pair 41
If it contains the pair 41
henryweng
2015-03-27 20:14:49
4 before 1
4 before 1
mathwrath
2015-03-27 20:14:49
If the 4 is right before the 1
If the 4 is right before the 1
copeland
2015-03-27 20:15:00
The only way a permutation of $1,2,3,4$ can fail to be quasi-increasing is if the 4 immediately precedes the 1.
The only way a permutation of $1,2,3,4$ can fail to be quasi-increasing is if the 4 immediately precedes the 1.
copeland
2015-03-27 20:15:01
Does this give any insight on how we might count the number of quasi-increasing permutations of length 4?
Does this give any insight on how we might count the number of quasi-increasing permutations of length 4?
Aviously
2015-03-27 20:15:27
$24-6=18$ for length of 4.
$24-6=18$ for length of 4.
Tommy2000
2015-03-27 20:15:27
total - failures
total - failures
swe1
2015-03-27 20:15:27
Complementary counting
Complementary counting
uberminecraft722
2015-03-27 20:15:27
count the wrong answers and subtract
count the wrong answers and subtract
shiningsunnyday
2015-03-27 20:15:27
Complement
Complement
_--__--_
2015-03-27 20:15:27
Casework, 24-6 = 18
Casework, 24-6 = 18
High
2015-03-27 20:15:27
4! - 3! = 18
4! - 3! = 18
copeland
2015-03-27 20:15:32
Well, we could just take the total number of permutations (there are 24) and subtract the number with 1 immediately preceding 4 (there are 6). This gives us $24 - 6 = 18$ permutations of length 4.
Well, we could just take the total number of permutations (there are 24) and subtract the number with 1 immediately preceding 4 (there are 6). This gives us $24 - 6 = 18$ permutations of length 4.
copeland
2015-03-27 20:15:41
However, is there any way we could count this more constructively given the information we know?
However, is there any way we could count this more constructively given the information we know?
copeland
2015-03-27 20:15:59
I get the feeling that recursion will be more valuable than complementary counting when $n$ becomes large.
I get the feeling that recursion will be more valuable than complementary counting when $n$ becomes large.
copeland
2015-03-27 20:16:04
How might we get from a permutation of length 3 to a permutation of length 4?
How might we get from a permutation of length 3 to a permutation of length 4?
swirlykick
2015-03-27 20:16:24
we start with a permutation of 3 that works, then we add 4 at the end, or before 2,3
we start with a permutation of 3 that works, then we add 4 at the end, or before 2,3
vinayak-kumar
2015-03-27 20:16:24
find a way to insert the largest numer
find a way to insert the largest numer
ShadowQueenPeach
2015-03-27 20:16:24
insert a 4
insert a 4
temp8909
2015-03-27 20:16:24
add 4
add 4
SockFoot
2015-03-27 20:16:24
add a $4$ somewhere that isn't bad
add a $4$ somewhere that isn't bad
cxiong
2015-03-27 20:16:24
add a 4 somewhere
add a 4 somewhere
Tommy2000
2015-03-27 20:16:24
We have locations to place the 4 in
We have locations to place the 4 in
lcsmart
2015-03-27 20:16:24
implanting a term $4$
implanting a term $4$
copeland
2015-03-27 20:16:30
If we take any permutation of length 3, we can create a quasi-increasing permutation of length 4 by inserting the 4 in any position except immediately before the 1.
If we take any permutation of length 3, we can create a quasi-increasing permutation of length 4 by inserting the 4 in any position except immediately before the 1.
copeland
2015-03-27 20:16:32
For example, if we had the permutation 123, we could stick the 4 in any blank except the first: __1__2__3__.
For example, if we had the permutation 123, we could stick the 4 in any blank except the first: __1__2__3__.
copeland
2015-03-27 20:16:38
This gives us 3 quasi-increasing permutations of length 4 for each permutation of length 3.
This gives us 3 quasi-increasing permutations of length 4 for each permutation of length 3.
copeland
2015-03-27 20:16:40
Can we generalize this to permutations of length $n$?
Can we generalize this to permutations of length $n$?
swirlykick
2015-03-27 20:18:06
P(n)=3P(n-1)
P(n)=3P(n-1)
swe1
2015-03-27 20:18:06
Add next number to end, or before $n-1$ or $n-2$
Add next number to end, or before $n-1$ or $n-2$
High
2015-03-27 20:18:06
permutations of n = 3*permutations of n - 1
permutations of n = 3*permutations of n - 1
mathwrath
2015-03-27 20:18:06
Yes: Before n-2, n-1, or at the end
Yes: Before n-2, n-1, or at the end
Abeymom
2015-03-27 20:18:06
multiply by three evertytime
multiply by three evertytime
24iam24
2015-03-27 20:18:06
is it always 3f(n-1)?
is it always 3f(n-1)?
czhu000
2015-03-27 20:18:06
It's $3 \times$ (the number of ways for $n-1$)
It's $3 \times$ (the number of ways for $n-1$)
DivideBy0
2015-03-27 20:18:06
there are always 3 places to insert n
there are always 3 places to insert n
chessderek
2015-03-27 20:18:06
It seems like a geometric sequence with a ratio of 3.
It seems like a geometric sequence with a ratio of 3.
Tommy2000
2015-03-27 20:18:06
$S_n=3S_{n-1}$
$S_n=3S_{n-1}$
copeland
2015-03-27 20:18:11
Yes! In fact, it generalizes immediately. If we take any quasi-increasing permutation of length $n-1$, we can place $n$ before $n-1$ or $n-2$, or at the end of the permutation to produce a quasi-increasing permutation of length $n$.
Yes! In fact, it generalizes immediately. If we take any quasi-increasing permutation of length $n-1$, we can place $n$ before $n-1$ or $n-2$, or at the end of the permutation to produce a quasi-increasing permutation of length $n$.
copeland
2015-03-27 20:18:13
So we get 3 quasi-increasing permutations of length $n$ for each quasi-increasing permutation of length $n-1$.
So we get 3 quasi-increasing permutations of length $n$ for each quasi-increasing permutation of length $n-1$.
copeland
2015-03-27 20:18:17
Moreover, if we erase $n$ in any quasi-increasing permutation of length $n$, we get a quasi-increasing permutation of length $n - 1$. So this gives us all of the quasi-increasing permutations of length $n$.
Moreover, if we erase $n$ in any quasi-increasing permutation of length $n$, we get a quasi-increasing permutation of length $n - 1$. So this gives us all of the quasi-increasing permutations of length $n$.
copeland
2015-03-27 20:18:29
That is, if we let $S_n$ be the number of quasi-increasing permutations of length $n$, we have $S_n = 3\cdot S_{n-1}$.
That is, if we let $S_n$ be the number of quasi-increasing permutations of length $n$, we have $S_n = 3\cdot S_{n-1}$.
copeland
2015-03-27 20:18:33
(Note that we need $n\ge 3$ here -- our argument clearly breaks down for $n = 1$ or $2$.)
(Note that we need $n\ge 3$ here -- our argument clearly breaks down for $n = 1$ or $2$.)
copeland
2015-03-27 20:18:34
So how do we get $S_{7}$?
So how do we get $S_{7}$?
Darn
2015-03-27 20:19:34
Keep multiplying by $3$!
Keep multiplying by $3$!
BPM14
2015-03-27 20:19:34
468 = 3^6*2
468 = 3^6*2
24iam24
2015-03-27 20:19:34
18x3x3x3
18x3x3x3
_--__--_
2015-03-27 20:19:34
S7 = S4 * 3 * 3 * 3 = 18 * 3 * 3 * 3 = 486
S7 = S4 * 3 * 3 * 3 = 18 * 3 * 3 * 3 = 486
Tommy2000
2015-03-27 20:19:34
so our answer 6*3^4=486
so our answer 6*3^4=486
chessderek
2015-03-27 20:19:34
we can multiply 18 by 3^3 to get 486
we can multiply 18 by 3^3 to get 486
DivideBy0
2015-03-27 20:19:34
Abeymom
2015-03-27 20:19:34
6*3^4
6*3^4
maverick8
2015-03-27 20:19:34
18*27
18*27
Dracae8
2015-03-27 20:19:34
2*3^5
2*3^5
High
2015-03-27 20:19:34
S_4 = 18, s_5 = 54, S_6= 162, S_7=486
S_4 = 18, s_5 = 54, S_6= 162, S_7=486
copeland
2015-03-27 20:19:36
We already know $S_4 = 18$, so we just start multiplying by 3 until we get to $S_{7} = 18\cdot 3^{3} = \boxed{486}$.
We already know $S_4 = 18$, so we just start multiplying by 3 until we get to $S_{7} = 18\cdot 3^{3} = \boxed{486}$.
24iam24
2015-03-27 20:20:06
2/3 of the way there!
2/3 of the way there!
chessderek
2015-03-27 20:20:06
2/3!
2/3!
IsaacZ123
2015-03-27 20:20:28
This problem looked tougher than it actually was
This problem looked tougher than it actually was
copeland
2015-03-27 20:20:41
There were a lot of problems on this that were either easier or harder than expected.
There were a lot of problems on this that were either easier or harder than expected.
copeland
2015-03-27 20:20:51
You never now until you solve them I guess.
You never now until you solve them I guess.
copeland
2015-03-27 20:20:54
11. The circumcircle of acute $\triangle ABC$ has center $O$. The line passing through point $O$ perpendicular to $\overline{OB}$ intersects lines $AB$ and $BC$ at $P$ and $Q$ respectively. Also $AB=5$, $BC=4$, $BQ=4.5$, and $BP=\dfrac mn$ where $m$ and $n$ are relatively prime positive integers. Find $m+n$.
11. The circumcircle of acute $\triangle ABC$ has center $O$. The line passing through point $O$ perpendicular to $\overline{OB}$ intersects lines $AB$ and $BC$ at $P$ and $Q$ respectively. Also $AB=5$, $BC=4$, $BQ=4.5$, and $BP=\dfrac mn$ where $m$ and $n$ are relatively prime positive integers. Find $m+n$.
copeland
2015-03-27 20:20:59
Let's start with a diagram, of course.
Let's start with a diagram, of course.
copeland
2015-03-27 20:21:05
copeland
2015-03-27 20:21:10
The diagram uses that $\triangle ABC$ is acute: If $P$ is not between $A$ and $B$ then $A$, $B$, and $C$ are all "above" the diameter meaning the triangle is obtuse.
The diagram uses that $\triangle ABC$ is acute: If $P$ is not between $A$ and $B$ then $A$, $B$, and $C$ are all "above" the diameter meaning the triangle is obtuse.
copeland
2015-03-27 20:21:21
(Those lengths on $BC$ and $BQ$ are a little confusing, sorry.)
(Those lengths on $BC$ and $BQ$ are a little confusing, sorry.)
copeland
2015-03-27 20:21:25
OK, so we have a circle. We have lines intersecting at points and we have distances. What tool pops into mind now?
OK, so we have a circle. We have lines intersecting at points and we have distances. What tool pops into mind now?
_--__--_
2015-03-27 20:21:37
You said "now", isn't it "know"?
You said "now", isn't it "know"?
tdeng
2015-03-27 20:21:37
we never now?
we never now?
SockFoot
2015-03-27 20:21:37
yeah man you never now
yeah man you never now
copeland
2015-03-27 20:21:40
Never now.
Never now.
copeland
2015-03-27 20:21:42
Always later.
Always later.
bluephoenix
2015-03-27 20:21:50
Power of a Point
Power of a Point
Rmehtany
2015-03-27 20:21:50
power of point
power of point
_--__--_
2015-03-27 20:21:50
Power of a Point
Power of a Point
tdeng
2015-03-27 20:21:50
power of a point
power of a point
cxiong
2015-03-27 20:21:50
power of a point?
power of a point?
Darn
2015-03-27 20:21:50
Power of a Point
Power of a Point
HANRUI11122001
2015-03-27 20:21:50
POWER OF A POINT
POWER OF A POINT
maverick8
2015-03-27 20:21:50
Pop
Pop
IsaacZ123
2015-03-27 20:21:50
power of a opint
power of a opint
copeland
2015-03-27 20:21:54
Power of a point! What point?
Power of a point! What point?
va2010
2015-03-27 20:22:35
point Q?
point Q?
maverick8
2015-03-27 20:22:35
Q
Q
nosaj
2015-03-27 20:22:35
Q
Q
bluephoenix
2015-03-27 20:22:35
Point Q
Point Q
vinayak-kumar
2015-03-27 20:22:35
Q
Q
azmath333
2015-03-27 20:22:35
Q?
Q?
copeland
2015-03-27 20:22:40
Q is good. What else is good?
Q is good. What else is good?
Rmehtany
2015-03-27 20:23:01
P for point
P for point
nosaj
2015-03-27 20:23:01
P
P
henryweng
2015-03-27 20:23:01
P
P
Rmehtany
2015-03-27 20:23:01
P
P
va2010
2015-03-27 20:23:01
P
P
copeland
2015-03-27 20:23:04
Both $P$ and $Q$ lie on two lines so let's use POP on $P$ and $Q$. Let me label some more edges.
Both $P$ and $Q$ lie on two lines so let's use POP on $P$ and $Q$. Let me label some more edges.
copeland
2015-03-27 20:23:06
Let's actually get the interesting distances in there, $BC$, $CQ$, $PA$, and $PB$:
Let's actually get the interesting distances in there, $BC$, $CQ$, $PA$, and $PB$:
copeland
2015-03-27 20:23:08
copeland
2015-03-27 20:23:13
What other length do we need?
What other length do we need?
Rmehtany
2015-03-27 20:24:10
OP
OP
amburger66
2015-03-27 20:24:10
radius?
radius?
zvxcqu
2015-03-27 20:24:10
OB
OB
Dayranger
2015-03-27 20:24:10
OB
OB
acegikmoqsuwy2000
2015-03-27 20:24:10
PQ
PQ
DivideBy0
2015-03-27 20:24:10
extension of PQ till it hits circle on the other side of O
extension of PQ till it hits circle on the other side of O
copeland
2015-03-27 20:24:13
In order to use POP we're also going to need the radius.
In order to use POP we're also going to need the radius.
copeland
2015-03-27 20:24:16
copeland
2015-03-27 20:25:53
With POP it's nice to have the distance from the center because difference of squares factorizations pop out. We'll see.
With POP it's nice to have the distance from the center because difference of squares factorizations pop out. We'll see.
copeland
2015-03-27 20:26:14
So let's also name $PO=u$ and $OQ=v$. What equations do we have?
So let's also name $PO=u$ and $OQ=v$. What equations do we have?
Tommy2000
2015-03-27 20:26:34
x+y=5
x+y=5
copeland
2015-03-27 20:26:37
Clearly we have \[x+y=5.\]
Clearly we have \[x+y=5.\]
IsaacZ123
2015-03-27 20:26:58
.5*4.5=(v-r)(v+r)
.5*4.5=(v-r)(v+r)
mathwrath
2015-03-27 20:26:58
v^2-r^2=9/4
v^2-r^2=9/4
copeland
2015-03-27 20:27:02
POP on $Q$ gives $(v+r)(v-r)=4.5\cdot0.5$ or \[v^2-r^2=\frac94.\]
POP on $Q$ gives $(v+r)(v-r)=4.5\cdot0.5$ or \[v^2-r^2=\frac94.\]
zvxcqu
2015-03-27 20:27:16
u^2+r^2=y^2
u^2+r^2=y^2
RegretDragunity
2015-03-27 20:27:16
u^2+r^2 = y^2
u^2+r^2 = y^2
math-rules
2015-03-27 20:27:31
r^2 + v^2 = (4.5)^2
r^2 + v^2 = (4.5)^2
copeland
2015-03-27 20:27:34
The Pythagorean Theorem on $\triangle POB$ gives \[u^2+r^2=y^2.\]
The Pythagorean Theorem on $\triangle QOB$ gives \[v^2+r^2=\left(\frac92\right)^2=\frac{81}4.\]
The Pythagorean Theorem on $\triangle POB$ gives \[u^2+r^2=y^2.\]
The Pythagorean Theorem on $\triangle QOB$ gives \[v^2+r^2=\left(\frac92\right)^2=\frac{81}4.\]
copeland
2015-03-27 20:28:05
We need one more. . .
We need one more. . .
copeland
2015-03-27 20:28:21
What about POP on $P$?
What about POP on $P$?
zvxcqu
2015-03-27 20:28:42
xy = r^2-u^2
xy = r^2-u^2
IsaacZ123
2015-03-27 20:28:42
xy=(r-u)(r+u)
xy=(r-u)(r+u)
AlisonH
2015-03-27 20:28:42
xy=(r-u)(u+v)
xy=(r-u)(u+v)
copeland
2015-03-27 20:28:48
POP on $P$ gives $(r-u)(r+u)=xy$ or \[r^2-u^2=xy.\]
POP on $P$ gives $(r-u)(r+u)=xy$ or \[r^2-u^2=xy.\]
copeland
2015-03-27 20:28:50
So we get
So we get
copeland
2015-03-27 20:28:51
\begin{eqnarray}
x+y&=5&\qquad(1)\\
r^2-u^2&=xy&\qquad(2)\\
v^2-r^2&=\frac94&\qquad(3)\\
u^2+r^2&=y^2&\qquad(4)\\
v^2+r^2&=\frac{81}4&\qquad(5)
\end{eqnarray}
\begin{eqnarray}
x+y&=5&\qquad(1)\\
r^2-u^2&=xy&\qquad(2)\\
v^2-r^2&=\frac94&\qquad(3)\\
u^2+r^2&=y^2&\qquad(4)\\
v^2+r^2&=\frac{81}4&\qquad(5)
\end{eqnarray}
cobbler
2015-03-27 20:29:15
What tool POPS into mind. I see what you did there ;-]
What tool POPS into mind. I see what you did there ;-]
copeland
2015-03-27 20:29:16
Thank you. Finally. . .
Thank you. Finally. . .
copeland
2015-03-27 20:29:22
We want to find $BP=y$.
We want to find $BP=y$.
copeland
2015-03-27 20:29:23
Which pair of equations gets us closer to $y$?
Which pair of equations gets us closer to $y$?
dhruv
2015-03-27 20:29:51
2 and 4
2 and 4
tdeng
2015-03-27 20:29:51
2,4
2,4
NextEinstein
2015-03-27 20:29:51
2, 4
2, 4
acegikmoqsuwy2000
2015-03-27 20:29:51
(4) and (2)
(4) and (2)
copeland
2015-03-27 20:29:53
Adding (2) and (4) gives
\[2r^2=xy+y^2.\]
Adding (2) and (4) gives
\[2r^2=xy+y^2.\]
copeland
2015-03-27 20:29:54
Now what?
Now what?
nosaj
2015-03-27 20:30:31
Factor RHS
Factor RHS
ShadowQueenPeach
2015-03-27 20:30:31
factor out y and use equation 1
factor out y and use equation 1
DivideBy0
2015-03-27 20:30:31
factor out y and plug in x+y
factor out y and plug in x+y
gxah
2015-03-27 20:30:31
substitute x+y=5
substitute x+y=5
copeland
2015-03-27 20:30:35
We can substitute $x+y=5$ to get
\[2r^2=xy+y^2=y(x+y)=5y.\]
We can substitute $x+y=5$ to get
\[2r^2=xy+y^2=y(x+y)=5y.\]
copeland
2015-03-27 20:30:37
OK, now all we need is $r^2$. How do we get that?
OK, now all we need is $r^2$. How do we get that?
tdeng
2015-03-27 20:31:00
subtract 3 and 5
subtract 3 and 5
IsaacZ123
2015-03-27 20:31:00
3 and 5?
3 and 5?
24iam24
2015-03-27 20:31:00
subtract 3 and 5
subtract 3 and 5
Tommy2000
2015-03-27 20:31:00
subtract 5 and 3
subtract 5 and 3
Darn
2015-03-27 20:31:00
equations (3) and (5)
equations (3) and (5)
amburger66
2015-03-27 20:31:00
3 and 5
3 and 5
copeland
2015-03-27 20:31:02
Subtracting (3) from (5) gives
\[
2r^2=\frac{81}4-\frac94=\frac{72}4=18.
\]
Subtracting (3) from (5) gives
\[
2r^2=\frac{81}4-\frac94=\frac{72}4=18.
\]
copeland
2015-03-27 20:31:03
And what is $y$?
And what is $y$?
acegikmoqsuwy2000
2015-03-27 20:31:52
18/5
18/5
_--__--_
2015-03-27 20:31:52
y = 18/5
y = 18/5
IsaacZ123
2015-03-27 20:31:52
18/5
18/5
SockFoot
2015-03-27 20:31:52
$\dfrac{18}{5}$
$\dfrac{18}{5}$
DivideBy0
2015-03-27 20:31:52
High
2015-03-27 20:31:52
18/5
18/5
owm
2015-03-27 20:31:52
18/5
18/5
malarm
2015-03-27 20:31:52
18/5
18/5
Mlux
2015-03-27 20:31:52
2*18*18/5
2*18*18/5
Rocksolid
2015-03-27 20:31:52
18/5
18/5
copeland
2015-03-27 20:31:55
\[y=\frac{2r^2}5=\frac{18}5.\]
\[y=\frac{2r^2}5=\frac{18}5.\]
copeland
2015-03-27 20:31:56
The answer is $18+5=\boxed{23}$.
The answer is $18+5=\boxed{23}$.
copeland
2015-03-27 20:32:01
There's a second solution to this problem using similar triangles. To get there you begin with the other possible first instinct on the problem: circumcircles want radii:
There's a second solution to this problem using similar triangles. To get there you begin with the other possible first instinct on the problem: circumcircles want radii:
copeland
2015-03-27 20:32:07
copeland
2015-03-27 20:32:14
Now you have 3 isosceles triangles whose angles are related somehow. Time for an angle chase!
Now you have 3 isosceles triangles whose angles are related somehow. Time for an angle chase!
copeland
2015-03-27 20:32:20
I'll leave that for you to think about on your own.
I'll leave that for you to think about on your own.
ompatel99
2015-03-27 20:32:34
11/15 done. It doesnt simplify
11/15 done. It doesnt simplify
copeland
2015-03-27 20:32:37
Tough.
Tough.
copeland
2015-03-27 20:32:46
Hopefully the rest of them will.
Hopefully the rest of them will.
copeland
2015-03-27 20:32:48
12. There are $2^{10} = 1024$ possible 10-letter strings in which each letter is either an $A$ or a $B$. Find the number of such strings that do not have more than 3 adjacent letters that are identical.
12. There are $2^{10} = 1024$ possible 10-letter strings in which each letter is either an $A$ or a $B$. Find the number of such strings that do not have more than 3 adjacent letters that are identical.
24iam24
2015-03-27 20:33:21
this was an amc 12 vid
this was an amc 12 vid
High
2015-03-27 20:33:21
AMC 12A #22
AMC 12A #22
vinayak-kumar
2015-03-27 20:33:21
Next up... featuring the easy version of AMC 12 problem 22
Next up... featuring the easy version of AMC 12 problem 22
Tommy2000
2015-03-27 20:33:21
AMC 12 #22
AMC 12 #22
nosaj
2015-03-27 20:33:21
The repeat question.
The repeat question.
thequantumguy
2015-03-27 20:33:21
just like AMC 12 2015 number 22
just like AMC 12 2015 number 22
swirlykick
2015-03-27 20:33:21
AMC12 A?
AMC12 A?
bluephoenix
2015-03-27 20:33:21
Isn't this like one of the AMC 12 this year?
Isn't this like one of the AMC 12 this year?
danusv
2015-03-27 20:33:21
Misplaced problem LOL
Misplaced problem LOL
math_cool
2015-03-27 20:33:21
AMC 12 lol
AMC 12 lol
mssmath
2015-03-27 20:33:21
MAA copies itself yeah
MAA copies itself yeah
copeland
2015-03-27 20:33:24
Freebie for you 12A folks. A significantly harder version of this problem appeared as 2015 AMC12A Problem 22. We will speed through this one.
Freebie for you 12A folks. A significantly harder version of this problem appeared as 2015 AMC12A Problem 22. We will speed through this one.
copeland
2015-03-27 20:33:34
Let's try some small cases. Let's set $S(n)$ to be the number of valid strings of length $n$.
Let's try some small cases. Let's set $S(n)$ to be the number of valid strings of length $n$.
copeland
2015-03-27 20:33:36
What is $S(1)$?
What is $S(1)$?
vinayak-kumar
2015-03-27 20:34:05
2
2
High
2015-03-27 20:34:05
2.
2.
mathwizard888
2015-03-27 20:34:05
2
2
Rmehtany
2015-03-27 20:34:05
2
2
nosaj
2015-03-27 20:34:05
2
2
ShadowQueenPeach
2015-03-27 20:34:05
2
2
copeland
2015-03-27 20:34:07
$S(1)=2.$ We can either write $A$ or $B.$
$S(1)=2.$ We can either write $A$ or $B.$
copeland
2015-03-27 20:34:08
What is $S(2)?$
What is $S(2)?$
swirlykick
2015-03-27 20:34:30
4
4
24iam24
2015-03-27 20:34:30
4
4
lcsmart
2015-03-27 20:34:30
4
4
RegretDragunity
2015-03-27 20:34:30
4
4
xayy
2015-03-27 20:34:30
4
4
rt03
2015-03-27 20:34:30
4
4
meeptopia
2015-03-27 20:34:30
4
4
Aviously
2015-03-27 20:34:30
4
4
simon1221
2015-03-27 20:34:30
4
4
ImpossibleTriangle
2015-03-27 20:34:30
4
4
Dayranger
2015-03-27 20:34:30
4
4
copeland
2015-03-27 20:34:34
$S(2)=4.$ We can have $\{AA,AB,BA,BB\}.$
$S(2)=4.$ We can have $\{AA,AB,BA,BB\}.$
copeland
2015-03-27 20:34:35
What is $S(3)?$
What is $S(3)?$
swirlykick
2015-03-27 20:34:49
8
8
Aviously
2015-03-27 20:34:49
8
8
AlisonH
2015-03-27 20:34:49
8
8
ucap
2015-03-27 20:34:49
8
8
rt03
2015-03-27 20:34:49
8
8
asp7479
2015-03-27 20:34:49
8
8
copeland
2015-03-27 20:34:52
$S(3)=8.$ We can have $\{AAA,AAB,ABA,ABB,BAA,BAB,BBA,BBB\}.$
$S(3)=8.$ We can have $\{AAA,AAB,ABA,ABB,BAA,BAB,BBA,BBB\}.$
copeland
2015-03-27 20:34:53
OK, so $S(n)=2^n.$ Clearly.
OK, so $S(n)=2^n.$ Clearly.
24iam24
2015-03-27 20:35:17
no
no
danusv
2015-03-27 20:35:17
NOPE!
NOPE!
Rmehtany
2015-03-27 20:35:17
no
no
cpma213
2015-03-27 20:35:17
Nope...
Nope...
24iam24
2015-03-27 20:35:17
never
never
RegretDragunity
2015-03-27 20:35:17
Nope, not just yet
Nope, not just yet
lcsmart
2015-03-27 20:35:19
very funny
very funny
copeland
2015-03-27 20:35:20
(I recycled that joke, too.)
(I recycled that joke, too.)
copeland
2015-03-27 20:35:26
No. . . that's wrong. You can usually put $A$ at the end, unless you already have 3 $A$s to begin with.
No. . . that's wrong. You can usually put $A$ at the end, unless you already have 3 $A$s to begin with.
copeland
2015-03-27 20:35:30
There's an even deeper problem with naive recursion in that we can't just chop off the last character and talk in terms of $S(n-1)$. Is there something we can do?
There's an even deeper problem with naive recursion in that we can't just chop off the last character and talk in terms of $S(n-1)$. Is there something we can do?
nosaj
2015-03-27 20:36:22
Use recursion based on how many consecutive letters there are at the end.
Use recursion based on how many consecutive letters there are at the end.
simon1221
2015-03-27 20:36:22
blocks of length 1 2 and 3?
blocks of length 1 2 and 3?
DivideBy0
2015-03-27 20:36:22
chop off the last set of equal characters
chop off the last set of equal characters
acegikmoqsuwy2000
2015-03-27 20:36:22
use S(n-1), S(n-2), and S(n-3)
use S(n-1), S(n-2), and S(n-3)
mssmath
2015-03-27 20:36:22
Chop of last set of same integers
Chop of last set of same integers
copeland
2015-03-27 20:36:25
We could instead chop off the last block of equal characters. What do we get?
We could instead chop off the last block of equal characters. What do we get?
Rmehtany
2015-03-27 20:37:12
S(n-1)+S(n-2)+S(n-3)
S(n-1)+S(n-2)+S(n-3)
vinayak-kumar
2015-03-27 20:37:12
you will get a sequence of either n-1, n-2, n-3
you will get a sequence of either n-1, n-2, n-3
cpma213
2015-03-27 20:37:12
S(n-1), S(n-2), S(n-3), they sum to S(n)
S(n-1), S(n-2), S(n-3), they sum to S(n)
copeland
2015-03-27 20:37:15
We can get a valid string by taking any string of length $n$ and deleting as many identical characters as possible from the end of the string (1, 2, or 3).
We can get a valid string by taking any string of length $n$ and deleting as many identical characters as possible from the end of the string (1, 2, or 3).
copeland
2015-03-27 20:37:16
Therefore the strings of length $n$ ends in one of three ways:
* math=inline]$\cdots BA$[/math A single character of some parity following a string of length $n-1$ that ends in the opposite parity
* math=inline]$\cdots BAA$[/math Two characters of some parity following a string of length $n-2$ that ends in the opposite parity
* math=inline]$\cdots BAAA$[/math Three characters of some parity following a string of length $n-3$ that ends in the opposite parity
Therefore the strings of length $n$ ends in one of three ways:
* math=inline]$\cdots BA$[/math A single character of some parity following a string of length $n-1$ that ends in the opposite parity
* math=inline]$\cdots BAA$[/math Two characters of some parity following a string of length $n-2$ that ends in the opposite parity
* math=inline]$\cdots BAAA$[/math Three characters of some parity following a string of length $n-3$ that ends in the opposite parity
copeland
2015-03-27 20:37:24
What happened!
What happened!
copeland
2015-03-27 20:37:36
Therefore the strings of length $n$ ends in one of three ways:
* [ $\cdots BA$ ] A single character of some parity following a string of length $n-1$ that ends in the opposite parity
* [ $\cdots BAA$ ] Two characters of some parity following a string of length $n-2$ that ends in the opposite parity
* [ $\cdots BAAA$ ] Three characters of some parity following a string of length $n-3$ that ends in the opposite parity
Therefore the strings of length $n$ ends in one of three ways:
* [ $\cdots BA$ ] A single character of some parity following a string of length $n-1$ that ends in the opposite parity
* [ $\cdots BAA$ ] Two characters of some parity following a string of length $n-2$ that ends in the opposite parity
* [ $\cdots BAAA$ ] Three characters of some parity following a string of length $n-3$ that ends in the opposite parity
copeland
2015-03-27 20:38:01
Furthermore note that this gives a bijection between the set of strings of length $n$ and the union of the sets of strings of length $n-1,$ $n-2,$ and $n-3$ given by deleting as many identical characters as possible from the end of the string.
Furthermore note that this gives a bijection between the set of strings of length $n$ and the union of the sets of strings of length $n-1,$ $n-2,$ and $n-3$ given by deleting as many identical characters as possible from the end of the string.
copeland
2015-03-27 20:38:05
This sets up a bijection. Every string of length $n$ gives a string of length $n-1,$ $n-2,$ or $n-3$ by deleting the final characters. We get all such strings uniquely in this way. So the set counted by $S(n)$ is in correspondence with the union of the 3 sets counted by $S(n-1),$ $S(n-2),$ and $S(n-3).$
This sets up a bijection. Every string of length $n$ gives a string of length $n-1,$ $n-2,$ or $n-3$ by deleting the final characters. We get all such strings uniquely in this way. So the set counted by $S(n)$ is in correspondence with the union of the 3 sets counted by $S(n-1),$ $S(n-2),$ and $S(n-3).$
copeland
2015-03-27 20:38:19
Deleting as many identical characters as possible from the end of each of our strings of length $n$ gives a bijection between the set of strings of length $n$ and the union of the sets of strings of length $n-1,$ $n-2,$ and $n-3.$
Deleting as many identical characters as possible from the end of each of our strings of length $n$ gives a bijection between the set of strings of length $n$ and the union of the sets of strings of length $n-1,$ $n-2,$ and $n-3.$
copeland
2015-03-27 20:38:20
What recursive formula does that give us?
What recursive formula does that give us?
ImpossibleTriangle
2015-03-27 20:39:18
s(n)=s(n-1)+s(n-2)+s(n-3)
s(n)=s(n-1)+s(n-2)+s(n-3)
acegikmoqsuwy2000
2015-03-27 20:39:18
S(n)=S(n-1)+S(n-2)+S(n-3)
S(n)=S(n-1)+S(n-2)+S(n-3)
High
2015-03-27 20:39:18
S(n)=S(n-1)+S(n-2)+S(n-3)
S(n)=S(n-1)+S(n-2)+S(n-3)
cpma213
2015-03-27 20:39:18
S(n) = S(n-1)+S(n-2)+S(n-3)
S(n) = S(n-1)+S(n-2)+S(n-3)
countingarithmetic
2015-03-27 20:39:18
S(n) = S(n-1)+S(n-2)+S(n-3)
S(n) = S(n-1)+S(n-2)+S(n-3)
vinayak-kumar
2015-03-27 20:39:18
s(n)=s(n-1)+s(n-2)+s(n-3)
s(n)=s(n-1)+s(n-2)+s(n-3)
ImpossibleTriangle
2015-03-27 20:39:18
s(n)=s(n-1)+s(n-2)+s(n-3
s(n)=s(n-1)+s(n-2)+s(n-3
24iam24
2015-03-27 20:39:18
add those 3
add those 3
dhruv
2015-03-27 20:39:18
S(n)=S(n-1)+S(n-2)+S(n-3)
S(n)=S(n-1)+S(n-2)+S(n-3)
Dayranger
2015-03-27 20:39:18
S(n) = S(n - 1) + S(n - 2) + S(n - 3)
S(n) = S(n - 1) + S(n - 2) + S(n - 3)
swirlykick
2015-03-27 20:39:18
S(N)=S(N-1)+S(N-2)+S(N-3)
S(N)=S(N-1)+S(N-2)+S(N-3)
owm
2015-03-27 20:39:18
S(n)=S(n-1)+S(n-2)+S(n-3)
S(n)=S(n-1)+S(n-2)+S(n-3)
copeland
2015-03-27 20:39:23
This tells us that
This tells us that
copeland
2015-03-27 20:39:24
\[S(n)=S(n-1)+S(n-2)+S(n-3).\]
\[S(n)=S(n-1)+S(n-2)+S(n-3).\]
copeland
2015-03-27 20:39:28
Now we just start listing. Fortunately we computed the first three already, and then each number is just the sum of the previous three:
Now we just start listing. Fortunately we computed the first three already, and then each number is just the sum of the previous three:
copeland
2015-03-27 20:39:30
\[\begin{array}{c|cccccccccc}
n &1&2&3& 4&5 &6&7&8&9&10\\
\hline
S(n)&2&4&8&14&26&48&88&162&298&\boxed{548}\\
\end{array}\]
\[\begin{array}{c|cccccccccc}
n &1&2&3& 4&5 &6&7&8&9&10\\
\hline
S(n)&2&4&8&14&26&48&88&162&298&\boxed{548}\\
\end{array}\]
copeland
2015-03-27 20:39:44
Alrighty.
Alrighty.
ompatel99
2015-03-27 20:39:55
12/15 Done. 4/5 or 80%!
12/15 Done. 4/5 or 80%!
copeland
2015-03-27 20:40:56
13. Define the sequence $a_1, a_2, a_3, \ldots$ by $a_n = \sum_{k=1}^n \sin(k)$, where $k$ represents radian measure. Find the index of the 100th term for which $a_n < 0$.
13. Define the sequence $a_1, a_2, a_3, \ldots$ by $a_n = \sum_{k=1}^n \sin(k)$, where $k$ represents radian measure. Find the index of the 100th term for which $a_n < 0$.
copeland
2015-03-27 20:41:02
What methods do we have in general for evaluating long series (not just trig series)? What sorts of tools have you used in problems before?
What methods do we have in general for evaluating long series (not just trig series)? What sorts of tools have you used in problems before?
tdeng
2015-03-27 20:41:45
telescoping
telescoping
DivideBy0
2015-03-27 20:41:45
telescoping
telescoping
andrewlin
2015-03-27 20:41:45
telescoping?
telescoping?
High
2015-03-27 20:41:45
Telescoping series
Telescoping series
math-rules
2015-03-27 20:41:45
cancellation/telescoping
cancellation/telescoping
ImpossibleTriangle
2015-03-27 20:41:45
cancell
cancell
nosaj
2015-03-27 20:41:45
telescoping
telescoping
mathwizard888
2015-03-27 20:41:45
telescopign
telescopign
macandcheese
2015-03-27 20:41:45
telescoping
telescoping
thequantumguy
2015-03-27 20:41:48
from your precalculus book
from your precalculus book
copeland
2015-03-27 20:41:55
(Yeah, this problem is in the precalc book. Too bad that's not an approved tool.)
(Yeah, this problem is in the precalc book. Too bad that's not an approved tool.)
copeland
2015-03-27 20:42:11
Unfortunately, this series doesn't look geometric or arithmetic, so the simplest tools don't do much. However, telescoping is an interesting idea.
Unfortunately, this series doesn't look geometric or arithmetic, so the simplest tools don't do much. However, telescoping is an interesting idea.
copeland
2015-03-27 20:42:16
I might think of telescoping when I see big summations of trig functions, because many trig identities turn things into differences.
I might think of telescoping when I see big summations of trig functions, because many trig identities turn things into differences.
copeland
2015-03-27 20:42:28
Are there any trig identities we might use to turn each of our terms into $\text{something} - \text{something else}$?
Are there any trig identities we might use to turn each of our terms into $\text{something} - \text{something else}$?
nosaj
2015-03-27 20:43:16
What is up with problem 13 on both AIMEs being from the precalc book?
What is up with problem 13 on both AIMEs being from the precalc book?
copeland
2015-03-27 20:43:18
I don't know. You can find the precalc book in the online bookstore here after the Math Jam:
http://www.artofproblemsolving.com/store/item/precalculus
I don't know. You can find the precalc book in the online bookstore here after the Math Jam:
http://www.artofproblemsolving.com/store/item/precalculus
ryanyz10
2015-03-27 20:43:54
pythag identities?
pythag identities?
copeland
2015-03-27 20:44:00
Which one?
Which one?
nosaj
2015-03-27 20:44:19
sum of angles forumla?
sum of angles forumla?
copeland
2015-03-27 20:44:31
Sum of angles is good but it gives a quadratic stuff.
Sum of angles is good but it gives a quadratic stuff.
copeland
2015-03-27 20:44:48
We want to multiply by something, say, in order to get degree 1 things.
We want to multiply by something, say, in order to get degree 1 things.
Dayranger
2015-03-27 20:44:56
sin 2k = 2 sin k cos k
sin 2k = 2 sin k cos k
copeland
2015-03-27 20:44:59
Yeah, like that one!
Yeah, like that one!
copeland
2015-03-27 20:45:06
Except there's no sum.
Except there's no sum.
copeland
2015-03-27 20:45:19
But I multiply a sine by a cosine and I get a single sine out.
But I multiply a sine by a cosine and I get a single sine out.
tdeng
2015-03-27 20:45:40
product to sum?
product to sum?
copeland
2015-03-27 20:45:43
Yeah, that's great.
Yeah, that's great.
copeland
2015-03-27 20:45:51
The product to sum formula for sine has this form: \[\sin(x)\sin(y) = \frac12 \left(\cos(x-y) - \cos(x+y)\right).\]
The product to sum formula for sine has this form: \[\sin(x)\sin(y) = \frac12 \left(\cos(x-y) - \cos(x+y)\right).\]
copeland
2015-03-27 20:45:55
How can we make our sum look more like this?
How can we make our sum look more like this?
copeland
2015-03-27 20:47:05
I have a sum of sines.
I have a sum of sines.
copeland
2015-03-27 20:47:16
I somehow want to turn each sine into a pair of things.
I somehow want to turn each sine into a pair of things.
copeland
2015-03-27 20:47:23
(In order to telescope)
(In order to telescope)
q12
2015-03-27 20:47:36
multiply everything by sin(1)
multiply everything by sin(1)
copeland
2015-03-27 20:47:43
We sure want to multiply by sin(something).
We sure want to multiply by sin(something).
copeland
2015-03-27 20:47:54
Let's just leave it as a variable. Maybe that will help us figure out what the something is.
Let's just leave it as a variable. Maybe that will help us figure out what the something is.
copeland
2015-03-27 20:48:00
For example, we can multiply by 1 in the form of $\dfrac{\sin(y)}{\sin(y)}$ for some $y$. Then our sum becomes \[ a_n = \frac{1}{\sin(y)}\sum_{k=1}^n \sin(k)\sin(y).\]
For example, we can multiply by 1 in the form of $\dfrac{\sin(y)}{\sin(y)}$ for some $y$. Then our sum becomes \[ a_n = \frac{1}{\sin(y)}\sum_{k=1}^n \sin(k)\sin(y).\]
copeland
2015-03-27 20:48:11
Are there any choices for $y$ that would make this series telescope?
Are there any choices for $y$ that would make this series telescope?
acegikmoqsuwy2000
2015-03-27 20:49:04
multiply by sin(pi/2)
multiply by sin(pi/2)
math-rules
2015-03-27 20:49:04
1/2
1/2
thequantumguy
2015-03-27 20:49:04
1/2
1/2
ucap
2015-03-27 20:49:04
1/2
1/2
copeland
2015-03-27 20:49:07
This sum evaluates cosine at $k-y,k+y$ then $k+1-y,k+1+y$ then $k+2-y,k+2+y$ then $k+3-y,k+3+y$, etc.
This sum evaluates cosine at $k-y,k+y$ then $k+1-y,k+1+y$ then $k+2-y,k+2+y$ then $k+3-y,k+3+y$, etc.
copeland
2015-03-27 20:49:19
In order for it to telescope we ought to try to arrange that neighboring terms are equal, so $k+y=k+1-y$ and $k+1+y=k+2-y$, etc.
In order for it to telescope we ought to try to arrange that neighboring terms are equal, so $k+y=k+1-y$ and $k+1+y=k+2-y$, etc.
copeland
2015-03-27 20:49:28
These all simplify to $y=\dfrac12$.
These all simplify to $y=\dfrac12$.
copeland
2015-03-27 20:49:42
When we plut in $y=\dfrac12$, all the intermediate terms cancel and we're left with: \[ a_n = \frac{1}{2\sin(\frac12)}\left(\cos\left(\frac12\right) - \cos\left(n + \frac 12\right)\right). \]
When we plut in $y=\dfrac12$, all the intermediate terms cancel and we're left with: \[ a_n = \frac{1}{2\sin(\frac12)}\left(\cos\left(\frac12\right) - \cos\left(n + \frac 12\right)\right). \]
SockFoot
2015-03-27 20:50:00
great
great
copeland
2015-03-27 20:50:01
Indeed.
Indeed.
copeland
2015-03-27 20:50:14
Now we see that $a_n < 0$ is equivalent to $\cos\left(\frac12\right) <\cos\left(n + \frac12\right)$.
Now we see that $a_n < 0$ is equivalent to $\cos\left(\frac12\right) <\cos\left(n + \frac12\right)$.
copeland
2015-03-27 20:50:24
Let's draw the region on the unit circle where $\cos(\theta)$ is greater than $\cos\left(\frac 12 \right)$ to get an idea of what's going on.
Let's draw the region on the unit circle where $\cos(\theta)$ is greater than $\cos\left(\frac 12 \right)$ to get an idea of what's going on.
copeland
2015-03-27 20:50:25
copeland
2015-03-27 20:50:30
What do we know about $n$ if $n + \frac 12$ is in this region?
What do we know about $n$ if $n + \frac 12$ is in this region?
copeland
2015-03-27 20:51:47
What is this region anyway?
What is this region anyway?
thequantumguy
2015-03-27 20:52:19
$n \in [2k\pi - 1, 2k\pi]$
$n \in [2k\pi - 1, 2k\pi]$
simon1221
2015-03-27 20:52:19
a triangle!
a triangle!
ompatel99
2015-03-27 20:52:19
A triangle
A triangle
nosaj
2015-03-27 20:52:19
a isosceles triangle!
a isosceles triangle!
copeland
2015-03-27 20:52:33
I drew a triangle, yes.
I drew a triangle, yes.
copeland
2015-03-27 20:52:52
But the region we are talking about is the angle.
But the region we are talking about is the angle.
copeland
2015-03-27 20:53:00
What angles are in that region?
What angles are in that region?
cpma213
2015-03-27 20:53:41
A sector of the unit circle capped at 1/2 radians above and below
A sector of the unit circle capped at 1/2 radians above and below
swirlykick
2015-03-27 20:53:41
1 radian?
1 radian?
copeland
2015-03-27 20:53:58
Notice that we're not really using $\pi$ at all. 1 radian is a truly weird number.
Notice that we're not really using $\pi$ at all. 1 radian is a truly weird number.
copeland
2015-03-27 20:54:11
This is the angle from -1/2 to 1/2.
This is the angle from -1/2 to 1/2.
copeland
2015-03-27 20:54:26
.3 is in there, -.22 is in there, etc.
.3 is in there, -.22 is in there, etc.
copeland
2015-03-27 20:54:36
What other numbers fall in there?
What other numbers fall in there?
Dayranger
2015-03-27 20:54:54
-.4
-.4
Dayranger
2015-03-27 20:54:54
.4
.4
Dayranger
2015-03-27 20:54:54
0
0
math_cool
2015-03-27 20:54:54
0
0
Tommy2000
2015-03-27 20:54:54
0
0
gxah
2015-03-27 20:54:54
(-1/2, 1/2)
(-1/2, 1/2)
ShadowQueenPeach
2015-03-27 20:54:54
anything from -1/2 to 1/2
anything from -1/2 to 1/2
copeland
2015-03-27 20:54:57
OK, great.
OK, great.
copeland
2015-03-27 20:55:05
Are those the only numbers that fall in there?
Are those the only numbers that fall in there?
copeland
2015-03-27 20:55:13
What about 6?
What about 6?
copeland
2015-03-27 20:55:19
Is 6 in there?
Is 6 in there?
saagar
2015-03-27 20:55:54
?YES
?YES
andrewlin
2015-03-27 20:55:54
yep!
yep!
Rocksolid
2015-03-27 20:55:54
yes
yes
DivideBy0
2015-03-27 20:55:54
well, mod 2pi
well, mod 2pi
ryanyz10
2015-03-27 20:55:54
oh wait yes
oh wait yes
uberminecraft722
2015-03-27 20:55:54
yes
yes
DivideBy0
2015-03-27 20:55:54
yeah; look at everything mod 2pi
yeah; look at everything mod 2pi
ompatel99
2015-03-27 20:55:54
Yes because it will come around twice then be in that region
Yes because it will come around twice then be in that region
tdeng
2015-03-27 20:55:54
yes
yes
High
2015-03-27 20:55:54
yes
yes
Rmink41
2015-03-27 20:55:54
YEAH
YEAH
SmartYuRuo
2015-03-27 20:55:54
yes
yes
xayy
2015-03-27 20:55:54
yes?
yes?
copeland
2015-03-27 20:55:56
6 is between $2\pi-1/2$ and $2\pi+1/2$.
6 is between $2\pi-1/2$ and $2\pi+1/2$.
copeland
2015-03-27 20:55:59
It is in there.
It is in there.
copeland
2015-03-27 20:56:06
What are all the real numbers that land in there?
What are all the real numbers that land in there?
Mlux
2015-03-27 20:57:46
you mean intergral
you mean intergral
nosaj
2015-03-27 20:57:46
must be between 2pi-1/2 and 2pi+1/2
must be between 2pi-1/2 and 2pi+1/2
ompatel99
2015-03-27 20:57:46
2pi*n-0.5 to 2pi*n+0.5 for integer n
2pi*n-0.5 to 2pi*n+0.5 for integer n
DivideBy0
2015-03-27 20:57:46
2pik - 1/2 to 2pik +1/2 for all integers k
2pik - 1/2 to 2pik +1/2 for all integers k
cpma213
2015-03-27 20:57:46
In General, between 2\pi x-1/2 and 2\pi x+1/2
In General, between 2\pi x-1/2 and 2\pi x+1/2
24iam24
2015-03-27 20:57:46
from 2kpi-1/2 to 2kpi+1/2 where k is any integer
from 2kpi-1/2 to 2kpi+1/2 where k is any integer
ImpossibleTriangle
2015-03-27 20:57:46
(2kpi -1/2, 2kpi + 1/2)
(2kpi -1/2, 2kpi + 1/2)
uberminecraft722
2015-03-27 20:57:46
[2(pi)k-(1/2),2(pi)k+(1/2)]
[2(pi)k-(1/2),2(pi)k+(1/2)]
copeland
2015-03-27 20:58:30
Right, exactly. This is the union of all regions of the form $(2\pi-1/2,2\pi+1/2)$.
Right, exactly. This is the union of all regions of the form $(2\pi-1/2,2\pi+1/2)$.
copeland
2015-03-27 20:58:31
What do we know about $n$ if $n + \frac 12$ is in this region?
What do we know about $n$ if $n + \frac 12$ is in this region?
thequantumguy
2015-03-27 20:58:49
n is in [2kpi - 1, 2kpi]
n is in [2kpi - 1, 2kpi]
Rocksolid
2015-03-27 20:58:49
n is between 2pi and 2pi -1
n is between 2pi and 2pi -1
ryanyz10
2015-03-27 20:58:49
in (2pi - 1, 2pi)?
in (2pi - 1, 2pi)?
copeland
2015-03-27 20:59:17
Right. And I have a really special name for "the integer that is just less than a number."
Right. And I have a really special name for "the integer that is just less than a number."
Rmehtany
2015-03-27 20:59:35
floor
floor
swirlykick
2015-03-27 20:59:35
Floor?
Floor?
dhruv
2015-03-27 20:59:35
floor
floor
vinayak-kumar
2015-03-27 20:59:35
floor function
floor function
thequantumguy
2015-03-27 20:59:35
floor
floor
Rocksolid
2015-03-27 20:59:35
floor
floor
copeland
2015-03-27 20:59:40
We see $n+ \frac 12$ must be between $2\pi m - \frac 12$ and $2\pi m + \frac 12$ for some positive integer $m$.
We see $n+ \frac 12$ must be between $2\pi m - \frac 12$ and $2\pi m + \frac 12$ for some positive integer $m$.
copeland
2015-03-27 20:59:42
Isolating $n$ that says $2\pi m - 1 < n < 2\pi m$ for some $m$, or $n = \lfloor 2\pi m \rfloor$.
Isolating $n$ that says $2\pi m - 1 < n < 2\pi m$ for some $m$, or $n = \lfloor 2\pi m \rfloor$.
ShadowQueenPeach
2015-03-27 20:59:50
the copeland?
the copeland?
copeland
2015-03-27 20:59:55
This is Palmer's vote.
This is Palmer's vote.
copeland
2015-03-27 21:00:09
What is the index of the 11th term?
What is the index of the 11th term?
copeland
2015-03-27 21:00:31
gosh.
gosh.
copeland
2015-03-27 21:00:37
What is the index of the 100th term?
What is the index of the 100th term?
thequantumguy
2015-03-27 21:01:11
which is 628
which is 628
swirlykick
2015-03-27 21:01:11
628
628
ompatel99
2015-03-27 21:01:11
floor(200pi)=628
floor(200pi)=628
saagar
2015-03-27 21:01:11
628?
628?
thequantumguy
2015-03-27 21:01:11
floor of 200 pi
floor of 200 pi
uberminecraft722
2015-03-27 21:01:11
628
628
thequantumguy
2015-03-27 21:01:11
which is 628
which is 628
raptorw
2015-03-27 21:01:11
628
628
andrewlin
2015-03-27 21:01:11
628
628
czhu000
2015-03-27 21:01:11
628
628
copeland
2015-03-27 21:01:22
So the index of the 100th term less than 0 is just $\lfloor 2\pi \cdot 100 \rfloor$ or $\boxed{628}$.
So the index of the 100th term less than 0 is just $\lfloor 2\pi \cdot 100 \rfloor$ or $\boxed{628}$.
copeland
2015-03-27 21:01:40
Incidentally, my original solution to this problem used complex numbers to write the series as the imaginary part of a geometric series. It looked a bit like this:
\begin{align*}
a_n
&=\sum_{k=1}^n\sin(k)\\
&=\operatorname{Im}\left(\sum_{k=1}^ne^{ik}\right)\\
&=\operatorname{Im}\left(\frac{e^{i(n+1)}-e^i}{e^{i}-1}\right)\\
&=\operatorname{Im}\left(\frac{e^{i(n+1/2)}-e^{i/2}}{e^{i/2}-e^{-i/2}}\right)\\
&=\operatorname{Im}\left(\frac{e^{i(n+1/2)}-e^{i/2}}{2i\sin(1/2)}\right)\\
&=\frac{-1}{2\sin(1/2)}\operatorname{Re}\left(e^{i(n+1/2)}-e^{i/2}\right)\\
&=\frac{\cos(n+\frac12)-\cos(\frac12)}{2\sin(\frac12)}\\
\end{align*}
Incidentally, my original solution to this problem used complex numbers to write the series as the imaginary part of a geometric series. It looked a bit like this:
\begin{align*}
a_n
&=\sum_{k=1}^n\sin(k)\\
&=\operatorname{Im}\left(\sum_{k=1}^ne^{ik}\right)\\
&=\operatorname{Im}\left(\frac{e^{i(n+1)}-e^i}{e^{i}-1}\right)\\
&=\operatorname{Im}\left(\frac{e^{i(n+1/2)}-e^{i/2}}{e^{i/2}-e^{-i/2}}\right)\\
&=\operatorname{Im}\left(\frac{e^{i(n+1/2)}-e^{i/2}}{2i\sin(1/2)}\right)\\
&=\frac{-1}{2\sin(1/2)}\operatorname{Re}\left(e^{i(n+1/2)}-e^{i/2}\right)\\
&=\frac{\cos(n+\frac12)-\cos(\frac12)}{2\sin(\frac12)}\\
\end{align*}
acegikmoqsuwy2000
2015-03-27 21:02:09
that's what I was going to do too
that's what I was going to do too
tdeng
2015-03-27 21:02:09
i think the way we did was easier
i think the way we did was easier
copeland
2015-03-27 21:02:21
To each his own.
To each his own.
copeland
2015-03-27 21:02:32
14. Let $x$ and $y$ be real numbers satisfying $x^4y^5+y^4x^5=810$ and $x^3y^6+y^3x^6=945$. Evaluate \[2x^3+(xy)^3+2y^3.\]
14. Let $x$ and $y$ be real numbers satisfying $x^4y^5+y^4x^5=810$ and $x^3y^6+y^3x^6=945$. Evaluate \[2x^3+(xy)^3+2y^3.\]
copeland
2015-03-27 21:02:42
What basic fact about polynomials pops into your head immediately when you see this problem?
What basic fact about polynomials pops into your head immediately when you see this problem?
ompatel99
2015-03-27 21:03:03
Symmetry?
Symmetry?
copeland
2015-03-27 21:03:04
What does symmetry tell us?
What does symmetry tell us?
nosaj
2015-03-27 21:03:26
roots are interchangable?
roots are interchangable?
DivideBy0
2015-03-27 21:03:26
replace x and y and get the same result
replace x and y and get the same result
copeland
2015-03-27 21:03:39
That is the definition. What do we know about symmetric polynomials?
That is the definition. What do we know about symmetric polynomials?
vinayak-kumar
2015-03-27 21:03:57
sum and product
sum and product
copeland
2015-03-27 21:04:07
We KNOW that any symmetric polynomial in $x$ and $y$ can be expressed in terms of $s=x+y$ and $p=xy$.
We KNOW that any symmetric polynomial in $x$ and $y$ can be expressed in terms of $s=x+y$ and $p=xy$.
copeland
2015-03-27 21:04:12
Rewriting all three expressions in terms of $s$ and $p$ is a fabulous first step and will take you to the answer with some work.
Rewriting all three expressions in terms of $s$ and $p$ is a fabulous first step and will take you to the answer with some work.
copeland
2015-03-27 21:04:15
This is Problem 14, though, so maybe there's something useful we can do first.
This is Problem 14, though, so maybe there's something useful we can do first.
copeland
2015-03-27 21:04:26
Can we combine the two given equations in a nice way? They have the same degree after all.
Can we combine the two given equations in a nice way? They have the same degree after all.
Tommy2000
2015-03-27 21:05:37
divide?
divide?
24iam24
2015-03-27 21:05:37
divide them?
divide them?
Rmink41
2015-03-27 21:05:37
Divide
Divide
DivideBy0
2015-03-27 21:05:37
divide?
divide?
copeland
2015-03-27 21:05:39
Dividing is neat. A lot of stuff will cancel. Any other things we could do?
Dividing is neat. A lot of stuff will cancel. Any other things we could do?
MathStudent2002
2015-03-27 21:06:27
$x^3y^3(x+y)^3=810\cdot 3+945$
$x^3y^3(x+y)^3=810\cdot 3+945$
nosaj
2015-03-27 21:06:27
Multiply the first equation by 3, then add.
Multiply the first equation by 3, then add.
mathwrath
2015-03-27 21:06:27
Add 3 times the first to the second and factor
Add 3 times the first to the second and factor
countingarithmetic
2015-03-27 21:06:27
945+3*810 = x^3*y^3*(x+y)^3
945+3*810 = x^3*y^3*(x+y)^3
copeland
2015-03-27 21:06:34
If we triple the first and add it to the second we get something nice:\[3(x^4y^5+y^4x^5)+(x^3y^6+y^3x^6)=3(810)+945.\]
My reflex here is to factor the integers a little on the right and expand the left side:\[x^3y^6+3x^4y^5+3y^4x^5+y^3x^6=(3\cdot6+7)\cdot135.\]
If we triple the first and add it to the second we get something nice:\[3(x^4y^5+y^4x^5)+(x^3y^6+y^3x^6)=3(810)+945.\]
My reflex here is to factor the integers a little on the right and expand the left side:\[x^3y^6+3x^4y^5+3y^4x^5+y^3x^6=(3\cdot6+7)\cdot135.\]
copeland
2015-03-27 21:06:44
How does the left side simplify?
How does the left side simplify?
mathwrath
2015-03-27 21:07:27
x^3y^3(x+y)^3
x^3y^3(x+y)^3
acegikmoqsuwy2000
2015-03-27 21:07:27
$[xy(x+y)]^3$
$[xy(x+y)]^3$
DivideBy0
2015-03-27 21:07:27
nosaj
2015-03-27 21:07:27
$x^3y^3(x+y)^3$
$x^3y^3(x+y)^3$
copeland
2015-03-27 21:07:28
Yeah, I guess I passed that already.
Yeah, I guess I passed that already.
copeland
2015-03-27 21:07:29
The left side simplifies to \[x^3y^3(y^3+3xy^2+3x^2y+x^3)=x^3y^3(x+y)^3\]
The left side simplifies to \[x^3y^3(y^3+3xy^2+3x^2y+x^3)=x^3y^3(x+y)^3\]
copeland
2015-03-27 21:07:31
Putting it all together gives \[p^3s^3=3^3\cdot5^3.\]
Putting it all together gives \[p^3s^3=3^3\cdot5^3.\]
copeland
2015-03-27 21:07:32
And?
And?
countingarithmetic
2015-03-27 21:08:05
15= x*y*(x+y)
15= x*y*(x+y)
andrewlin
2015-03-27 21:08:05
ps=15
ps=15
24iam24
2015-03-27 21:08:05
ps=15
ps=15
Dayranger
2015-03-27 21:08:05
ps = 15
ps = 15
copeland
2015-03-27 21:08:09
Since $x$ and $y$ are real numbers, the cube roots are well-defined.
Since $x$ and $y$ are real numbers, the cube roots are well-defined.
copeland
2015-03-27 21:08:10
We get $ps=15$.
We get $ps=15$.
copeland
2015-03-27 21:08:17
Now what should we do?
Now what should we do?
DivideBy0
2015-03-27 21:09:34
factor the first equation and plug in p and s
factor the first equation and plug in p and s
andrewlin
2015-03-27 21:09:34
we have p^4*s=810 so we can solve for p and s
we have p^4*s=810 so we can solve for p and s
copeland
2015-03-27 21:09:38
Let's go back to the first equation,\[x^4y^5+x^5y^4=810.\]
Let's go back to the first equation,\[x^4y^5+x^5y^4=810.\]
copeland
2015-03-27 21:09:43
This equation is divisible by $ps$. Specifically we have \[p^4s=810.\] So. . .
This equation is divisible by $ps$. Specifically we have \[p^4s=810.\] So. . .
ompatel99
2015-03-27 21:10:25
p^3=54
p^3=54
BPM14
2015-03-27 21:10:25
p^3 = 54
p^3 = 54
gxah
2015-03-27 21:10:25
p^3 = 54
p^3 = 54
henryweng
2015-03-27 21:10:25
p3=54
p3=54
acegikmoqsuwy2000
2015-03-27 21:10:25
p^3 = 54
p^3 = 54
copeland
2015-03-27 21:10:28
Dividing by $ps=15$ gives us $p^3=54$.
Dividing by $ps=15$ gives us $p^3=54$.
copeland
2015-03-27 21:10:32
Now what?
Now what?
nosaj
2015-03-27 21:11:04
so that means we can find $p$ and $s$!
so that means we can find $p$ and $s$!
math_cool
2015-03-27 21:11:04
Solve for s
Solve for s
thequantumguy
2015-03-27 21:11:04
solve for p to find s
solve for p to find s
guilt
2015-03-27 21:11:04
We can solve for p and then subsequently q
We can solve for p and then subsequently q
Ramanan369
2015-03-27 21:11:04
solve for s
solve for s
copeland
2015-03-27 21:11:09
Sure that also tells us $p$ and $s$ but hang on for a second with that - we might not need them.
Sure that also tells us $p$ and $s$ but hang on for a second with that - we might not need them.
copeland
2015-03-27 21:11:27
What can we do with our newly discovered $p^3$?
What can we do with our newly discovered $p^3$?
24iam24
2015-03-27 21:11:50
we just got the middle part of what we wanted
we just got the middle part of what we wanted
mssmath
2015-03-27 21:11:50
(xy)^3 is done
(xy)^3 is done
Dayranger
2015-03-27 21:11:50
(xy)^3 = 54
(xy)^3 = 54
RJ810
2015-03-27 21:11:50
put it into the final question
put it into the final question
saagar
2015-03-27 21:11:54
(xy)^3=54
(xy)^3=54
copeland
2015-03-27 21:12:07
That's great. We have the middle part of what we want now.
That's great. We have the middle part of what we want now.
copeland
2015-03-27 21:12:09
What are we missing?
What are we missing?
saagar
2015-03-27 21:12:31
2(x^3+y^3)
2(x^3+y^3)
ryanyz10
2015-03-27 21:12:31
2(x^3 + y^3)
2(x^3 + y^3)
RJ810
2015-03-27 21:12:31
x^3+y^3
x^3+y^3
math_cool
2015-03-27 21:12:31
2(x^3 + y^3)
2(x^3 + y^3)
High
2015-03-27 21:12:31
2(x^3+y^3)
2(x^3+y^3)
copeland
2015-03-27 21:12:38
Now we just need to find $2(x^3+y^3)$. How can we find that?
Now we just need to find $2(x^3+y^3)$. How can we find that?
andrewlin
2015-03-27 21:13:06
we can use p^3 in the second equation to get x^3+y^3=35/2
we can use p^3 in the second equation to get x^3+y^3=35/2
swirlykick
2015-03-27 21:13:06
second equation
second equation
czhu000
2015-03-27 21:13:06
Factor $p^3$ out of the second equation to get the remaining part
Factor $p^3$ out of the second equation to get the remaining part
guilt
2015-03-27 21:13:06
divide the 2nd equation by p^3
divide the 2nd equation by p^3
czhu000
2015-03-27 21:13:06
Factor $p^3$ out of the second equation and multiply by 2
Factor $p^3$ out of the second equation and multiply by 2
Rmink41
2015-03-27 21:13:06
Using eq 2
Using eq 2
czhu000
2015-03-27 21:13:06
Divide $p^3$ out of the second equation and multiply by 2
Divide $p^3$ out of the second equation and multiply by 2
copeland
2015-03-27 21:13:13
The second equation is divisible by $p^3$. It is \[p^3(x^3+y^3)=945.\]
The second equation is divisible by $p^3$. It is \[p^3(x^3+y^3)=945.\]
copeland
2015-03-27 21:13:16
Now we know \[x^3+y^3=\frac{945}{54}=\frac{9\cdot105}{9\cdot6}=\frac{105}6=\frac{35}2.\]
Now we know \[x^3+y^3=\frac{945}{54}=\frac{9\cdot105}{9\cdot6}=\frac{105}6=\frac{35}2.\]
copeland
2015-03-27 21:13:23
What do we do now?
What do we do now?
nosaj
2015-03-27 21:13:57
add
add
Rmink41
2015-03-27 21:13:57
Multiply by 2 and add the middle
Multiply by 2 and add the middle
dhruv
2015-03-27 21:13:57
35+54=89
35+54=89
bluephoenix
2015-03-27 21:13:57
multiply by 2 to get 35
multiply by 2 to get 35
24iam24
2015-03-27 21:13:57
35+54=89
35+54=89
caressezhu
2015-03-27 21:13:57
89
89
RJ810
2015-03-27 21:13:57
mult by 2 and add by 54
mult by 2 and add by 54
math_cool
2015-03-27 21:13:57
35 + 54 = 089
35 + 54 = 089
Dayranger
2015-03-27 21:13:57
35 + 54 = 89
35 + 54 = 89
xayy
2015-03-27 21:13:57
multiply it by 2
multiply it by 2
BPM14
2015-03-27 21:13:57
35+54 = 89
35+54 = 89
RJ810
2015-03-27 21:13:57
answer is 89
answer is 89
Mlux
2015-03-27 21:13:57
35 + 54
35 + 54
copeland
2015-03-27 21:13:59
OH, hey, we're done!
OH, hey, we're done!
copeland
2015-03-27 21:14:01
We want \[2x^3+(xy)^3+2y^3=2(x^3+y^2)+p^3=35+54=\boxed{89}.\]
We want \[2x^3+(xy)^3+2y^3=2(x^3+y^2)+p^3=35+54=\boxed{89}.\]
copeland
2015-03-27 21:14:04
Awesome.
Awesome.
copeland
2015-03-27 21:14:10
Alright. Ready to go home?
Alright. Ready to go home?
Mlux
2015-03-27 21:14:24
14/15
14/15
tdeng
2015-03-27 21:14:24
14/15 of the way home
14/15 of the way home
acegikmoqsuwy2000
2015-03-27 21:14:24
Nope 1 more!! only 14/15 done
Nope 1 more!! only 14/15 done
firemike
2015-03-27 21:14:26
no!
no!
copeland
2015-03-27 21:14:34
OK, let's do 15 then.
OK, let's do 15 then.
copeland
2015-03-27 21:14:38
15. Circles $\mathcal{P}$ and $\mathcal{Q}$ have radii 1 and 4, respectively, and are externally tangent at point $A$. Point $B$ is on $\mathcal{P}$ and point $C$ is on $\mathcal{Q}$ so that line $BC$ is a common external tangent of the two circles. A line $\ell$ through $A$ intersects $\mathcal{P}$ again at $D$ and intersects $\mathcal{Q}$ again at $E$. Points $B$ and $C$ lie on the same side of $\ell$, and the areas of $\triangle DBA$ and $\triangle ACE$ are equal. This common area is $\frac mn$, where $m$ and $n$ are relatively prime positive integers. Find $m+n$.
15. Circles $\mathcal{P}$ and $\mathcal{Q}$ have radii 1 and 4, respectively, and are externally tangent at point $A$. Point $B$ is on $\mathcal{P}$ and point $C$ is on $\mathcal{Q}$ so that line $BC$ is a common external tangent of the two circles. A line $\ell$ through $A$ intersects $\mathcal{P}$ again at $D$ and intersects $\mathcal{Q}$ again at $E$. Points $B$ and $C$ lie on the same side of $\ell$, and the areas of $\triangle DBA$ and $\triangle ACE$ are equal. This common area is $\frac mn$, where $m$ and $n$ are relatively prime positive integers. Find $m+n$.
copeland
2015-03-27 21:14:40
copeland
2015-03-27 21:14:58
Cool diagram!
Cool diagram!
copeland
2015-03-27 21:14:59
I like this diagram since it has a lot of different pieces that you can focus on and start playing with. Let's see if we can get to the right start for this problem.
I like this diagram since it has a lot of different pieces that you can focus on and start playing with. Let's see if we can get to the right start for this problem.
copeland
2015-03-27 21:15:05
The line $\ell$ is the variable here. If you ignore it, all that is left in the configuration are two circles and a line, all of which are mutually tangent. If you've seen a lot of Olympiad-level geometry, what might you be inclined to do with such a diagram?
The line $\ell$ is the variable here. If you ignore it, all that is left in the configuration are two circles and a line, all of which are mutually tangent. If you've seen a lot of Olympiad-level geometry, what might you be inclined to do with such a diagram?
copeland
2015-03-27 21:16:02
Look again at $\ell$. As you vary it, it creates a lot of paired objects. It creates points $D$ and $E$, it creates lines $DA$ and $AE$, and it creates arcs $DA$ and $AE$.
Look again at $\ell$. As you vary it, it creates a lot of paired objects. It creates points $D$ and $E$, it creates lines $DA$ and $AE$, and it creates arcs $DA$ and $AE$.
copeland
2015-03-27 21:16:21
Notice anything interesting along those lines?
Notice anything interesting along those lines?
saagar
2015-03-27 21:17:03
AE=4DA
AE=4DA
acegikmoqsuwy2000
2015-03-27 21:17:03
measure of the minor arcs DA and AE are the same?
measure of the minor arcs DA and AE are the same?
elf123
2015-03-27 21:17:03
same arc length
same arc length
copeland
2015-03-27 21:17:07
Hey, that's crazy.
Hey, that's crazy.
copeland
2015-03-27 21:17:08
You might just know that those two arcs are the same angle measure. That comes from comparing those angles to the angles with the tangent line:
You might just know that those two arcs are the same angle measure. That comes from comparing those angles to the angles with the tangent line:
copeland
2015-03-27 21:17:13
copeland
2015-03-27 21:17:20
The arcs have equal inscribed angles so the arcs are equal angles. (The ratio of the length is then $1:4$.)
The arcs have equal inscribed angles so the arcs are equal angles. (The ratio of the length is then $1:4$.)
copeland
2015-03-27 21:17:22
How else could we compare those two arcs?
How else could we compare those two arcs?
copeland
2015-03-27 21:18:57
So here's some more intuition: I have all kinds of paired objects all over the place.
So here's some more intuition: I have all kinds of paired objects all over the place.
copeland
2015-03-27 21:19:03
Paired points, paired arcs, etc.
Paired points, paired arcs, etc.
copeland
2015-03-27 21:19:24
Is there some nice transformation that let's us discuss them together?
Is there some nice transformation that let's us discuss them together?
saagar
2015-03-27 21:20:00
dilation?
dilation?
24iam24
2015-03-27 21:20:00
dilation?
dilation?
thequantumguy
2015-03-27 21:20:00
rotatiom
rotatiom
tdeng
2015-03-27 21:20:00
rotation
rotation
math-rules
2015-03-27 21:20:00
dilation?
dilation?
copeland
2015-03-27 21:20:05
Neither one is quite enough.
Neither one is quite enough.
Rmehtany
2015-03-27 21:20:37
together
together
SockFoot
2015-03-27 21:20:37
homothety
homothety
math-rules
2015-03-27 21:20:37
homothety?
homothety?
chezbgone
2015-03-27 21:20:37
spiral similarity?
spiral similarity?
bluephoenix
2015-03-27 21:20:37
homotheicy?
homotheicy?
andrewlin
2015-03-27 21:20:37
BOTH?
BOTH?
Rmehtany
2015-03-27 21:20:37
dilate and rotate
dilate and rotate
copeland
2015-03-27 21:20:46
We could either perform a homothety through $A$ or a rotation and dilation about $A$. (In fact, those are the same operation.)
We could either perform a homothety through $A$ or a rotation and dilation about $A$. (In fact, those are the same operation.)
copeland
2015-03-27 21:20:48
If homothety is new to you, it means we take every point, stretch it by some factor, and in this case also reflect it through $A$. Just keep saying 'rotation and stretch'.
If homothety is new to you, it means we take every point, stretch it by some factor, and in this case also reflect it through $A$. Just keep saying 'rotation and stretch'.
copeland
2015-03-27 21:21:29
In general, the nice rule of thumb is that if you have tangent circles and lines and whatnot, try to find a transformation that interchanges some of the elements.
In general, the nice rule of thumb is that if you have tangent circles and lines and whatnot, try to find a transformation that interchanges some of the elements.
copeland
2015-03-27 21:21:40
So really, this diagram kinda wants us to perform one of these operations because even $\ell$ finds them to be interesting - rotation by $\pi$ fixes $\ell$ as well.
So really, this diagram kinda wants us to perform one of these operations because even $\ell$ finds them to be interesting - rotation by $\pi$ fixes $\ell$ as well.
copeland
2015-03-27 21:21:42
Which is better? We have options:
$\bullet$ Homothety by $-4$ taking the little circle to the big circle
$\bullet$ Homothety by $-\dfrac14$ taking the big circle to the little circle
Which is better? We have options:
$\bullet$ Homothety by $-4$ taking the little circle to the big circle
$\bullet$ Homothety by $-\dfrac14$ taking the big circle to the little circle
copeland
2015-03-27 21:21:50
Which of these two operations is best?
Which of these two operations is best?
Rmehtany
2015-03-27 21:22:05
little to big
little to big
High
2015-03-27 21:22:05
little to big
little to big
SKundu13
2015-03-27 21:22:05
Homothety by -4
Homothety by -4
firemike
2015-03-27 21:22:05
small to big
small to big
24iam24
2015-03-27 21:22:05
first no fractions
first no fractions
saagar
2015-03-27 21:22:05
make the smaller one bigger
make the smaller one bigger
ryanyz10
2015-03-27 21:22:05
first one... less messy
first one... less messy
copeland
2015-03-27 21:22:08
We ought to look at the other elements of the diagram to answer this question.
We ought to look at the other elements of the diagram to answer this question.
copeland
2015-03-27 21:22:10
Both take one of the circles to the other. (Little to big or big to little.)
Both turn the remaining circle into nonsense. (Big to really big or little to really little.)
Both take $\ell$ and give back $\ell$.
Both take one of the arcs to the other arc.
Both take one of the circles to the other. (Little to big or big to little.)
Both turn the remaining circle into nonsense. (Big to really big or little to really little.)
Both take $\ell$ and give back $\ell$.
Both take one of the arcs to the other arc.
copeland
2015-03-27 21:22:23
Both operations are kind to the tangent line. The big homothety does this:
Both operations are kind to the tangent line. The big homothety does this:
copeland
2015-03-27 21:22:30
copeland
2015-03-27 21:22:36
copeland
2015-03-27 21:22:39
The little homothety does this:
The little homothety does this:
copeland
2015-03-27 21:22:41
copeland
2015-03-27 21:22:45
Those give us effectively the same diagram, but the first is clearly nicer since it is easier to read.
Those give us effectively the same diagram, but the first is clearly nicer since it is easier to read.
copeland
2015-03-27 21:22:45
Why does this transformation jive well with the tangent line?
Why does this transformation jive well with the tangent line?
chezbgone
2015-03-27 21:23:54
B'C is a diameter
B'C is a diameter
xayy
2015-03-27 21:23:54
parallel?
parallel?
nosaj
2015-03-27 21:23:54
red line is parallel to tangent line?
red line is parallel to tangent line?
saagar
2015-03-27 21:23:54
tangents are perpendicular and so the two lines are parallel
tangents are perpendicular and so the two lines are parallel
copeland
2015-03-27 21:24:03
Like we said before, this is also a rotation by $\pi$ and then a rescaling.
Like we said before, this is also a rotation by $\pi$ and then a rescaling.
copeland
2015-03-27 21:24:05
A rotation by $\pi$ takes lines to parallel lines. The top and bottom are parallel lines. That means $B'$ and $C$ are antipodes. $BC$ is is a diameter!
A rotation by $\pi$ takes lines to parallel lines. The top and bottom are parallel lines. That means $B'$ and $C$ are antipodes. $BC$ is is a diameter!
copeland
2015-03-27 21:24:10
Now we have a cyclic quadrilateral in which one diagonal is a diameter. That is really awesome. Let me clean up the diagram for you:
Now we have a cyclic quadrilateral in which one diagonal is a diameter. That is really awesome. Let me clean up the diagram for you:
copeland
2015-03-27 21:24:15
copeland
2015-03-27 21:24:19
What do we know about the red region?
What do we know about the red region?
mathwrath
2015-03-27 21:24:48
16 times the area of the gray region
16 times the area of the gray region
24iam24
2015-03-27 21:24:48
it is 16 times the gray
it is 16 times the gray
High
2015-03-27 21:24:48
area of little triangle*16
area of little triangle*16
countingarithmetic
2015-03-27 21:24:48
16 times the size of the gray region
16 times the size of the gray region
acegikmoqsuwy2000
2015-03-27 21:24:48
its 16 times the grey region
its 16 times the grey region
saagar
2015-03-27 21:24:48
16 times the gray triangle?
16 times the gray triangle?
copeland
2015-03-27 21:24:51
Since the original grey regions were the same size and we dilated by a factor of 4, the red region is 16 times the size of the grey region.
Since the original grey regions were the same size and we dilated by a factor of 4, the red region is 16 times the size of the grey region.
copeland
2015-03-27 21:24:55
We also ought to know exactly what the dimensions of $\triangle AB'C$ are. Remember that $A,$ $B,$ and $C,$ are fixed. We were a little bit too quick when we abandoned the original figure. Let's circle back.
We also ought to know exactly what the dimensions of $\triangle AB'C$ are. Remember that $A,$ $B,$ and $C,$ are fixed. We were a little bit too quick when we abandoned the original figure. Let's circle back.
copeland
2015-03-27 21:25:03
What should we draw on the original figure to figure out where $A$ is?
What should we draw on the original figure to figure out where $A$ is?
copeland
2015-03-27 21:25:48
Tommy2000
2015-03-27 21:26:13
radii
radii
saagar
2015-03-27 21:26:13
draw a radius?
draw a radius?
copeland
2015-03-27 21:26:15
Let's connect the centers with radii:
Let's connect the centers with radii:
copeland
2015-03-27 21:26:16
copeland
2015-03-27 21:26:25
What kind of triangle is that?
What kind of triangle is that?
Dayranger
2015-03-27 21:26:41
haha, CIRCLE back
haha, CIRCLE back
copeland
2015-03-27 21:26:43
Thanks.
Thanks.
rcbg0713
2015-03-27 21:27:08
right
right
bluephoenix
2015-03-27 21:27:08
right triangle
right triangle
saagar
2015-03-27 21:27:08
im not sure, but it looks like the RIGHT triangle
im not sure, but it looks like the RIGHT triangle
copeland
2015-03-27 21:27:11
per force.
per force.
copeland
2015-03-27 21:27:19
I just dropped a perpendicular.
I just dropped a perpendicular.
copeland
2015-03-27 21:27:29
But read the problem. Think about the radii.
But read the problem. Think about the radii.
High
2015-03-27 21:27:51
3,4,5
3,4,5
swe1
2015-03-27 21:27:51
3-4-5
3-4-5
acegikmoqsuwy2000
2015-03-27 21:27:51
3,4,5 triangle
3,4,5 triangle
amburger66
2015-03-27 21:27:51
3,4,5
3,4,5
acegikmoqsuwy2000
2015-03-27 21:27:51
its a 3,4,5 right triangle
its a 3,4,5 right triangle
ryanyz10
2015-03-27 21:27:51
is it not a 3-4-5?
is it not a 3-4-5?
High
2015-03-27 21:27:51
3-4-5 right triangle
3-4-5 right triangle
danzhi
2015-03-27 21:27:51
3,4,5
3,4,5
DivideBy0
2015-03-27 21:27:51
3-4-5
3-4-5
bengals
2015-03-27 21:27:51
3-4-5?
3-4-5?
math_cool
2015-03-27 21:27:51
3-4-5
3-4-5
copeland
2015-03-27 21:27:53
That is a 3-4-5 triangle.
That is a 3-4-5 triangle.
copeland
2015-03-27 21:27:57
copeland
2015-03-27 21:28:14
What about this little triangle?
What about this little triangle?
copeland
2015-03-27 21:28:44
(The a-b-4 one.)
(The a-b-4 one.)
caressezhu
2015-03-27 21:29:11
3-4-5
3-4-5
amburger66
2015-03-27 21:29:11
also a 3,4,5
also a 3,4,5
saagar
2015-03-27 21:29:11
its proportions are 4/5 of the 3-4-5
its proportions are 4/5 of the 3-4-5
ryanyz10
2015-03-27 21:29:11
also a 3-4-5?
also a 3-4-5?
copeland
2015-03-27 21:29:25
It is also 3-4-5. What is $a$?
It is also 3-4-5. What is $a$?
ryanyz10
2015-03-27 21:29:39
16/5
16/5
acegikmoqsuwy2000
2015-03-27 21:29:39
16/5
16/5
derpyuniverse
2015-03-27 21:29:39
16/5
16/5
24iam24
2015-03-27 21:29:39
16/5
16/5
meeptopia
2015-03-27 21:29:39
16/5
16/5
malarm
2015-03-27 21:29:39
16/5
16/5
copeland
2015-03-27 21:29:41
$a$ is the "4 side" of a 3-4-5 triangle. The hypotenuse has length 4 so the side length is $\dfrac45\cdot4=\dfrac{16}5$.
$a$ is the "4 side" of a 3-4-5 triangle. The hypotenuse has length 4 so the side length is $\dfrac45\cdot4=\dfrac{16}5$.
copeland
2015-03-27 21:29:42
Likewise $b$ is the "3 side" so $b=\dfrac45\cdot3=\dfrac{12}5$.
Likewise $b$ is the "3 side" so $b=\dfrac45\cdot3=\dfrac{12}5$.
copeland
2015-03-27 21:29:48
What is $c$?
What is $c$?
24iam24
2015-03-27 21:30:54
16sqrt5/5
16sqrt5/5
acegikmoqsuwy2000
2015-03-27 21:30:54
$\frac{16sqrt 5}{5}$
$\frac{16sqrt 5}{5}$
ryanyz10
2015-03-27 21:30:54
16sqrt(5)/5
16sqrt(5)/5
amburger66
2015-03-27 21:30:54
(16sqrt5)5
(16sqrt5)5
copeland
2015-03-27 21:30:58
We have right triangle $APB'$ with base $a=\dfrac{16}5$ and height $b+r=\dfrac{12}5+4=\dfrac{32}5$. That is a $1-2-\sqrt5$ triangle so \[c=\dfrac{16}5\sqrt5=\dfrac{16}{\sqrt5}.\]
We have right triangle $APB'$ with base $a=\dfrac{16}5$ and height $b+r=\dfrac{12}5+4=\dfrac{32}5$. That is a $1-2-\sqrt5$ triangle so \[c=\dfrac{16}5\sqrt5=\dfrac{16}{\sqrt5}.\]
copeland
2015-03-27 21:31:04
What is $d$?
What is $d$?
ryanyz10
2015-03-27 21:32:24
8/sqrt(5)
8/sqrt(5)
mathwrath
2015-03-27 21:32:24
8sqrt5/5
8sqrt5/5
24iam24
2015-03-27 21:32:24
8sqrt5/5
8sqrt5/5
Tommy2000
2015-03-27 21:32:24
8sqrt5/5
8sqrt5/5
acegikmoqsuwy2000
2015-03-27 21:32:24
$\frac{8\sqrt 5} {5}$
$\frac{8\sqrt 5} {5}$
amburger66
2015-03-27 21:32:24
8sqrt5/5
8sqrt5/5
Rmink41
2015-03-27 21:32:24
I mean 8sqrt5
I mean 8sqrt5
tdeng
2015-03-27 21:32:24
8/sqrt5
8/sqrt5
AlisonH
2015-03-27 21:32:24
8\sqrt{5}/5
8\sqrt{5}/5
copeland
2015-03-27 21:32:27
We could similarly use Pythagoras, but notice that $\triangle AB'P\sim \triangle CAP$ so \[\dfrac{AC}{CP}=\dfrac{B'A}{AP}=\sqrt5.\] Therefore
\[d=(r-b)\sqrt5=\left(4-\frac{12}5\right)\sqrt5=\frac{8\sqrt5}5=\frac8{\sqrt5}.\]
We could similarly use Pythagoras, but notice that $\triangle AB'P\sim \triangle CAP$ so \[\dfrac{AC}{CP}=\dfrac{B'A}{AP}=\sqrt5.\] Therefore
\[d=(r-b)\sqrt5=\left(4-\frac{12}5\right)\sqrt5=\frac{8\sqrt5}5=\frac8{\sqrt5}.\]
copeland
2015-03-27 21:32:31
Back to our diagram:
Back to our diagram:
copeland
2015-03-27 21:32:34
copeland
2015-03-27 21:32:39
Let's give all these regions names. I'm also going to label the other 2 edge lengths.
Let's give all these regions names. I'm also going to label the other 2 edge lengths.
copeland
2015-03-27 21:32:40
acegikmoqsuwy2000
2015-03-27 21:32:58
yay colors!
yay colors!
copeland
2015-03-27 21:33:12
You'd think I could make less 80s colors but rgb space is weird space.
You'd think I could make less 80s colors but rgb space is weird space.
copeland
2015-03-27 21:33:13
Now what does our ratios statement tell us?
Now what does our ratios statement tell us?
tdeng
2015-03-27 21:33:35
what if someone is color blind
what if someone is color blind
copeland
2015-03-27 21:33:36
If you're colorblind, this diagram probably looks better.
If you're colorblind, this diagram probably looks better.
elf123
2015-03-27 21:34:45
1:16 blue to purple and green to light blue
1:16 blue to purple and green to light blue
ryanyz10
2015-03-27 21:34:45
w + x = 16(z + y)
w + x = 16(z + y)
math_cool
2015-03-27 21:34:45
AEB' = 16 AEC
AEB' = 16 AEC
copeland
2015-03-27 21:34:48
We know that $\dfrac{W+X}{Z+Y}=16$.
We know that $\dfrac{W+X}{Z+Y}=16$.
copeland
2015-03-27 21:34:49
But look at $Z$ and $W$. What do we know about how their areas are related?
But look at $Z$ and $W$. What do we know about how their areas are related?
High
2015-03-27 21:36:12
ratio of areas is BZ/ZC
ratio of areas is BZ/ZC
amburger66
2015-03-27 21:36:12
it's equal to CZ:ZB'
it's equal to CZ:ZB'
xayy
2015-03-27 21:36:12
the heights' ratio
the heights' ratio
Tommy2000
2015-03-27 21:36:12
ratio of the bases
ratio of the bases
copeland
2015-03-27 21:36:17
Since these two triangles have the same altitude to the diameter, their ratio is equal to the ratio of the lengths of the sections that $AE$ dissects $B'C$ into.
Since these two triangles have the same altitude to the diameter, their ratio is equal to the ratio of the lengths of the sections that $AE$ dissects $B'C$ into.
copeland
2015-03-27 21:36:29
(That point is not $Z$. I meant for $Z$ to be the area.)
(That point is not $Z$. I meant for $Z$ to be the area.)
copeland
2015-03-27 21:36:31
What else?
What else?
copeland
2015-03-27 21:36:55
What can we say about $X$ and $Y$?
What can we say about $X$ and $Y$?
Tommy2000
2015-03-27 21:37:20
Same for X and Y
Same for X and Y
ryanyz10
2015-03-27 21:37:20
same ratio!!!
same ratio!!!
meeptopia
2015-03-27 21:37:20
same thing?
same thing?
amburger66
2015-03-27 21:37:20
same ratio
same ratio
24iam24
2015-03-27 21:37:20
has the same ratio
has the same ratio
copeland
2015-03-27 21:37:22
The same argument works or $X$ and $Y$. Their areas are also in the same ratio as the ratio of the lengths of the two sections of $B'C$.
The same argument works or $X$ and $Y$. Their areas are also in the same ratio as the ratio of the lengths of the two sections of $B'C$.
copeland
2015-03-27 21:37:23
Specifically, we know that $\dfrac WZ=\dfrac XY$ is some constant $k$.
Specifically, we know that $\dfrac WZ=\dfrac XY$ is some constant $k$.
copeland
2015-03-27 21:37:25
And what can we conclude?
And what can we conclude?
math_cool
2015-03-27 21:38:24
Similar triangles: W = 16 Y and X = 16 Z
Similar triangles: W = 16 Y and X = 16 Z
bengals
2015-03-27 21:38:24
w/z = 16
w/z = 16
DivideBy0
2015-03-27 21:38:24
k = 16
k = 16
amburger66
2015-03-27 21:38:24
that ratio is 16?
that ratio is 16?
Tommy2000
2015-03-27 21:38:24
k=16
k=16
DivideBy0
2015-03-27 21:38:24
W/Z = 16
W/Z = 16
mathwrath
2015-03-27 21:38:24
k=16
k=16
caressezhu
2015-03-27 21:38:37
W+X=16Z+16Y
W+X=16Z+16Y
copeland
2015-03-27 21:38:38
We can conclude that \[16=\frac{W+X}{Z+Y}=\frac{kZ+kY}{Z+Y}=k.\] Therefore $k=16$ and the ratios $\dfrac WZ=\dfrac XY=16$.
We can conclude that \[16=\frac{W+X}{Z+Y}=\frac{kZ+kY}{Z+Y}=k.\] Therefore $k=16$ and the ratios $\dfrac WZ=\dfrac XY=16$.
copeland
2015-03-27 21:38:44
Cool.
Cool.
copeland
2015-03-27 21:38:45
How can we finish from here?
How can we finish from here?
copeland
2015-03-27 21:40:11
For example, we could go analytic geometry all the way. We know $A$ and we know the point where $B'C$ intersects $AE$ is $\dfrac{16}{17}$ of the way down. Therefore we could find $E$ by intersecting the circle with the line. That might be the easiest way but let's do something a little different.
For example, we could go analytic geometry all the way. We know $A$ and we know the point where $B'C$ intersects $AE$ is $\dfrac{16}{17}$ of the way down. Therefore we could find $E$ by intersecting the circle with the line. That might be the easiest way but let's do something a little different.
copeland
2015-03-27 21:40:22
We already have some lengths in this diagram. We also have area ratios. Let's just try to force out the rest of those areas.
We already have some lengths in this diagram. We also have area ratios. Let's just try to force out the rest of those areas.
copeland
2015-03-27 21:40:28
Remember that we have a cyclic quadrilateral. We haven't really used that very much yet. What is the nice property of cyclic quadrilaterals (beyond that they live in circles)?
Remember that we have a cyclic quadrilateral. We haven't really used that very much yet. What is the nice property of cyclic quadrilaterals (beyond that they live in circles)?
DivideBy0
2015-03-27 21:41:36
ptolomy's
ptolomy's
thequantumguy
2015-03-27 21:41:36
ptomely
ptomely
BPM14
2015-03-27 21:41:36
ABD = ACD
ABD = ACD
tdeng
2015-03-27 21:41:36
ptolemys theorem?
ptolemys theorem?
High
2015-03-27 21:41:36
Ptolemy's theorem
Ptolemy's theorem
acegikmoqsuwy2000
2015-03-27 21:41:36
ptolemy's theorem?
ptolemy's theorem?
24iam24
2015-03-27 21:41:36
supplementary angles opposite angles
supplementary angles opposite angles
lazboy
2015-03-27 21:41:36
Ptolemy?
Ptolemy?
copeland
2015-03-27 21:41:41
Here are two facts. Let's think about the angles statement.
Here are two facts. Let's think about the angles statement.
copeland
2015-03-27 21:41:42
The opposite angles are supplementary.
The opposite angles are supplementary.
copeland
2015-03-27 21:41:44
We used that $A$ and $E$ are right angles. We haven't used that $C$ and $B'$ are supplementary. Does that tell us anything about the area?
We used that $A$ and $E$ are right angles. We haven't used that $C$ and $B'$ are supplementary. Does that tell us anything about the area?
copeland
2015-03-27 21:42:53
Remember we can compute the area from angles. . .
Remember we can compute the area from angles. . .
24iam24
2015-03-27 21:43:29
area of triangle equals absin(x)/2
area of triangle equals absin(x)/2
Tommy2000
2015-03-27 21:43:29
sines are equal so use absinc/2
sines are equal so use absinc/2
vinayak-kumar
2015-03-27 21:43:29
i meant area of triangle with sines
i meant area of triangle with sines
copeland
2015-03-27 21:43:31
\begin{align*}
[\triangle AB'E]=\frac12AB'\cdot B'E\sin(B')\\
[\triangle AEC]=\frac12AC\cdot CE\sin(C)
\end{align*}
\begin{align*}
[\triangle AB'E]=\frac12AB'\cdot B'E\sin(B')\\
[\triangle AEC]=\frac12AC\cdot CE\sin(C)
\end{align*}
copeland
2015-03-27 21:43:32
However. . .
However. . .
24iam24
2015-03-27 21:43:48
sin(x)=sin(180-x)
sin(x)=sin(180-x)
countingarithmetic
2015-03-27 21:43:48
same sine
same sine
High
2015-03-27 21:43:48
the sines are equal
the sines are equal
copeland
2015-03-27 21:43:51
Supplementary angles have equal sines. Therefore
\begin{align*}
\frac{[\triangle AB'E]}{AB'\cdot B'E}=\frac12\sin(B')\\
\frac{[\triangle ACE]}{AC\cdot CE}=\frac12\sin(C)
\end{align*}
so
\[
\frac{[\triangle AB'E]}{AB'\cdot B'E}=\frac{[\triangle ACE]}{AC\cdot CE}
\]
Supplementary angles have equal sines. Therefore
\begin{align*}
\frac{[\triangle AB'E]}{AB'\cdot B'E}=\frac12\sin(B')\\
\frac{[\triangle ACE]}{AC\cdot CE}=\frac12\sin(C)
\end{align*}
so
\[
\frac{[\triangle AB'E]}{AB'\cdot B'E}=\frac{[\triangle ACE]}{AC\cdot CE}
\]
copeland
2015-03-27 21:43:59
What else can we use?
What else can we use?
Tommy2000
2015-03-27 21:44:35
The ratio of the areas is 16
The ratio of the areas is 16
24iam24
2015-03-27 21:44:35
also ratio of areas is 16
also ratio of areas is 16
countingarithmetic
2015-03-27 21:44:35
AB' = 2AC
AB' = 2AC
rlz
2015-03-27 21:44:35
We know the ratio of the areas
We know the ratio of the areas
copeland
2015-03-27 21:44:38
We know the ratios of those areas:\[16=
\frac{[\triangle AB'E]}{[\triangle ACE]}
=\frac{AB'\cdot B'E}{AC\cdot CE}
=\frac{\frac{16}{\sqrt5}\cdot x}{\frac{8}{\sqrt5}\cdot y}
=\frac{2x}{y}.\]
We know the ratios of those areas:\[16=
\frac{[\triangle AB'E]}{[\triangle ACE]}
=\frac{AB'\cdot B'E}{AC\cdot CE}
=\frac{\frac{16}{\sqrt5}\cdot x}{\frac{8}{\sqrt5}\cdot y}
=\frac{2x}{y}.\]
copeland
2015-03-27 21:44:44
That means $x=8y$.
That means $x=8y$.
copeland
2015-03-27 21:44:46
Where do we get another relation between $x$ and $y$?
Where do we get another relation between $x$ and $y$?
countingarithmetic
2015-03-27 21:45:42
$x^2 + y^2 = 64$
$x^2 + y^2 = 64$
24iam24
2015-03-27 21:45:42
x^2+y^2=64
x^2+y^2=64
Tommy2000
2015-03-27 21:45:42
pythagorean theorem!
pythagorean theorem!
Dayranger
2015-03-27 21:45:42
x^2 + y^2 = 64
x^2 + y^2 = 64
acegikmoqsuwy2000
2015-03-27 21:45:42
x^2+y^2 = 64
x^2+y^2 = 64
Tommy2000
2015-03-27 21:45:42
x^2+y^2=64
x^2+y^2=64
copeland
2015-03-27 21:45:45
The Pythagorean Theorem on $\triangle B'EC$ gives us $x^2+y^2=8^2$
The Pythagorean Theorem on $\triangle B'EC$ gives us $x^2+y^2=8^2$
copeland
2015-03-27 21:45:47
Solving that tells us \[64=x^2+y^2=\left(8y\right)^2+y^2=65y^2.\]
Solving that tells us \[64=x^2+y^2=\left(8y\right)^2+y^2=65y^2.\]
copeland
2015-03-27 21:45:51
That gives us $y$.
That gives us $y$.
copeland
2015-03-27 21:45:53
Finally we want to compute the area $Z+Y$. That's hard. What can we compute?
Finally we want to compute the area $Z+Y$. That's hard. What can we compute?
24iam24
2015-03-27 21:46:30
X+Y
X+Y
copeland
2015-03-27 21:46:36
And what's so great about X+Y?
And what's so great about X+Y?
BPM14
2015-03-27 21:47:06
they have a ratio
they have a ratio
elf123
2015-03-27 21:47:06
it equals 17y
it equals 17y
copeland
2015-03-27 21:47:09
Right.
Right.
copeland
2015-03-27 21:47:18
$X+Y$ lets us compute
\[17(Z+Y)=(Z+Y)+(X+W)=(Z+W)+(X+Y).\]
$X+Y$ lets us compute
\[17(Z+Y)=(Z+Y)+(X+W)=(Z+W)+(X+Y).\]
copeland
2015-03-27 21:47:23
What is $Z+W$?
What is $Z+W$?
24iam24
2015-03-27 21:48:03
64/5
64/5
Tommy2000
2015-03-27 21:48:03
64/5
64/5
acegikmoqsuwy2000
2015-03-27 21:48:03
64/5
64/5
copeland
2015-03-27 21:48:06
That's a right triangle. Its area is $\dfrac12\cdot\dfrac{16}{\sqrt5}\cdot\dfrac{8}{\sqrt5}=\dfrac{64}5$.
That's a right triangle. Its area is $\dfrac12\cdot\dfrac{16}{\sqrt5}\cdot\dfrac{8}{\sqrt5}=\dfrac{64}5$.
copeland
2015-03-27 21:48:07
What is $X+Y$?
What is $X+Y$?
copeland
2015-03-27 21:48:47
(Remember that $x=8y$ and $64=65y^2$.)
(Remember that $x=8y$ and $64=65y^2$.)
tdeng
2015-03-27 21:49:36
256/65
256/65
Tommy2000
2015-03-27 21:49:36
256/65
256/65
mathwizard888
2015-03-27 21:49:36
256/65
256/65
copeland
2015-03-27 21:49:40
That's also a right triangle. Its area is
\begin{align*}
\frac12xy
&=\frac12\cdot8y^2\\
&=4\cdot\frac{64}{65}\\
&=\frac{256}{65}.
\end{align*}
That's also a right triangle. Its area is
\begin{align*}
\frac12xy
&=\frac12\cdot8y^2\\
&=4\cdot\frac{64}{65}\\
&=\frac{256}{65}.
\end{align*}
copeland
2015-03-27 21:49:47
So what is the answer?
So what is the answer?
caressezhu
2015-03-27 21:50:46
129
129
Tommy2000
2015-03-27 21:50:46
64/65
64/65
derpyuniverse
2015-03-27 21:50:46
64/65
64/65
24iam24
2015-03-27 21:50:46
add them and /17
add them and /17
copeland
2015-03-27 21:50:50
The answer is
\[
\frac1{17}\left(\frac{64}{5}+\frac{256}{65}\right)
=\frac{64}{17}\left(\frac{13}{65}+\frac{4}{65}\right)
=\frac{64\cdot17}{17\cdot65}
=\frac{64}{65}.
\]
The answer is
\[
\frac1{17}\left(\frac{64}{5}+\frac{256}{65}\right)
=\frac{64}{17}\left(\frac{13}{65}+\frac{4}{65}\right)
=\frac{64\cdot17}{17\cdot65}
=\frac{64}{65}.
\]
copeland
2015-03-27 21:50:51
The final answer is $64+65=\boxed{129}$.
The final answer is $64+65=\boxed{129}$.
copeland
2015-03-27 21:51:12
Alright, next problem. . .
Alright, next problem. . .
nosaj
2015-03-27 21:51:14
we are 1/1 of the way done!
we are 1/1 of the way done!
copeland
2015-03-27 21:51:20
Oh wait. It's time to go home now.
Oh wait. It's time to go home now.
copeland
2015-03-27 21:51:40
Thanks for coming, I had a great time.
Thanks for coming, I had a great time.
_--__--_
2015-03-27 21:51:57
we are already at home (most of us)
we are already at home (most of us)
countingarithmetic
2015-03-27 21:51:57
Thank you!
Thank you!
lazboy
2015-03-27 21:51:57
Thank you, Mr. Copeland!
Thank you, Mr. Copeland!
xayy
2015-03-27 21:51:57
bye
bye
Dayranger
2015-03-27 21:51:57
ME TOO!!!
ME TOO!!!
BPM14
2015-03-27 21:52:23
Can we use you on our next AIME?
Can we use you on our next AIME?
copeland
2015-03-27 21:52:25
Maybe not the best idea. I did not include (most of) the arithmetic errors I made in today's class.
Maybe not the best idea. I did not include (most of) the arithmetic errors I made in today's class.
DivideBy0
2015-03-27 21:52:35
thanks!!
thanks!!
czhu000
2015-03-27 21:52:35
Thanks, bye!
Thanks, bye!
1915933
2015-03-27 21:52:35
Thank you! Have a good evening!
Thank you! Have a good evening!
math_cool
2015-03-27 21:52:35
thanks!
thanks!
ryanyz10
2015-03-27 21:52:35
thanks!
thanks!
caressezhu
2015-03-27 21:52:35
Thank you, I enjoy it!
Thank you, I enjoy it!
malarm
2015-03-27 21:52:35
thank YOU
thank YOU
BPM14
2015-03-27 21:52:35
Thanks!
Thanks!
Guendabiaani
2015-03-27 21:52:35
Thanks!
Thanks!
Dayranger
2015-03-27 21:52:35
Thank you, Mr. Copeland
Thank you, Mr. Copeland
bengals
2015-03-27 21:52:35
thank you
thank you
acegikmoqsuwy2000
2015-03-27 21:52:39
AoPS is my home.
AoPS is my home.
Copyright © 2024 AoPS Incorporated. This page is copyrighted material. You can view and print this page for your own use, but you cannot share the contents of this file with others.